Sunteți pe pagina 1din 123

Universidad La Salle.

Facultad Mexicana de Medicina.


Curso de Extensin Universitaria para la Preparacin del Examen Nacional para
Aspirantes a Residencias Mdicas.
Examen de Diagnstico 2015
Modalidad Semipresencial

1.- Paciente que acude a revisin por 8 semanas de amenorrea, asintomtica. En la


ecografa vaginal se observa una tumoracin anexial izquierda, sin evidencia de embrin.
En el tero no se visualiza saco gestacional. La actitud ms correcta en ste caso es:

a)
b)
c)
d)

Ingreso para legrado.


Culdocentsis.
Ingreso y determinacin de niveles de B-HGC.
Salpingectoma urgente.

EMBARZO ECTOPICO
FACTORES DE RIESGO
Anomalas tubarias

Enfermedad plvica inflamatoria 30-50%


Adherencias (endometriosis)
Antecedente de salpingoclasia
Alteraciones anatmicas

EMBARAZO ECTPICO
< 6500mUI

USG normal

Cuantificacin seriada
de HGC

Descenso

Dilatacin y legrado
64

Estabilizacin o
incremento

Laparoscopia Dx

Aumento

Repetir USG
27/01/2014

Diagnstico:

HGC > 1500 U/L


Sin evidencia de saco
Lquido libre en fondo de saco
Sensibilidad 63%
Especificidad 100%

Ankum WM., Mol Bw.,Van der Veen F.,Bossuyt PM.


Risk factors for ectopic pregnancy:a meta-analysis. Fertil Steril 1996:65:1093-9.

2.- Recibe en la sala de urgencia a una paciente que inicia con convulsiones por
preclampsia usted decide administrar el siguiente frmaco ya que es el de eleccin en
sta patologa:

a) Diacepam.
b) Fenitona.
c) Sulfato de magnesio.
d) Donadores de xido ntrico.
Manejo de la Preeclampsia
1. Manejo ambulatorio: HTA sin proteinuria significativa, se recomienda el reposo en
cama. Monitoreo de TA, peso, presencia de protenas en orina. Ecografas peridicas
para ver el feto y evaluar posibles retardo de crecimiento.
2. Manejo hospitalario: para mujeres con HTA inducida por el embarazo y 2+ o ms o
proteinuria significativa y en quienes fall el manejo ambulatorio.
3. Laboratorio y evaluacin del peso: debe realizarse diariamente. Evaluacin de la
dinmica fetal. Monitoreo de sntomas como cefalea, alteraciones visuales y dolor
epigstrico.
4. El parto es el tratamiento de eleccin: el cual debe realizarse cuando el feto est
maduro pero puede realizarse en forma temprana si la salud de la madre est en peligro o
si hay evidencia de distress fetal. El parto est indicado cuando la paciente cumple con
los criterios de preeclampsia severa. Betametasona 12.5 mg IM dos veces por da puede
estimular la maduracin de los pulmones fetales.
5. Terapia antihipertensiva: est indicada slo si la TA es persistentemente > 160/110 ,
es importante disminuir la TA hasta una diastlica de 90 a 100 mmHg porque la presin
normal podra resultar en hipoperfusin de la placenta. Los diurticos nunca estn
indicados, estas pacientes ya son hipovolmicas. Los IECA no deben ser usados durante
el embarazo. Las medicaciones de largo plazo, incluyen alfa metildopa, atenolol y
labetalol.

6. Terapia anticonvulsivante:
A- Profilaxis de las convulsiones: est indicada en todas las pacientes pre-eclmpticas
durante el trabajo de parto y el parto y por un mnimo de 24 hrs luego del mismo. Algunos
mantienen la terapia con magnesio hasta que comienza la diuresis. El Sulfato de
Magnesio es la droga de eleccin. La dosis profilctica es de 4 a 6 g de sulfato de
magnesio IV y contina con 2 g c/ hora.
B- Tratamiento de las convulsiones: Sulfato de Magnesio 1 g/min IV hasta controlar
las convulsiones hasta un mximo de 4 a 6 g. El nivel teraputico es de 4 meq/l. Toxicidad
del magnesio: ausencia de reflejo patelar, debilidad muscular, parlisis respiratoria y
depresin cardaca, 10 ml al 10 % de gluconato de calcio puede ser administrada IV. La
terapia con sulfato de magnesio contina por lo menos 24 horas en el post parto, la
terapia puede detenerse si la excrecin urinaria es > 200 ml/h por cuatro horas
consecutivas.
C- Prevencin: 81 mg de aspirina diarios pueden ser administrados luego del primer
trimestre en mujeres con hipertensin crnica o historia previa de preeclampsia, sin
embargo la eficacia de esta indicacin ha sido cuestionada.

Myers JE, Baker PN. Hupertensive diseases and eclampsia. Curr Opin Obstet Gynecol
2002; 14: 119-125
2. Tierney, McPhee, Papadakis. Diagnstico clnico y tratamiento 2003. 38 ed, Mxico,
Manual Moderno, 2003: 770-773
3. Wilson MI, Goodwin TM, Pan VI, Ingles SA. Molecular epidemiology of preeclampsia.
Obstet and Gynecol Survey 2003; 58(1):39-66
4. Burrow GM. Complicaciones mdicas durante el embarazo. 4 ed, Mxico, McGraw-Hill
panamericana: 1996: 1-25
5. Guyton AC, Hall JE. Embarazo y lactancia en: Tratado de fisiologa mdica, 10 ed,
Mxico, McGraw-Hill Interamericana 2001: 1135-45
6. Vaticon D. Fisiologa de la fecundacin, embarazo, parto y lactancia, en: Tresguerres
JAF. Fisiologa Humana. Mxico, Interamericana McGraw-Hill, 1992: 1086-1109
7. Pridjian G, Puschett JB. Preeclampisa. Part 1: Clinical and Pathophysiologic
Considerations. Obstet and Gynecol Survey 2002; 57 (9): 598-618
8. Pridjian G, Puschett JB. Preeclampisa. Part I1: Experimental and Genetic
Considerations. Obstet and Gynecol Survey 2002; 57 (9): 619-40
9. IMSS. Embarazo de alto riesgo. Gua diagnstica teraputica. Rev Med IMSS 1998;
36(1):45-60

3.- Masculino de 7 meses de edad presenta una historia de estreimiento que ha venido en
incremento, con antecedente de dos impactaciones fecales, durante ste mes expulsa
heces duras una vez a la semana. Su exploracin fsica con mal incremento ponderal. El
diagnstico ms probable del menor es:

a)
b)
c)
d)

Hipotiroidismo
Envenenmiento por plomo
Estreimiento funcional
Enfermedad de Hirschprung

La enfermedad de Hirschsprung (EH) es considerada una enfermedad congnita


caracterizada por una ausencia de clulas ganglionares en el plexo mientrico de Auerbach
y en el submucoso de Meissner, en el recto y otros segmentos del colon en forma
ascendente (de caudal a ceflico) (1). Esta alteracin produce una anormalidad de la
motilidad intestinal, que se manifiesta ms frecuentemente como una obstruccin intestinal
(2).
La EH puede ser clasificada segn el segmento intestinal comprometido. As puede
dividirse en: 1.- Segmento corto: cuando no compromete ms all de la unin
rectosigmoidea;
2.- Ultracorto: si slo afecta esfnter interno o algunos centmetros prximos a dicho
esfnter;
3.-Segmento largo cuando el segmento aganglinico afecta ms all de la unin
rectosigmoidea. Otros autores slo distinguen dos tipos: segmento corto y segmento largo,
siendo controversial la existencia de EH de segmento ultracorto, ya que esta compromete
menos de 5 cm del recto distal.
La mayora de los casos corresponde a EH de segmento corto (75 a 80%), una dcima
parte de ellos sera un aganglionismo ultracorto. El 20% restante pertenecera a EH de
segmento largo, incluyndose en este grupo aquellos que afectan los segmentos
proximales al ngulo esplnico.
La EH forma parte de los trastornos conocidos como disganglionismos que incluyen
tambin el hipoganglionismo y la displasia neuronal intestinal.
Expresin Clnica
Dentro de los sntomas que permiten una sospecha precoz, se encuentra el estreimiento o
constipacin, definida en el recin nacido como el retraso en la eliminacin de meconio
mayor a 48 horas asociada a distensin abdominal, y en los nios mayores como
deposiciones infrecuentes de consistencia aumentada (. El 98% de los lactantes elimina el
meconio en las primeras 48 horas de vida. Los prematuros eliminan ms tardamente el
meconio, pero la EH es rara en prematuros. De los pacientes con EH, slo el 60% elimina
el meconio despus de las 48 horas, por lo que este signo no es patognomnico de la
enfermedad.
La mayora de los nios que presentan aganglionosis congnita, son sintomticos los
primeros das o las primeras semanas luego del nacimiento (2). Alrededor de dos tercios de
los pacientes presenta sntomas dentro de los tres primeros meses de vida y 80%
desarrolla sntomas dentro del primer ao de vida. Slo un 10% de los pacientes inicia
sntomas entre los 3 y 14 aos de edad y en general se trata de pacientes con enfermedad
de segmento ultracorto. (13)
Los recin nacidos y lactantes pequeos presentan con frecuencia signos de obstruccin
intestinal, distensin abdominal, vmitos biliosos e intolerancia a la alimentacin. La
inspeccin anal y la radiografa pueden orientarnos hacia una causa mecnica de
obstruccin, pero no descarta EH. Si la obstruccin no tiene una causa mecnica, adems
de pensar en una EH, debe plantearse el diagnstico diferencial con hipotiroidismo,
insuficiencia suprarrenal, hipokalemia, hipercalcemia, hipomagnesemia, y en casos
excepcionales alteraciones neuromusculares.
Cuando la sintomatologa es poco evidente,
Puede presentarse como un cuadro de constipacin crnica, con historia de dificultad en la
eliminacin de deposiciones, masas fecales palpables en fosa ilaca izquierda y un tacto

rectal en que no se encuentran deposiciones en la ampolla rectal y esfnter anal


hipertnico. En muchas ocasiones la estimulacin rectal provoca salida explosiva de heces
lquidas de olor ftido (17). Por lo tanto, frente a pacientes con constipacin crnica, en los
cuales se ha descartado causa mecnica de obstruccin intestinal, que no cede a las
medidas dietticas ni farmacolgicas, debe plantearse el diagnstico de
EH. Tambin puede encontrarse dilatacin de asas intestinales, adelgazamiento de la
pared abdominal, alteraciones de la nutricin y el crecimiento.
En nios mayores, los sntomas ms comunes incluyen constipacin crnica progresiva,
impactacin fecal recurrente, mal incremento ponderal y malnutricin.

Rev. Ped. Elec. 2008, Vol 5, N 1. ISSN 0718-0918


Servicio Salud Metropolitano Norte
Facultad de Medicina Hospital Clnico de Nios
Departamento de Pediatra y Ciruga Infantil Roberto Del Ro
Referencias:
1. De Manueles J. Enfermedad de Hirschsprung.
Protocolos diagnsticos y teraputicos en pediatra. Sociedad Espaola de Pediatra. Pag.
56-60.
2. Feldmon T., Wershil B. Hirschsprung Disease. Pediatrics in review. Vol 23. N 11, August
2003.
3. Luis L.A., Encinas J.L., Avila L.F., et cols.
Enfermedad de Hirschsprung: enseanzas de los ltimos 100 casos. Cir Pediatr 2006;
19:177181.
4. J.M. Gil-Vener y cols. Diagnstico dieferncial de Hirschsprung-neurodisplasia intestinal.
Fiabilidad de las pruebas diagnsticas. Cir Pediatr 2006; 19: 91-94.
5. M. Lpez, y cols. ndices de fiabilidad de la manometra anorrectal para el diagnstico de
la enfermedad de Hirschsprung en cualquier edad. Cir Pediatr 2005; 18:13-16
6. Hernndez F., Rivas S., vila L.F., Daz M., ET cols. Aganglionismos extensos.
Tratamiento y resultados a largo plazo. Cir Pediatr 2003; 16: 54-57.
7. Goulet O. y cols. Intestinal transplantation in children: preliminary experience en Paris.
JPEN
J Parenter Enteral Nutr 1999; 23 (5 Suppl)
8. Pea A. Enfermedad de Hirschsprung. Los avances y las preguntas no contestadas. Cir
Pediatr 2002: 15:46-47
9. Polliotto S, Heinen F, Anduna G, Korman R. Evaluacin de resultado a tres aos de
nuestra primera experiencia en el tratamiento laparoscpico de la enfermedad de
Hirschsprung. Cir Pediatr 2001; 14: 85-87

4.- Se trata de un recin nacido al que se decide colocar una sonda orogstrica, al darnos
cuenta de que dicha sonda no avanza y apreciar en una radiografa toracoabdomial la
ausencia de aire en el intestino, Sospechamos de?

a) Atresia de esfago tipo II y V


b) Atresia de esfago tipo III y IV
c) Hernia diafragmtica congnita
d) Atresia de esfago tipo I y II.

. Consiste en la interrupcin de la luz esofgica con o sin comunicacin con la va area.


No existe an una definicin precisa del trmino "long gap". De acuerdo a nuestra
experiencia se trata de aquellas AE, independientemente del tipo, caracterizadas por una
distancia entre cabos esofgicos lo suficientemente grande como para imposibilitar la
realizacin de una anastomosis primaria trmino-terminal.
De acuerdo a nuestro criterio, basado en la clasificacin de Ladd, las atresias de esfago
pueden clasificarse en los tipos I, II, III, IV y V
1). La Tipo I es la atresia de esfago aislada sin fstula traqueoesofgica o atresia pura.
No tiene comunicacin con la va area. Representa aproximadamente el 5% de los casos
observados y es la segunda en frecuencia. Puede diagnosticarse con ms facilidad en el
embarazo, que cursa tpicamente con polihidramnios y ausencia de imagen gstrica en las
ecografas prenatales. Al nacer, los neonatos presentan el abdomen excavado por falta de
pasaje de aire al intestino. En todos los casos ambos cabos esofgicos se encuentran muy
separados entre s.
La Tipo II es una forma muy rara de atresia de esfago que representa del 1 al
3% de los casos. Existe una fstula traqueoesofgica desde el cabo superior del esfago a
la trquea cervical. Al igual que en las Tipo I, no hay pasaje de aire al intestino distal y
ambos extremos se encuentran muy alejados entre s.
La Tipo III es la forma observada en el 90% de los pacientes. Comnmente se la conoce
como sinnimo de atresia de esfago. Se caracteriza por una bolsa esofgica
comunicacin entre el extremo distal esofgico y la trquea (fstula trqueoesofagica al
cabo inferior), que puede nacer a cualquier altura de la trquea y excepcionalmente,
tambin en los bronquios. En estos casos el embarazo no cursa tpicamente con
polihidramnios y suele verse una imagen gstrica normal en las ecografas prenatales. Al
nacer los neonatos suelen presentar el abdomen distendido por el gran pasaje de aire que
existe desde la va area hacia el estmago e intestino. La distancia entre ambos cabos es
por lo general inferior a dos o tres vrtebras. La coexistencia de atresia de esfago Tipo III
con atresia duodenal, intestinal o malformaciones anorrectales constituye un cuadro de
urgencia; el gran flujo de aire desviado hacia el sistema digestivo a travs de la fstula
traqueoesofgica inferior provoca distensin abdominal severa y eventualmente, incluso,
perforacin intestinal.
La Tipo IV se caracteriza por una fstula trqueoesofagica en ambos cabos esofgicos. Su
observacin es poco frecuente. Representa menos del 2% de los casos. La fstula superior
suele encontrarse por accidente cuando se efecta la correccin quirrgica inicial. En
ocasiones pasa inadvertida por el cirujano y su diagnstico es efectuado en forma tarda.

La Tipo V no es en realidad una atresia de esfago. El esfago se encuentra permeable y


es de buen calibre.
En estos pacientes lo que se observa es una fstula traqueoesofgicaaislada, generalmente
de ubicacin cervical o torcica alta. El diagnstico se realiza excepcionalmente en el
periodo neonatal ya que los sntomas suelen aparecer en la infancia.

Bibliografa: Urgencias en Pediatra, Interamericana.McGraw Hill. Captulo: Urgencias


Mdico Quirrgicas, Seccin XXIII, pg. 774-778.
Operative Pediatric Surgery. Moritz M. Ziegler. International Edition, pg. 349- 354.
Ciruga Peditrica, Ashcraft - Holder Interamericana.McGraw Hill pg. 257 a 277.

5.- En nuestro pas consideramos en los enfermos con diabetes mellitus tipo 2 como un
criterio de metas de control metablico el siguiente:
a) Glucemia en ayuno 70-130 (mg/dl), glucemia postprandial de 2 hrs. < 180
(mg/dl), colesterol total < 200 (mg/dl), triglicridos en ayuno < 150 (mg/dl) y
colesterol HDL > 45 (mg/dl)
b) Glucemia en ayuno 100- 115 (mg/dl), glucemia postprandial de 2 hrs. < 200
(mg/dl), colesterol total < 200 (mg/dl), triglicridos en ayuno < 150 (mg/dl) y
colesterol HDL > 40 (mg/dl)
c) Glucemia en ayuno > 126 (mg/dl), glucemia postprandial de 2 hrs. > 200 (mg/dl),
colesterol total < 200 (mg/dl), triglicridos en ayuno > 150 (mg/dl) y colesterol HDL
> 40 (mg/dl)
d) Glucemia en ayuno < de 90 (mg/dl), glucemia postprandial de 2 hrs. < 200 (mg/dl),
colesterol total < 200 (mg/dl), triglicridos en ayuno < 150 (mg/dl) y colesterol HDL
> 40 (mg/dl)

METAS DE CONTROL
GLUCOSA CAPILAR PREPRANDIAL
70-130 mg/dL

GLUCOSA 2 HORAS POSTPRANDIAL


Sangre total: < 180 mg/dL

METAS DE CONTROL
Control metablico
Colesterol total

< 200

Colesterol-LDL

< 100

Triglicridos

< 150

Colesterol-HDL

> 45 hombres
> 55 mujeres

Tensin arterial: 140/80

6.- Se trata de paciente femenino de 42 aos diagnosticado con sndrome de Cushing al


encontrar una ACTH muy baja o suprimida, la primera posibilidad de diagnstico es:

a)
b)
c)
d)

Enfermedad hipotlamo-hipofisiaria
Hiperplasia suprarrenal congnita
Adenoma hipofisiario
Adenoma suprarreanal

Aproximadamente 20 a 25% de los pacientes con sndrome de Cushing tienen una


neoplasia suprarrenal. En la mayora de estos casos existe una produccin autnoma de
cortisol por la neoplasia, lo que conduce a descenso en los niveles de ACTH a rangos
indetectables o menores a 2 pmol/L o 10 pg/ml, por retroalimentacin negativa de la
secrecin de ACTH.
Williams GH, Dluhy RG. Enfermedades de la corteza suprarrenal. En Jameson JL (ed):
Harrison. Endocrinologa. 1a ed. Madrid. MacGraw-Hill Espaa, 2006: 126-127.

7.- Se realiza una endoscopia digestiva a un paciente de 57 aos, se reporta el


diagnstico de sospecha de esfago de Barrett. Entre las siguientes, la conducta ms
adecuada con ste paciente es:
a)

Indicar tratamiento mdico con inhibidor de la bomba de protones 40 mg da en


ayuno durante 1 ao y regresar a valoracin.
b) Esperar el resultado de la biopsia antes de tomar decisiones, para corroborar
y tener seguridad del diagnstico
c) Prepararlo para la realizacin de funduplicatura.
d) Realizar esofaguectoma.

Esfago de Barret.
La Enfermedad por Reflujo Gastroesofgico es una de las causas ms frecuentes de
consulta mdica en la prctica diaria y esta se puede acompaar de Esofagitis por Reflujo
(es la inflamacin del esfago causada por el reflujo del contenido gstrico al esfago), La
Esofagitis por Reflujo se diagnostica mediante Endoscopa, y este procedimiento permite
la biopsia, la cual nos permite confirmar la presencia de Esfago de Barret, una
complicacin poco frecuente pero de gran relevancia clnica.
El Esfago de Barret es una lesin premaligna, que predispone al Adenocarcinoma
Esofgico, patologa neoplsica que ha aumentado su incidencia en los ltimos aos,
incluso desplazando al Carcinoma Epidermoide.

DIAGNOSTICO.
IPresentacin Clnica. La presencia de EB no provoca sntomas por s mismo. Ms
bien, los sntomas son una consecuencia de la Enfermedad por Reflujo Gastro-Esofgico
(ERGE) o sus complicaciones. La mayora de los pacientes tienen antecedentes
prolongados de sntomas de RGE como pirosis y regurgitacin (2, 6, 10).
Otros pacientes, aproximadamente la tercera parte, se presentan oligo asintomticos (2,
8, 13), lo que sugiere una disminucin de la sensibilidad al reflujo cido por parte del
epitelio de Barret. De hecho, ms del 90% de los pacientes con EB no buscan atencin
mdica y el trastorno pasa inadvertido hasta que el proceso se complica por el desarrollo
de cncer, debutando con disfagia (por formacin de estenosis) o con hemorragia
digestiva (por ulceraciones profundas de la mucosa lesionada). (10)
II- Fibroendoscopa Digestiva Alta con toma de Biopsia. La endoscopia con biopsias
dirigidas constituye el patrn oro para el diagnstico del EB (2, 6). Sin embargo, debido a
la gran cantidad de pacientes con RGE y la baja frecuencia de EB, no parece costoefectivo el realizar endoscopia a todos ellos (13). Se ha sugerido que los siguientes
pacientes con RGE deben ser sometidos a una endoscopia (9):
RGE complicado (disfagia, estenosis, lcera, hemorragia);
RGE con esofagograma que muestre patrn reticular o seudomembranas;
RGE con sintomatologa persistente a pesar del tratamiento;
RGE asociado a esclerodermia.
Se sospecha EB en la endoscopia por la presencia de epitelio color naranja a rojo,
aterciopelado, de tipo gstrico; que contrasta con el color rosado blanquecino del epitelio
esofgico normal (1, 2, 4, 8). La lnea de transicin entre ambos epitelios (cambio
mucoso) puede ser regular (circunferencial) , ms frecuentemente, irregular (en forma de
lengetas o islotes)(1, 2, 8,). La extensin de la metaplasia tambin es variable, pudiendo
abarcar desde 2 cm de longitud a partir de la unin gastro-esofgica (UGE) o puede
extenderse incluso hasta el esfago cervical (1).
Para confirmar el diagnstico debe obtenerse la biopsia. Las muestras deben tomarse,
fundamentalmente, del lmite de la UGE y hasta 1 2 cm por debajo de la misma, ya que
esta es la zona de mayor riesgo de desarrollar un Adenocarcinoma, y es en esta parte
ms proximal del segmento metaplsico donde se sita, principalmente, el epitelio
columnar especializado (2). Por supuesto que deben tomarse muestras para biopsias de
todo el segmento metaplsico y, especialmente, de aquellas zonas en las que se observe
alguna alteracin macroscpica.
Microscpicamente pueden identificarse tres tipos de epitelio de Barret (1, 2, 4, 8)
Metaplasia gstrica fndica: es similar al epitelio del cuerpo o del fondo gstrico, y
presenta clulas parietales y principales;
Metaplasia gstrica cardiaca o transicional: como el epitelio del cardias gstrico, que
exhibe acmulos profundos de glndulas mucosas y criptas; y
Metaplasia tipo intestinal o tambin llamado epitelio columnar especializado: que tiene
caractersticas de la mucosa gstrica y de la intestinal, que presenta clulas
caliciformes dispersas entre las clulas cilndricas.

BIBLIOGRAFIA
1.

2.
3.

4.
5.

6.

7.

8.

9.

Badaloni AE. Enfermedad por reflujo gastroesofgico. En: Ferraina P, Oria A, ed.
Ciruga de Mitchans, Vol 1. 5 edicin. Buenos Aires, Argentina. Editorial El Ateneo.
392-399.
Cucarella JF. Esfago de Barret. Gaceta de la sociedad espaola de patologa
digestiva [en lnea] 1999. [fecha de acceso 10 de noviembre de 2004] . URL.
Beers MH. Trastornos del esfago. Cncer esofgico. En: El Manual Merck, 17
edicin. Edicin electrnica en CD-ROM. Madrid, Espaa: Ediciones Harcourt: 1999;
seccin 3, cap 20.
McGarrity TJ. Barret`s esophagus: the continuing conundrum. Surveillance should be
confined to the surgically fit. BJM 2000; 321: 1238-1239
Heading R. Es el seguimiento endoscpico del esfago de Barret una prdida de
tiempo y esfuerzo? XXX Congreso Chileno de Gastroenterologa. Medwave. Ao 4. N
2. [en lnea] Edicin Marzo 2004. [fecha de acceso 28 de diciembre de 2004]. URL.
Murra Saca JA. Esfago de Barret. Atlas de Video Endoscopia Gastrointestinal de El
Salvador.[en lnea] 2004 [fecha de acceso 10 de noviembre de 2004] . URL.
Disponible en:
Ayre AM, Benitez Fernandez A, Cocco JE, y col. Tratamiento del cncer de esfago:
revisin. Revista de Postgrado de la VI Ctedra de Medicina [en lnea] 2003 [fecha de
acceso: 15 de enero de 2005]; 126: 37-41. URL. Disponible en:
Crawford JM. El tracto gastrointestinal. Esfago de Barret. En: Cotran, Kumar,
Robbins, ed. Patologa Estructural y Funcional. 5 edicin. Madrid, Espaa: Ediciones
McGraw-Hill-Interamericana. 845
Rodriguez A. Esfago de Barret. Boletn de la Escuela de Medicina. Universidad
Catlica de Chile. [en lnea] 1998 [fecha de acceso: 28 de diciembre de 2004] Vol 27.
N 1. 1998. URL. Disponible

8.- Femenino de 25 aos, acude a urgencias por cuadro de 3 das de evolucin


caracterizado por dolor en cuadrante superior derecho, de inicio insidioso, intermitente,
opresivo, sin irradiacin y de intensidad 7/10. Ingiere paracetamol para cefalea ocasional
y anticonceptivos orales desde hace varios aos. El ultrasonido demuestra una imagen
bien delimitada, hiperecica de 2 cm de dimetro en el lbulo heptico derecho.
El diagnstico ms probable en esta paciente es:
a)

Carcinoma hepatocelular.

b)

Quiste heptico.

c)

Adenoma heptico.

d)

Metstasis heptica.

Los adenomas hepticos (AH) son proliferaciones benignas de hepatocitos, que suelen
presentarse en mujeres entre los 20 y los 40 aos de edad y con antecedentes de toma
de anticonceptivos orales. Infrecuentemente se presentan en varones, siendo la
proporcin de varones: mujeres de 1:11.
El sntoma ms frecuente es el dolor abdominal vago crnico, que si se localiza en el
cuadrante superior derecho suele ser por una hemorragia intratumoral.
Actualmente, las tcnicas de imagen con mejor rendimiento para su diagnstico son la TC
helicoidal multifsica y la resonancia magntica (RM).

Hugh TJ, Poston GJ. Benign liver tumors and masses. Surgery of the liver and biliary tract.
3 edicin. London, 2000; tomo 2 p.1397-1422

9.- Masculino que acude al servicio de consulta externa presenta prurito y lagrimeo en
ambos ojos, hiperemia conjuntival, fotofobia, exudado ms o menos viscoso y formacin
papilar en la conjuntiva tarsal. El diagnstico ms probable es:

a) Queratoconjuntivitis seca.
b) Conjuntivitis alrgica.
c) Conjuntivitis bacteriana.
d) Conjuntivitis vrica.

Conjuntivitis alrgica.
Condicin inflamatoria ocular bilateral, crnica y recurrente, que se presenta
predominantemente en la infancia y con mayor incidencia en el sexo masculino. Las
exacerbaciones se asocian a ciertas estaciones del ao (donde el nombre de
"primaveral") en las cuales se supone que existe una mayor cantidad de alergenos en el
medio ambiente; por ejemplo, el polen de las plantas. Estos pacientes frecuentemente
presentan historia personal o familiar de atopia. El sntoma principal es el prurito ocular
intenso, adems de secrecin mucoacuosa matutina, fotofobia severa y lagrimeo.
Clnicamente se observa intensa hiperemia y quemosis conjuntival bulbar, presencia de
papilas gigantes en la conjuntiva tarsal, secrecin mucoide abundante, ocasionalmente se
observan infiltrados eosinfilos a nivel de limbo que se conocen como "puntos de Trantas"
y en etapas crnicas se aprecia acmulo de pigmento en conjuntiva bulbar y opacificacin
corneal secundaria a queratopata punteada. Es frecuente que estos pacientes presenten
una sobreinfeccin debido al contacto de las manos con estructuras oculares por el prurito
incontrolable. Se ha demostrado presencia de eosinfilos e inmunoglobulina IgE; sin
embargo, el diagnstico es esencialmente clnico. El tratamiento se divide en medidas
preventivas, paliativas y antiinflamatorias.
Las medidas preventivas se orientan a evitar la exposicin directa al sol, utilizacin de
visera y lentes oscuros; el tratamiento sintomtico se basa en la aplicacin de fomentos
fros, lubricantes y vasoconstrictores tpicos y el uso de antihistamnicos orales. Como
antiinflamatorios tpicos en etapas agudas se emplean cursos cortos de esteroides. Los

estabilizadores de clulas cebadas (cromoglicato de sodio) y antihistamnicos tpicos


tienen efecto a largo plazo. Generalmente estos pacientes presentan mejora importante
en la intensidad y frecuencia de los episodios al alcanzar la adolescencia.

Referencias Bibliogrficas:
Miller Stephen JH. Parsons Diseases of the Eye. 18th ed. Edinburgh, London, Melbourne
and New York: Churchil Livigstone; 1990.
Snchez Salorio M, Rodrguez Ares T, Alio y Sanz JL, Pita D, Fontanela JR, Grau M et al.
Conjuntivitis, Edika-Med S.A. Barcelona: CIBA VISION, 1992.
Foulks GN, Pavan-Langston D. Cornea and External Disease. In: Pavan-Langston D,
editor. Manual of Ocular Diagnosis and Therapy. 2nd ed. Boston: Little, Brown & Co. 1991.
p. 100-102.

10.- Se trata de masculino de 58 aos quien presenta sntomas de probable infarto agudo
al miocardio, Cul de los siguientes marcadores tiene mayor sensibilidad para realizar
ste diagnstico?
a)
b)
c)
d)

CPK
Transaminasa
Deshidrogenada lctica
Troponina

Es muy conocida la falta de especificidad de la CPK. La elevacin de la actividad de esta


enzima se produce tanto en el infarto de miocardio (IAM) como en afecciones
caracterizadas por un grado variable de necrosis muscular. Por ello en los ltimos aos se
han realizado, y se siguen realizando, considerables esfuerzos para encontrar nuevos
marcadores analticos que sean capaces de diferenciar estos cuadros. Entre ellos, la
determinacin de la isoenzima miocrdica de la CPK de forma cuantificada (CPK-MBMasa) resulta tambin poco especfica ya que se eleva tambin en caso de necrosis de
msculo estriado; est descrita su elevacin en traumatismos, rabdomiolisis,
convulsiones1, miopatas agudas y crnicas2, insuficiencia renal en dilisis3 e incluso en el
ejercicio intenso4,5; y lo mismo podemos afirmar de la mioglobina6.
Ms recientemente se han determinado las troponinas T e I en sus isoformas especficas
de msculo cardiaco, que tienen una secuencia diferente de aminocidos a las de
msculo estriado no cardiaco, lo que permite el desarrollo de inmunoensayos especficos.
El complejo de las troponinas T, C, e I est estrechamente unido al filamento de
tropomiosina. La T se encarga de la unin a tropomiosina; la C es iniciadora de la
contraccin tras unirse al calcio, y la I se llama as por ser inhibidora de la contraccin en

reposo7. Esta especificidad ha sido demostrada en varios trabajos que muestran valores
normales de troponina-I en procesos que cursan con necrosis de msculo estriado2,8.
Adems de ser muy especficas, las troponinas son altamente sensibles en el infarto de
miocardio. Mair y col encuentran una sensibilidad del 100% para el diagnstico si se hace
la determinacin de troponina-I pasadas 6 horas del comienzo del dolor9. La cintica de
estos marcadores en pacientes con IAM puede resumirse de esta forma:
1. La CPK total comienza a elevarse a las 4-8 horas del comienzo de los sntomas, y
permanece elevada hasta que se normaliza a las 48-72 horas10.
2. La isoenzima MB de la CPK (CPK-MB) aparece en el suero tres horas despus del
comienzo del IAM, con un pico mximo a las 18-20 horas, alcanzando valores 16 veces
superiores al normal. A partir de este punto desciende lentamente y persiste elevada al
menos 2 das11.
3. La troponina-I se eleva a partir de las 2-3 horas del comienzo de los sntomas, con un
valor mximo a las 16 horas. Desciende bruscamente hasta las 48 horas, y a partir de
entonces se produce un lento descenso; puede detectarse todava el 7-8 da11.
4. La mioglobina es la primera que se eleva. Da las cifras ms altas de sensibilidad en el
plazo de dos horas del comienzo del dolor10 con respecto a la troponina-I y la CPK-MB.
Alcanza su pico a las 8 horas y desciende bruscamente a valores normales a las 18
horas11
Utilidad de la troponina-I, CPK-MB y mioglobina en el diagnstico del infarto de miocardio
y
de
los
procesos
de
necrosis
muscular
de
origen
no
cardiaco
Use of troponin-I, CPK-MB and myoglobin in the diagnosis of myocardial infarct and
processes of muscular necrosis of non-cardiac origin J.I. Ibez1, R. Sobrado1, M. Rivero2,
J.M. Olite3, I. Idoate3, I. Berrozpe1, E. Arina1, L. Metola1, J. Sesma1
1. Unidad de Urgencias.
2. Servicio de Medicina Interna.
3. Servicio de Bioqumica Clnica. Hospital Virgen del Camino Pamplona.

11.- Femenino de 76 aos que acude a consulta por presentar una mcula pigmentada
heterocroma, de contorno irregular, de unos 2 x 3 cm de dimetro, localizada en mejilla
izquierda, que ha experimentado un crecimiento muy lento en los ltimos aos. El
diagnstico ms probable es:

a)
b)
c)
d)

Melanoma lentiginoso acral.


Carcinoma Basocelular pigmentado.
Lentigo maligno.
Eritema fijo pigmentario.

El lentigo maligno puede definirse como una mcula pigmentada extensa localizada en
una zona de la piel expuesta al dao actnico, habitualmente desarrollada en ancianos y
que consiste en una proliferacin de melanocitos atpicos situados sobre una epidermis
atrfica.
La importancia del lentigo maligno reside en que puede ser el sustrato de un melanoma
maligno.
El lentigo maligno tambin ha sido denominado, lentigo de Hutchinson, lentigo senil,
melanosis precancerosa circunscrita, lentigo malin des viellards, melanosis premaligna,
melanosis circunscrita preblastomatosa y melanocitoma no nevoide precanceroso.
El lentigo maligno puede ser considerado como la nica variedad de displasia
melanoctica intraepidrmica capaz de evolucionar a un melanoma invasivo.
El porcentaje de melanomas que se suponen derivados de un lentigo maligno es
pequeo, 2.5 a 5%. El lentigo maligno es fcilmente reconocible y habitualmente tratable
en las fases ms precoces de su desarrollo.
El lentigo maligno fue descrito en 1892 por Hutchinson. Dubreuilh remarc su naturaleza
precancerosa en 1894. Mishima determin que el precursor histognico del lentigo
maligno es diferente al precursor del nevus juncional descrito por Allen y Spitz. La
histopatologa macroscpica y microscpica del lentigo maligno fue documentada por
Whyte y Helwig (1) y por Clark y Mihm.

Lentigo maligno 01. Dermatoscopia. Pigmentacin asimtrica de las salidas foliculares


(flechas pequeas), estructura romboidal (flecha grande), oclusin de las salidas
foliculares (crculo) y puntos azul-gris (asterisco) Lentigo Maligno

Cox NH, Aitchison TC, Sirel JM, Mackie RM. Comparison between lentigo maligna
melanoma and other hystogenetic types of malignant melanoma of the head and neck. Br
J Cancer 1996;
73:940-4.
Cohen LM, McCall MW, Zax RH. Mohs micrographic surgery for lentigo maligna and
lentigo maligna melanoma. Dermatol Surg 1998; 24:673-77

12.- A 26-year-old woman presents with malodorus gray-wellow discharge. You take a wet
mount preparation and observe Clue cells. The agent of this infection and its treatment
is:
a)
b)
c)
d)

Gardnerella vaginalis / Ketoconazole


Trichomona vaginalis / metronidazole / treat the partner
Candida albicans / nistatin
Gardnerella vaginalis / clindamycin

Gardnerella vaginalis fue clasificada como una sola especie y fue establecida como
agente causal de la vaginosis (antes conocida como vaginitis inespecfica). El cuadro
clnico que presenta es caracterizado por una secrecin blanca o blanco-griscea que se
percibe generalmente despus de la relacin sexual con olor ftido aminado (pescado). El
diagnstico certero es la base para evitar posibles complicaciones como la enfermedad
inflamatoria pelviana y las complicaciones del embarazo. El tratamiento se basa
principalmente en los frmacos como: metronidazol y clindamicina, debido a su
efectividad y espectro, pero como todos se deben emplear con adecuada prudencia
debido a su toxicidad. Adems de que se deben corregir o modificar los factores
predisponentes, ya que esta patologa va en aumento convirtindose por su frecuencia en
un problema de salud pblica.
Bibliografa
1. Hernndez F. Gardnerella vaginalis mobiluncus en la etiologa de la vaginosis
bacteriana. Rev Costarricense Ciencias Mdicas 1998; 19: 57-61.
2. Hansen EA. Gardnerella. Rev Ginecol 2005; 25: 99.

3. Espinosa I, Lorenzo M, Bentancourt A, Rivern Y, Romero M. Caracterizacin


bioqumica y antignica de diferentes aislamientos de Gardnerella vaginalis.
Rev Cubana Invest Biomed 2005; 24: 22-7.
4. Taylor F. Vaginal flora morphotypic profiles and assessment of bacterial vaginosis in
women at risk for HIV infection.Infect Dis Obstet Gynecol
2004; 12: 121-6.

13.- Se trata de paciente masculino de 14 aos de edad con malos hbitos higinico
dietticos, convivencia con aves, gatos e ingestin de berros. Inicia su padecimiento
actual hace dos semanas con fiebre de 39C sin predominio de horario, nusea y dolor en
hipocondrio derecho. A la exploracin fsica con palidez generalizada y hepatomegalia 33-5. Resto de la exploracin sin datos patolgicos. Laboratorio con BH, con Hb de 11.5,
Hto de 40, leucocitos; 16,500, eosinfilos; 38%, linfocitos; 30%, formas inmaduras; 4,
plaquetas; 270,000, Fosfatasa alcalina; 280 UI/L. El diagnstico ms probable es:

a) Larva migrans visceral


c) Fasciolosis
d) Gnatostomiasis
e) Estrongiloidosis

Definicin
La Fasciolosis es una zoonosis parasitaria causada por la Fasciola heptica que ocasiona
patologa y sintomatologa hepato-biliar.
Clnica
Se considera las siguientes formas de presentacin clnica:
a) Sintomtica: Aguda o invasiva, crnica o de localizacin y extraheptica.
1. Aguda o invasiva: Hay tres elementos esenciales a identificar: hepatomegalia dolorosa,
fiebre y eosinofilia con cifras que superan frecuentemente el 30-40%.
2. Crnica o de localizacin: La sintomatologa y signologa corresponden a padecimiento
crnico
hepato
biliar
incluyendo
clicos
biliares
y
litiasis
biliar.
3. Extraheptica: Incluye ndulos subcutneos en el hipocondrio derecho, seno derecho,
escpula derecha con poco dolor local y signos inflamatorios. Eosinofilia alta.

b) Asintomtica: En algunas personas los sntomas o signos suelen pasar desapercibidos.


Diagnstico de Laboratorio

En la forma aguda, la bsqueda de huevos en las deposiciones es intil, ya que las


formas juveniles estn en el tejido heptico, por lo tanto las pruebas inmunobiolgicas son
importantes. Son tiles la inmunoelectroforesis o inmunodifusin buscando el arco 2 de
Caprn (4). El inmunoblot o westernblot tiene buena sensibilidad y especificidad (5,6). Se
han identificado fracciones antignicas en las cistenilproteasas de F. hepatica (7).
En las formas crnicas, la bsqueda de huevos en heces es lo indicado. Son tiles la
sedimentacin rpida de Lumbreras (8). Recientemente se han preparado anticuerpos
monoclonales contra el parsito y ello ha permitido elaborar la tcnica de ELISA para
detectar los coproantgenos (E/S) del parsito en heces. La ecografa de vas biliares
(v.b.) puede detectar al parsito movindose en las v.b. o vescula. En formas
extrahepticas, la eosinofilia alta es orientadora; F. hepatica en las biopsias confirma el
diagnstico.
14.- Masculino de 30 aos postraumatizado por colisin automovilstica con cuadro de
prdida progresiva de fuerza en miembros inferiores, los reflejos osteotendinosos estn
abolidos. Al realizarse radiografas simples muestran una fractura por compresin de L1
con desplazamiento del muro posterior y acuamiento anterior de un 50%. Para valorar
la ocupacin del canal raqudeo, cul de las siguientes pruebas indicara?

a) Una gammagrafa sea.


b) Una tomografa cervical.
c) Rx ap y lateral con foco en L5
d) Una TAC vertebral centrada en regin dorso lumbar.

Tomografa computarizada (TC)


En lneas generales, podemos decir que sus indicaciones (con carcter urgente) en
los TRM, son todas aquellas lesiones detectadas o sospechadas en las radiografas
simples, que puedan suponer riesgo de lesin medular por desplazamientos
ulteriores (lesiones inestables), o que ya estn produciendo dao neurolgico
susceptible de mejorar o estabilizarse tras descompresin quirrgica. Por lo tanto
debe realizarse en todos los pacientes con fracturas, luxaciones y fracturas-luxaciones
inestables, y en aquellos con dficit neurolgico, preferentemente incompleto. En
pacientes con lesiones estables en las radiografas simples y sin dficit neurolgico,
puede diferirse en funcin de la presin asistencial y de la disponibilidad del servicio de
Radiologa. En la prctica, suele indicarse tambin cuando no se visualizan determinadas
zonas de la columna, generalmente C1-C2 y C6-Dl. En pacientes con TCE grave deben
realizarse cortes de estas zonas, si no se ven claramente en las radiografas o en el

"scout" cervical, e incluso de forma rutinaria, dada la frecuente asociacin de estas


lesiones. La TC proporciona una excelente visualizacin de las estructuras raqudeas
principalmente de los elementos posteriores y del canal medular, por lo que se pueden ver
con nitidez los desplazamientos y fragmentos seos que puedan estrecharlo o invadirlo.
Asimismo, pueden verse fracturas o desplazamientos inadvertidos en las radiografas y
nos permite valorar mejor estas lesiones, definiendo claramente las lneas de fractura,
cuantificando exactamente los desplazamientos, y en definitiva, evaluando la estabilidad
vertebral.
Habitualmente se realizan cortes de 5 mm de espesor, aunque la exploracin detallada de
la columna cervical pueda requerir cortes ms delgados, lo que alargar el tiempo de
estudio, factor importante en el manejo de los traumatismos. Los equipos de ltimas
generaciones han conseguido acortar este tiempo y permiten adems, la reconstruccin
sagital o coronal a partir de los cortes axiales, mejorando la definicin y la evaluacin del
canal medular.
La TC es til tambin para valorar las articulaciones interapofisarias y los agujeros de
conjuncin, as como los hematomas paravertebrales y retroperitoneales. Una ventaja
adicional es la de ofrecernos informacin suplementaria sobre ciertas partes blandas del
cuello y de las cavidades torcica y abdominal.
Las fracturas horizontales que no coincidan con el plano de la TC pueden no visualizarse,
como la de odontoides o algunas por compresin. Los hematomas epidurales,
hematomielia y hernias discales pueden verse tambin con TC, aunque la RM define
mejor estas lesiones. La contusin y el edema medular, las lesiones y avulsiones
radiculares y los desgarros durales requieren estudio mielogrfico adicional y/o RM. La
mielo-TC se realiza con inyeccin de contraste intratecal por puncin lumbar o cervical,
que obliga a la movilizacin del paciente o retirada del collarn cervical, adems de otros
inconvenientes como tiempo de estudio y reacciones adversas, por lo que generalmente
no son tiles en el manejo urgente del paciente con TRM.

Balliger, Phillip W. Cerril. Atlas de posiciones radiogrficas y procedimientos radiolgicos.


7. ed.; Espaa: Masson.
Goaz P. W. Radiologa oral (principios e interpretacin). 3. Ed. Espaa; ed.;
Mosby.1995

15.- Su tutor en el curso de infectologa de pregrado le pide que revise a un paciente al


cual se le ha diagnosticado endocarditis en la vlvula mitral. Como parte de su
exploracin fsica usted revisa el fondo de ojo en busca de:

a)
b)
c)
d)

Mancha de Roth
Ndulos de Osler
Lesiones de Janeway
Hemorragias en astilla

Las manchas de Roth son hemorragias retinales (retinianas) con centro plido o de color
blanco compuestas y rodeadas de fibrina coagulada o cmulos de hemates. Estas
manchas pueden observarse en el fondo de ojo mediante el uso de un oftalmoscopio.1
Las manchas de Roth generalmente son causadas vasculitis mediada por
inmunocomplejos debido a la endocarditis bacteriana, tambin pueden ser observadas en
leucemia, diabetes, anemia perniciosa, isquemia y raramente en retinopata por VIH.
Fowler VG Jr, Scheld WM, Bayer AS. Endocarditis and Intravascular Infections. In:
Mandell GL, Bennett JE, Dolin R, eds. Principles and Practice of Infectious Diseases. 7th
ed. Philadelphia, Pa: Elsevier Churchill Livingstone; 2009; chapt 77.

16.- Se trata de paciente masculino de 3 aos de edad que acude a urgencias con una
historia de secrecin nasal purulenta y de mal olor unilateral desde hace 5 das. El
diagnstico ms frecuente es:
a)
b)
c)
d)

Atresia de coanas unilateral.


Rinitis crnica por Rinovirus.
Fibrosarcoma del correte nasal.
Cuerpo extrao intranasal.

Una de las patologas ms frecuentes que enfrenta el otorrinolaringlogo en su prctica


mdica, es la presencia de cuerpos extraos animados o inanimados en vas
areodigestivas superiores. La mayora de las veces su tratamiento (extraccin) no reviste
mayor importancia sobre todo cuando no han sido manipulados por mdicos de primer
contacto, pero en otras, se convierte en un procedimiento muy difcil de realizar, ya sea
por las complicaciones que se produjeron o por la presencia del mismo.
En la poblacin infantil es ms frecuente por la curiosidad propia de esa edad y la
tendencia a colocarse una variedad de elementos en las fosas nasales.
A veces existen ciertas dificultades en el diagnstico, pero la trada sintomtica de
obstruccin nasal, rinorrea unilateral y halitosis, nos sugiere la posibilidad de un cuerpo
extrao hasta que no se demuestre lo contrario. Independientemente de su naturaleza
animada o inanimada, stos pueden alojarse y/o impactarse en distintos niveles.
El tratamiento electivo es la extraccin por vas naturales de los mismos; en los nios que
no colaboran es necesario recurrir a la anestesia general.
1. Manual de Urgencias en Otorrinolaringologa. Manuel Toms Barbern, Julio GarcaPolo Alguacil, Guillermo Til Prez

17.- Se trata de masculino de 64 aos de edad que acude a consulta por presentar dolor,
calor y tumefaccin en la rodilla derecha. De los siguientes hallazgos el ms til para
establecer el diagnstico de pseudogota en este enfermo es:
a) Crecimiento de articulaciones interfalngicas proximales e interfalngicas distales
b) Altas concentraciones sricas de cido rico
b) Cristales con birrefringencia negativa del lquido de la rodilla
d) Calcio en el menisco en una radiografa de la rodilla afectada

Allen R. M. MMS Medicina Interna. 5. Edicin. National Medical Series. Mc. Graw Hill.
2006. (captulo 10 IV B 5 b).

El hallazgo de un menisco calcificado en la radiografa de la rodilla afectada es un dato


diagnstico de enfermedad por dihidrato de pirofosfato de calcio, que sugiere que la
inflamacin de la rodilla es causada por seudogota. El crecimiento de las articulaciones
interfalngicas proximales y distales sugiere slo osteoartritis y no una causa especfica.
El aumento srico de urato se relaciona con gota. El aumento de cristales con
birrefringencia en examen de luz polarizada compensada roja, de lquido sinovial, es
especfico para el diagnstico de seudogota. Hay muchas causas de derrame inflamatorio
aparte de esta entidad.

18.- Se trata de paciente femenino de 45 aos que tras 20 min. Posteriores al inicio de
una transfusin presenta fiebre, ansiedad, dolor lumbar severo, nausea y enrojecimiento
facial, por lo que usted sospecha de:
a) Reaccin hemoltica aguda
b) Reaccin febril
c) Toxicidad por citrato
d) Hepatitis

Reaccin Hemoltica
Definicin:
Destruccin acelerada del eritrocito. De acuerdo a la causa puede ser inmune o no
inmune, por el sitio de destruccin puede ser intra o extravascular y por el tiempo de
aparicin puede ser aguda o retardada.
Incidencia
Reaccin hemoltica aguda: Las referencias internacionales reportan una incidencia de
reaccin hemoltica aguda de 1 en 6

000 en 30 000 unidades transfundidas, con una tasa de mortalidad de 1 en 500 000 a 1
en 1 000 000 de unidades.
Del total de las reacciones hemolticas agudas, el 6% resultan fatales. La FDA reporta que
alrededor del 41% de las muertes por transfusin son causadas por incompatibilidad ABO,
con una incidencia de mortalidad de 1 en 200 000 pacientes transfundidos.
Reaccin hemoltica retardada. La incidencia es de 1 en 2 500 a 1 en 4 000 unidades
transfundidas. La mortalidad es de 1 en
3.85 millones de unidades y de 1 en 1.15 millones de pacientes transfundidos.

Fisiopatogenia
La reaccin hemoltica transfusional ms grave se presenta cuando interactan los
eritrocitos transfundidos con los anticuerpos preformados en el receptor. La reaccin
antgenoanticuerpo puede no activar complemento de acuerdo a la inmunoglobulina
implicada, lo que conduce a hemlisis intra o extravascular. En la hemlisis extravascular
algunas citocinas con actividad inflamatoria y vasoactiva intervienen en la reaccin como:
Factor de necrosis tumoral alfa, Interleucina 1, 6, 8 y Protena quimioatractante de
macrfago (MCP), as como la liberacin de sustancias tromboplsticas que explican el
cuadro clnico caracterstico de la reaccin hemoltica transfusional. En la hemlisis
extravascular el eritrocito sensibilizado es destruido por el sistema fagoctico
mononuclear.

REACCION HEMOLITICA AGUDA INMEDIATA DE TIPO INMUNE:


Es la reaccin ya descrita por incompatibilidad ABO y como consecuencia de ella al iniciar
la transfusin en pocos minutos pueden ser lisados la mitad ms de los de los hemates
transfundidos.
La reaccin hemoltica aguda se produce de forma brusca, tras la infusin de varios
mililitros de sangre aparece un cuadro de fiebre,escalofrios,dolor lumbar, dolor opresivo
en torax , cefalea nauseas con sin vomitos e incluso shock,dependiendo de la cantidad
y rapidez de la sangre transfundida.
Los sintomas dependen de los pacientes pero la fiebre aparece en todos los casos.
La reaccin es inmediata al inicio de la transfusin
En los enfermos anestesiados estas reacciones quedan enmascaradas y ser mnimas, por
lo que la hemlisis puede debutar con taquicardia e hipotensin brusca trs el inicio de la
transfusin,si persiste la transfusin puede aparecer oliguria,hemoglobinuria,CID
fracaso renal postoperatorio debido a algn efecto txico de la Hb a la situacin de
shock.

Diagnstico Diferencial
Contaminacin Bacteriana del componente sanguneo
Hemlisis no inmune:
- Mecnica
- Trmica
- Osmtica

Lecturas recomendadas:
Rev Mex Med Tran, Vol. 3, Nm. 1, pp 18-21 Enero - Abril, 2010
Hospital de Pediatra CMN Siglo XXI Boletn La Calidad Abr May Jun 2004

19.- Masculino de 46 aos de edad que inicia con dolor intenso en fosa renal izquierda
ante la sospecha de litiasis renoureteral se realizan estudios de laboratorio y gabinetem,
los Rx demuestran clculos radio lcidos, los cuales estn relacionados a:
a)
b)
c)
d)

Calcio
Estruvita
Cistina
cido rico

-Bruce E. Jarrell, R. Anthony Carabasi, Nacional Medical Series for Independent Study.
Wiliams & Wilkins, 3rd Edition: 451-475.

Los clculos que se pueden presentar en la va urinaria son formados por calcio, cido
rico y cistina, los nicos radiolucidos son los de cido rico que representan un reto
diagnstico.

20.- Se trata de masculino de 60 aos de edad con una historia de fumar 25


cajetillas/ao, est siendo evaluado para la colocacin de un bypass coronario. De las
siguientes pruebas preoperatorias, la mejor para valorar la funcin respiratoria en este
paciente es:

a)
b)
c)
d)

FVC
PaCO2
PaO2
FEV1/FVC

La relacin FEV1/ FVC le proporciona la capacidad de las funciones pulmonares del


paciente y la FEV1 le indica si hay una obstruccin de la va area, la FVC Y VR aisladas
le indican si hay una obstruccin nicamente, la PaCO2 Y PaO2 solo le indican si hay
retencin de Co2 y la oxemia del paciente.

1.- Fishman AP, , Fishman JA, Grippi MA, Kaisser LR, Seor RM. Pulmonary Diseases
and disorder. 3a. Edicin McGraw-Hill, EUA, 2006.
2.- Fraser, R ; Neil, C; Par, P; Diseases of the Chest, Third Edition, Editorial Elsevier,
2005.
3.- Murray and Nadels; Textbook Respiratory Medicine, Vol 1-2, Elsevier editorial, 2005.

21.- Femenino de 51 aos, con mioma uterino de tamao equivalente a una gestacin de
12 semanas, que presenta hipermenorreas y hemoglobinemia de 9 gr%. No se demuestra
patologa asociada. Se encuentra en espera para la prctica de una histerectoma
programada a realizar en 4 meses. En esta paciente est indicado el tratamiento
preoperatorio con:

a)
b)
c)
d)

Anlogos de la GnRH.
Estrgenos.
Inhibidores de la fibrinlisis.
Derivados del cornezuelo del centeno.

Anlogos de la GnRH: Son derivados de la hormona GnRH en donde se ha realizado


una sustitucin peptdica en posicin 6 y en algunos casos en la 10, obteniendo
compuestos hasta unas 200 veces ms potentes debido a mayor afinidad por los
receptores y a su resistencia a la degradacin por peptidasas. Aunque su accin inicial
produce un incremento en la produccin de FSH-LH (efecto flure-up o llamarada) tras 5-6
das de exposicin contnua, los receptores son internalizados producindose un estado
de hipogonadismo hipogonadotropo y niveles de estradiol similares a los de la
postmenopausia. Los anlogos de GnRH estn disponibles en distintas frmulas:
administracin nasal (varias aplicaciones al da), subcutnea (aplicacin diaria) o
intramuscular (preparados depot mensuales o trimestrales) (Shaw RW 1999).
Marco Filicori y sus colaboradores de la Universidad de Bolonia fueron los primeros en
utilizar en 1983 los aGnRH en un estudio que confirm su eficacia para reducir el tamao
de los miomas uterinos y secundariamente sntomas como alteraciones menstruales,
dolor plvico y sntomas de presin local. Otros autores como Minaguchi H y
colaboradores continan comprobando la efectividad de los anlogos de la GnRH en el
tratamiento del mioma uterino tras evaluar en el ao 2000 seis estudios con un total de
602 pacientes tratadas con nafarelina. La disminucin del tamao se calcula entre un 3070%, y se ha observado como el mayor porcentaje de reduccin ocurre tras el primer mes
de tratamiento, no existiendo reducciones o siendo stas mnimas despus del tercer mes
(Healy et al 1986; Friedman et al 1989; Matta et al 1989; Williams y Shaw 1990). En
miomas pediculados o con gran proporcin de calcio o colgeno (hialinizacin) la repuesta
es tambin menor. Debe tenerse en cuenta que si despus de dos meses de tratamiento
no se ha producido un significativo descenso del tamao del mioma, ste ya no debe ser
esperado y debe pensarse en la posibilidad de la existencia de un tumor muscular
maligno no diagnosticado (Messia AF et al 1998). Tras finalizar el tratamiento y
recuperarse el estado de hipogonadismo, el mioma retorna rpidamente a su tamao
inicial (Friedman AJ et al 1987; Matta WH et al 1989).

En casos prximos a la menopausia, la reduccin del tamao del mioma y su


sintomatologa, permitira hablar de una solucin mdica del problema, pero en todo caso
la utilizacin de aGnRH facilitara la intervencin quirrgica al acortar el tiempo de
intervencin, la hemorragia y el acceso a localizaciones complicadas como el caso de
miomas interligamentarios o situados en istmo o crvix. En el caso de la ciruga
histeroscpica la reduccin del tiempo de ciruga permitira reducir el volumen de fluidos
aportado a cavidad uterina y los riegos de absorcin e hiponatremia.
Los mecanismos de accin por los que los aGnRH actan son: o Hipoestrogenemia: es
necesario mantener la hipoestrogenemia, pues la elevacin de sus niveles lleva a un
rpido incremento del tamao del mioma. El crecimiento del mioma es dependiente de los
niveles de estrgenos (aumentan de tamao con el embarazo y se reducen durante la
menopausia o el tratamiento con aGnRH, pudiendo volver a crecer durante la THS), pero
aunque los estrgenos parecen ser importantes en el crecimiento del mioma, su relacin
debe ser algo ms compleja pues no se han descrito incrementos significativos del
tamao de miomas durante el tratamiento con gonadotrofinas en RA (situaciones con
elevados niveles de E2), algunos de ellos no se modifican durante el embarazo o incluso

decrecen y se han encontrado crecimientos despus del tratamiento con citrato de


clomifeno (antiestrgeno). En relacin con la hipoestrogenemia podran estar los cambios
inducidos en el flujo vascular uterino (incrementos en el ndice de resistencia de las
arterias uterinas) que suponen una reduccin de la vascularizacin o las modificaciones
de distintos factores de crecimiento. o Cambios histolgicos: el tratamiento con aGnRH
puede producir degeneracin roja, infiltracin linfocitaria, y necrosis, as como reduccin
de la proliferacin celular e incremento de la apoptosis. Pero en otras circunstancias no es
posible encontrar 7 diferencias. No se ha encontrado una relacin entre los cambios
histolgicos y el porcentaje de reduccin del tamao del tero, y existe una gran
variabilidad entre distintas pacientes o entre distintos miomas de una misma paciente, no
existiendo pues un patrn histolgico caracterstico de respuesta ante el tratamiento con
aGnRH.
BIBLIOGRAFA
1. Abad L, Abad de Velasco L, Parilla JJ. Etiopatogenia. Papel de las hormonas
esteroideas, factores de crecimiento y otras sustancias. Cuad Med Reprod
1999;5(1):15-29.
2. Albano C, Platteau P, Devroey P. Gonadotropin-releasing hormone antagonist: how
good is the new hope? Curr Opin Obstet Gynecol
2001;13(3):257-62.
3. Coutinho EM.Treatment of large fibroids with high doses of gestrinone.
Gynecol Obstet Invest 1990;30(1):44-47.
4. Chavez NF, Stewart EA. Medical treatment of uterine fibroids. Clin Obstet
Gynecol 2001;44(2):327-84.
5. De Leo V, la Marca A, Morgante G. Shortterm treatment of uterine
fibromyomas with danazol. Gynecol Obstet Invest 1999;47(4):258-262.
6. Eldar-Geva T, Healy DL. Other medical management of uterine fibroids.
Baillieres Clin Obstet Gynaecol 1998;12(2):269-88.
7. Felberbaum RE, Germer U, Ludwig M, Riethmuller-Winzen H, Heise S,
Buttge I, Bauer O, Reissmann T, Engel J, Diedrich K. Treatment of uterine fibroids with a
slow-release formulation of the gonadotrophin releasing hormone
antagonist Cetrorelix.HumReprod 1998;13(6):1660-8.

22.-Se trata de mujer de 20 aos, gesta 1, tuvo un parto con un producto nico masculino
de 3,600 g. la calificacin de Apgar en el RN fue de 9 al primer minuto y 9 a los 5 minutos.
La revisin de sus registros de trabajo de parto mostr que tuvo ruptura de membranas 7
horas antes del parto. 40 horas despus del parto la paciente presenta: temperatura 38.2
C, FC 105 x, TA 110/70, FR 16x; E.F. dolor leve a la palpacin del tero, las mamas se
encuentran sin eritema, ni dolor a la palpacin, no dolor a la compresin de las
pantorrillas. Cul de los siguientes pasos es el ms indicado antes de iniciar terapia
con antibiticos?

a) Hemocultivo
b) Cultivo de secrecin vaginal

c) Examen general de orina y cultivo


d) Espirometra por incentivo
.
El vaciamiento incompleto ocasiona orina residual, distensin vesical excesiva y estasis,
adems del cateterismo intermitente con sonda vesical durante el trabajo de parto. Por lo
tanto, la vejiga en el puerperio est predispuesta a infecciones. El dolor leve a la
palpacin del tero puede ser normal en el puerperio y no se debe suponer de inmediato
endometritis puerperal. Cuando se sospecha endometritis, los cultivos de secrecin
vaginal tienen poca utilidad porque se encuentran los mimos microorganismos que en
mujeres purperas sanas. Los hemocultivos son apropiados para la valoracin
diagnstica de la fiebre puerperal, pero no son el paso inicial. La espirometra por
incentivo se utiliza en el posoperatorio de inmediato para fomentar la expansin pulmonar
y disminuir las atelectasias. El legrado uterino se utiliza para tratar la hemorragia
Morgan M, Siddighi S. Ginecologa y obstetricia, National Medical Series. 5 edicin. Mc
Graw Hill. Pp. 29.

23.- Se trata de masculino de 6 aos de edad, que cursa con odinofagia y fiebre no
cuantificada desde hace ms de tres das. E.F. orofaringe con exudado membranoso y
petequias en paladar blando y pilares anteriores. Lengua de fresa roja. To. 39.5 Oc, usted
elige el siguiente frmaco por ser el medicamento de eleccin en esta patologa.

a)
b)
c)
d)

Cefuroxime.
Penicilina benzatnica.
Amoxicilina.
Eritromicina.

ESCARLATINA
Tratamiento: 1) Reducir las complicaciones, 2) erradicar al
S. pyogenes.
Penicilina benzatnica: 20,000-50,000 U/kg/do (nica).
< 20 kg: 600,000 U IM.
> 20 kg: 1,200,000 U IM.
Penicilina V (fenoximetilpenicilina): 25-50 mg/kg/da cada 6
hrs. 250-500 mg cada 6 hrs. por 10 das. Separado de los
alimentos mejora absorcin.
Eritromicina: 30-50 mg/kg/da cada 6 hrs. 250-500 mg

cada 6 hrs. por 10 das. Alergia a la penicilina.

Streptococcus pyogenes (hemoltico del grupo A) es el agente etiolgico de la


escarlatina, no ofrece resistencia a la penicilina benzatnica y con dosis nica ofrece un
buen ndice de curacin, erradicacin y reduccin de complicaciones.
Gonzlez-Saldaa N, Infectologa Clnica Peditrica, 7 edicin, pginas 403-407

24.- Se trata de masculino de 32 semanas de gestacin que presenta ruptura de


membranas de ms de 18 horas de evolucin, sin progreso de trabajo de parto por lo que
se decide realizar cesrea, recibe maniobras habituales de reanimacin y posteriormente
presenta apnea, por lo que se da presin positiva intermitente (PPI) durante 30 segundos,
al evaluarlo presenta pobre esfuerzo respiratorio por lo que requiere PPI nuevamente por 5
minutos sin presentar esfuerzo respiratorio efectivo.
El hallazgo radiolgico ms probable es:
a) Infiltrados cotonosos
b) Rx. Normal
c) Broncograma areo, disminucin
reticulonodular de diferente grado.
d) Atelectasia apical derecha

del

volumen

pulmonar,

infiltrado

SINDROME DE DISTRES RESPIRATORIO POR DEFICIENCIA DE FACTOR


SURFACTANTE. (ENFERMEDAD DE LA MEMBRANA HIALINA)
Hallazgos radiolgicos
Pulmones pequeos Patrn granular fino Broncograma areo que se extiende a la periferia.

El patrn granular refleja los hallazgos histolgicos de dilatacin de conductos alveolares y


bronquiolos terminales sobre un colapso alveolar generalizado.
Al sobredistenderse los conductos alveolares y bronquiolos terminales, aparecen burbujas
pequeas, redondeadas de 1 a 2 mm de dimetro.
Durante la fase espiratoria desaparecen el broncograma areo y el patrn granular,
vindose los pulmones totalmente opacos.
Diagnstico diferencial.- Pueden verse opacidades pulmonares similares en: Neumona
neonatal, linfangiectasia pulmonar, sndrome de retencin de lquido y cardiopatas
congnitas asociadas con obstruccin venosa pulmonar.
Sin embargo a diferencia de los pacientes con EMH, los volmenes pulmonares en estas
patologas estn normales o aumentados

Clasificacin radiolgica de la enfermedad de membrana hialina


GRADO I. Granularidad pulmonar fina, broncograma areo confinado a los bordes de la
silueta cardiotmica claramente definidos.
GRADO II. Pulmones ligeramente menos radiolcidos, broncogramas areos proyectados
por encima de los bordes cardiotmicos.
GRADO III. Las densidades son ms confluentes, broncogramas ms extensos.
GRADO IV. Completa opacificacin pulmonar, ausencia de broncograma areo.
Figura 8. Enfermedad de Membrana Hialina

25.- Femenino de 45 aos inicia recientemente con fatiga, somnolencia, piel seca,
estreimiento y aumento de peso de 5 kg. Su tiroides est firme y tiene el doble del
tamao normal. Las pruebas de laboratorio que confirma el presunto diagnstico de
hipotiroidismo es:

a)
b)
c)
d)

Tiroxina srica (T4)


Triyodotironina srica (T3)
Captacin de resina T3
Hormona estimulante de tiroides (TSH) en suero

Hipotiroidismo
La instauracin es habitualmente lenta y progresiva. Los sntomas se relacionan con una
disminucin en la actividad funcional de todos los sistemas del organismo. Los ms
clsicos son cansancio, intolerancia al fro (carcter muy friolero), apata e indiferencia,
depresin, disminucin de memoria y de la capacidad de concentracin mental, piel seca,
cabello seco y quebradizo, fragilidad de uas, palidez de piel, aumento de peso,
estreimiento pertinaz y somnolencia excesiva. En situaciones extremas puede
evolucionar hacia la insuficiencia cardiaca, la hinchazn generalizada (mixedema),
insuficiencia respiratoria y abocar al coma mixedematoso con prdida de conocimiento
que conlleva un alto grado de mortalidad.
Al igual que el resto de enfermedades del tiroides, el hipotiroidismo es ms frecuente en
el sexo femenino. Es a partir de los 40-50 aos cuando las mujeres tienden a desarrollar
con ms frecuencia hipotiroidismo de causa autoinmune (tiroiditis de Hashimoto). El
periodo postparto es igualmente propenso a la aparicin de este problema. La ciruga de
tiroides y la aplicacin de yodo radioactivo representan situaciones de riesgo para el
desarrollo de hipotiroidismo, lo que obliga a controlar evolutivamente la funcin tiroidea en
estos casos.Los recin nacidos de madres hipertiroideas, hayan recibido o no tratamiento
antitiroideo durante la gestacin, deben ser evaluados en este sentido. Las personas en
las que se detectan anticuerpos antitiroideos (antimicrosomales, antitiroglobulina) tienden
a desarrollar con el tiempo alteraciones de la funcin tiroidea, por lo que deben ser
evaluados crnicamente de forma peridica.
La determinacin de TSH es el parmetro ms sensible para el diagnstico del
hipotiroidismo. Su elevacin es indicativa de que la funcin del tiroides es insuficiente.
Este fenmeno se produce antes de que comiencen a descender en la sangre las
concentraciones de hormonas tiroideas. Generalmente, en el hipotiroidismo establecido,
adems de la elevacin de TSH, se produce un descenso de T4. El nivel de T3 con
frecuencia se encuentra dentro de la normalidad. As pues, cuando aparecen sntomas

sugestivos, el mdico solicitar una determinacin de TSH que es el mejor mtodo para
descartar que exista hipotiroidismo. Puede acompaarse de una determinacin de T4 y de
anticuerpos antitiroideos si se desea conocer si la causa se debe a fenmenos de
autoinmunidad. En los casos de hipotiroidismo secundario debido a disminucin de la
secrecin de TSH por parte de la hipfisis, el diagnstico se basa en confirmar
concentraciones disminuidas de T4 y TSH en la sangre. Cuando la elevacin de TSH se
acompaa de niveles normales de T4 la condicin es conocida con el nombre de
hipotiroidismo subclnico. Si existe bocio puede ser conveniente realizar una ecografa
tiroidea. Cuando existe sospecha de alteraciones en el desarrollo de la glndula o de
deficiencia enzimtica, puede ser til obtener una gammagrafa tiroidea. Si se confirma un
diagnstico de hipotiroidismo de causa autoinmune, es habitual evaluar la asociacin de
alteraciones en otras glndulas como las suprarrenales, paratiroides o gnadas.

REFERENCIAS:
(1.) Anderson R, Harnes J. 1975. Thyroid hormones secretion rates in growing and mature
goats. J Anim Sci 40: 11301135.
(2.) Anke M, Henning A, Grun M, Partschefeld M, Groppel B. 1977. Der einluss des
mangan, zink, kupfer, jod, selen, molybdan und nickelmangels aauf die
fortpflanzuggsleistung des wiederkauers. Mathem Natur Reihe (Leipzig) 26: 283-292.
(3.) Balbuena O. 2003. Nutricin Mineral del Ganado. Sitio Argentino de Produccin
Animal: 1-5, www.produccionanimal. com.ar.
(4.) Beckett GJ, Beddows SE, Morrice PC, Nicol F, Arthur JR. 1987. Inhibition of hepatic
deiodination of thyroxine is caused by selenium deficiency in rats. Biochem J 248: 443447.
(5.) Brem JJ, Pochon DO, Roux JP, Trulls H. 1998. Exploracin diagnstica de la funcin
tiroidea en ovinos. Rev Vet 8/9: 23-26.
(6.) Castillo V. 2001. Cambios de la funcin tiroidea en cachorros alimentados con dietas
comerciales con alto contenido de yodo. On line: http://www.idealibrary.com.

26.- Mujer de 23 aos de edad que presenta en la cara diseminado comedones, ppulas y
pstulas, crecimiento excesivo de vello en mejillas mentn y cuello, acompaada de
seborrea, refiere cada de pelo exagerada. Inici desde la adolescencia. El diagnstico
clnico es:
a)
b)
c)
d)

Acn e Hirsutismo
Sndrome de masculinizacin
Lupus eritematoso discoide
Alopecia androgentica

El hirsutismo es el crecimiento excesivo de vello terminal en mujeres siguiendo un patrn


masculino de distribucin, en zonas andrgeno-dependientes: patillas, barbilla, cuello,
areolas mamarias, trax, en rea inmediatamente superior o inferior al ombligo, as como
en muslos, espalda. Frecuentemente se asocia a acn, calvicie con patrn masculino
(alopecia andrognica) e irregularidades menstruales.
Es un trastorno que afecta aproximadamente al 10% de las mujeres en edad frtil, y
puede ser leve, lo que representa una variacin del patrn de crecimiento normal, y en
raras ocasiones es signo de un trastorno subyacente grave.
Por lo general, es idioptico, pero puede estar relacionado al exceso de andrgenos,
como el sndrome de ovario poliqustico o la hiperplasia suprarrenal congnita.
Los pacientes con acn presentan de un modo ms o menos frecuente seborrea de cuero
cabelludo, con o sin pitiriasis.
Otras alteraciones andrgeno-dependientes pueden asociarse, pero no de un modo
habitual, como sucede con el hirsutismo y la alopeca, que pueden presentarse en menos
del 10% de las pacientes.
En el Sndrome de Cushing Hipofisario, la ACTH estimula la produccin suprarrenal de
hormonas andro-gnicas, resultando esto en acn e hirsutismo.
En el sndrome de poliquistosis ovrica (Stein-Leventhal), hay un incremento de la
secrecin gonadal de andrgenos a nivel del hilio y de la teca interna. La hipersecrecin
de andrgenos dotados de bioactividad puede inducir acn e hirsutismo. La presencia de
acn en una mujer -adolescente o adulta - con niveles de testosterona srica superiores a
300 ng/dl sugiere la presencia de un tumor ovrico.
En la Anorexia Nerviosa, el acn se desencadena generalmente en la fase de
recuperacin, donde puede coexistir con un ovario poliqustico. En fase de amenorrea
LHRH, LH, FSH, estrgenos y progesterona estn disminudos. El cortisol plasmtico est
elevado con vida media prolongada y prdida de la variacin diurna, otro factor que puede
producir acn.
No hay que olvidar aquellos sndromes donde el acn coexiste con importantes trastornos
osteoarti-culares, como ser el Sndrome de Apert y el Sndrome SAPHO.
El acn se halla generalmente presente en los pacientes epilpticos - tal vez provocado
en gran medida por la medicacin - y en pacientes psiquitricos (manaco-depresivos y
obsesivo-compulsivos). Es importante tener en cuenta la asociacin de una neurosis de
angustia inespecfica y el acn.

1. Azziz R, Carmina E, Sawaya ME. Idiopathic hirsutism. Endocr Rev 2000;21:34762.


Hirsutismo. Gonzalez Guerra. Servicio de dermatologa.Fundacin Jimnez Daz (Madrid)

27.- Se trata de femenino de 54 aos de edad afebril con antecedente de pancreatitis


crnica, presenta una masa abdominal palpable y amilasa srica persistentemente
elevada. La mayor probabilidad de diagnstico es:

a)
b)
c)
d)

Cistadenoma pancretico.
Carcinoma pancretico.
Coleccin pancretica aguda
Pseudoquiste pancretico.

El pseudoquiste pancretico es una coleccin de jugo pancretico localizada, por lo


general, en el interior o alrededor del parnquima pancretico. El pseudoquiste
pancretico est confinado por una capa no epitelializada de tejido necrtico,
fibrtico y de granulacin, que se desarrolla tras una lesin pancretica. Para su
formacin requiere un mnimo de cuatro semanas desde que ese dao se produce.
El pseudoquiste pancretico es una complicacin tanto de la pancreatitis aguda
como de la crnica. Si bien la mayora de los pseudoquistes pancreticos se
localizan en la cabeza y el cuerpo del pncreas, hasta un 20% de los mismos son
extrapancreticos (1). Se han descrito pseudoquistes pancreticos en mltiples
localizaciones, como cavidad pleural, mediastino y pelvis (2). Se presenta el caso
de un pseudoquiste pancretico de localizacin heptica que apareci en el curso
de una agudizacin de una pancreatitis crnica, y que se resolvi sin necesidad de
drenaje.
Diagnstico:
Manifestaciones Clnicas:
1.
2.
3.
4.
5.
6.
7.
8.

Pacientes en la cuarta o quinta dcada de vida, antecedentes etiolgicos.


Pacientes con pancreatitis aguda que no resuelve luego de 5 a 7 das de
tratamiento o luego de mejora recae.
Sensacin de cuerpo extrao y pesadez en la mitad superior del abdomen.
Si pancreatitis crnica, dolor abdominal o sntomas por compresin de vscera.
Nauseas, vmitos y prdida de peso por obstruccin duodenal.
Ictero, si compresin del coldoco.
Masa en abdomen superior, lisa y dura, muchas veces insensible.
Ms raramente ascitis y derrame pleural.

Complementarios Diagnsticos:
1.
2.

Ultrasonografa Abdominal: Muchas veces diagnstico, preferido para vigilancia.


TAC: Ideal para diagnstico.

Bibliografa
1. Hamm VB, Franzen N. Atypically located pancreatic pseudocyst in liver,
spleen, stomach wall and mediastinum: their CT diagnosis. Rofo 1993; 159 (6):
522-7
2. Vitas GJ, Sarr MG. Selected management of pancreatic pseudocyst:
Operative versus expectant management. Surgery 1992; 111 (2): 123-30.
3. Mofredj A, Cadranel JF, Dautreaux M, Kazerouni F, Hadj-Nacer K, Deplaix P,
et al. Pancreatic pseudocyst located in the liver: a case report and literature
review. J Clin Gastroenterol 2000; 30 (1): 81-3
4. Balzan S, Kianmanesh R, Farges O, Sauvanet A, O'toole D, Levy P, et al. Right
intrahepatic pseudocyst following acute pancreatitis: an unusual location after acute
pancreatitis. J Hepatobiliary Pancreat Surg 2005; 12 (2): 135-7.

28.- Se trata de femenino de 65 aos que desde hace dos semanas presenta ictericia.
Hace dos das se aade confusin mental. A la EF FR 20, FC 110, TA 90/60,
Temperatura 39, no responde a comandos verbales, pero se aleja del dolor que se
provoca al palpar el hipocondrio derecho y epigastrio. Los datos clnicos en el caso
anterior son sugestivos de:

a)
b)
c)
d)

Coledocolitiasis
Cncer de pncreas
Colangitis
Cirrosis

Diagnstico
El diagnstico de colangitis se basa en la asociacin de signos y sntomas de infeccin
con los propios de una obstruccin biliar. La presentacin clsica es la aparicin de dolor
en hipocondrio derecho o epigastrio junto a fiebre, generalmente alta, con escalofros, e
ictericia (trada de Charcot). Cuando se aade confusin mental y shock (sepsis) se
denomina Pentada de Reynolds, que se observa con menor frecuencia, pero
habitualmente se asocia con una colangitis supurada grave. Sin embargo, la correlacin
entre la clnica tpica, las formas atpicas y la presencia de pus en la va biliar es pobre y
en muchos casos de colangitis faltan algunos de estos rasgos. Algunos enfermos, sobre

todo de edad avanzada, pueden tener confusin mental o shock sin fiebre, o existir una
leucocitiosis con desviacin izquierda como nica manifestacin de la infeccin, por lo que
se debe sospechar la existencia de una colangitis subyacente, sobre todo en enfermos de
edad con algunas de estas manifestaciones. El laboratorio muestra hallazgos de
obstruccin biliar con aumento variable de la bilirrubina y enzimas de colestasis. Es
habitual encontrar leucocitosis con desviacin izquierda, siendo por lo general las cifras
ms altas, en torno a 20.000 por mm3 , reflejo de las formas ms graves. La ecografa es
la tcnica de eleccin para detectar la existencia de obstruccin biliar por su elevada
eficacia y versatilidad, pudindose detectar tambin complicaciones de la colangitis, como
el absceso heptico. Dentro de la colangitis se pueden establecer unas formas leves,
generalmente de buen pronstico, y unas formas graves, donde se concentra la mayor
parte de la mortalidad. Las primeras se manifiestan como cuadros febriles sin signos de
afectacin sistmica que se autolimitan espontneamente o bajo tratamiento mdico en
24-48 horas. Las formas inicialmente graves son las que asocian confusin mental,
hipotensin, shock o fracaso renal. La edad es un factor constante de aumento de la
morbimortalidad.
Referencias bibliogrficas
1. Bilhartz LE, Horton JD. Gallstone disease and its complications. En: Gastrointestinal
and liver diseases. Sleisenger and Fordtran. Filadelfia: WB Saunders Co., 1998; 948-972.
2. Chung-Mau L, Chi-Leung L, Lai ECS, Sheuns-Tat F, Wong J. Early versus delayed
laparoscopic cholecystectomy for treatment of acute cholecystitis. Ann Surg 1996; 223:
37-42.
3. Hamy A, Visset J, Likholatnikov D, Lerta F, Gibaud H, Savigny B et al. Percutaneus
cholecystostomy for acute cholecystitis in critically ill patients. Surgery 1997; 121: 398-401.
4. Harris A, Chong Hen Chang A, Torres-Viera C, Hammett R, Carr-Locke D. Metaanalysis of antibiotic prophylaxis in endoscopic retrograde cholangiopancreatography
(ERCP). Endoscopy 1999; 31: 718-724.
5. Hermann RE. Surgery for acute and chronic cholecystitis. Surg Clin North Am 1990; 70:
1.263-1.275.
6. Koo Kp, Thirlby RC. Laparoscopic cholecystectomy in acute cholecystitis. What is the
optimal timing for operation? Arch Surg 1996; 131: 540-545.
7. Lai ECS, Mok FPT, Tan ESY, Lo CM, Fan ST, You KT et al. Endoscopic biliary drainage
for severe acute cholangitis. N Engl J Med 1992; 326: 1.582-1.586.
8. Marton KI, Doubilet P. How to image de gallbladder in suspected cholecystitis. Ann Int
Med 1988; 109: 722-727.
9. Van den Hazel SJ, Speelman P, Tytgat GNJ, Dankert J, Van Leeuwen DJ. Role of
antibiotics in the treatment and prevention of acute and recurrent cholangitis. Clin Infect
Dis 1994; 19: 279-286.

10. Westphal J-F, Brogard J-M. Biliary tract infections. A guide to drug treatment. Drugs
1999; 57: 81-91.

29.- Se trata de masculino diagnosticado con sfilis congnita tarda, una de las
manifestaciones de esta infeccin es:
a)
b)
c)
d)

Queratitis
Hepatomegalia
Rinorrea
Pseudoparlisis de Parrot

La sfilis congnita tarda se manifiesta despus de los 2 aos de vida, el sntoma ms


frecuente es la queratitis, que es parte de la trada de Hutchinson (queratitis, hipocausia y
dientes en tonel o Hutchinson), as como deformidades seas (tibias en sable, frente
olmpica, engrosamiento clavicular), rodilla de clutton, paresia juvenil y tabes dorsal. La
hepatomegalia, rinorrea, roseola sifiltica y la pseudoparlisis de
parrot son
manifestaciones de la sfilis precoz que se manifiesta en los primeros dos aos de vida.
Evans HE, Frenkel LD: Congenital Syphilis. Clin Perinatol 1994; 21(1)
Honeyman: Manual de Dermatologa, 2da ed. 1988; pg. 46
Lukehart SA, Holmes KK: Sfilis. en: Principios de Medicina Interna. 12 ed. Ed. Harrison.
Cap. 128, pg. 765

30.- Masculino de 61 aos que llega al servicio de urgencias con dolor torcico de ms de
12 hrs. de evolucin, que inicia en forma sbita, mejora al estar sentado, no tiene
antecedentes de importancia, habr que descartar de primera instancia:

a)
b)
c)
d)

TEP
Enfermedad cido pptica
Cardiopata isqumica
Lesin de grandes vasos

El manejo de un paciente con este tipo de dolor, requiere una evaluacin de la severidad,
localizacin y caractersticas peculiares de dicho dolor. Muy pocos sntomas suponen una
urgencia tan obligatoria como lo es el dolor torcico. Tanto el mdico como el paciente
saben que la isquemia miocrdica puede ser causa de muerte sbita, pudiendo generar
ansiedad en ambos. La importancia y dificultad en la valoracin del dolor torcico radica

en la multitud de causas posibles y en el diferente pronstico segn la patologa


subyacente. Al problema diagnstico inherente a un sndrome esencialmente clnico, se
aade la dificultad de etiquetar el dolor en poco tiempo (ayudados slo por la clnica, una
Rx de trax y un ECG), dada la importancia de iniciar con prontitud el tratamiento ms
adecuado en los pacientes con patologa potencialmente letal. Aunque el dolor o la
molestia precordial constituye una de las manifestaciones principales de cardiopata, es
muy importante recordar que puede originarse no slo en el corazn, sino tambin en:
estructuras cardiacas intratorcicas como la aorta, la arteria pulmonar, rbol
broncopulmonar, pleura, mediastino, esfago y diafragma; tejidos del cuello o la pared
torcica, incluidos piel, msculos torcicos, regin cervicodorsal, uniones costocondrales,
mamas, nervios sensoriales o mdula espinal y rganos abdominales como estmago,
duodeno, pncreas o vescula biliar; adems de dolor artificial o funcional.
Manifestaciones clnicas en cardiopata isqumica
Estas se pueden agrupar en cuatro grandes grupos o categoras, las cuales son:
Angina de pecho: en donde la obstruccin del riego arterial no es lo suficientemente
persistente como para causar muerte del tejido muscular cardaco; hay tres tipos que son
la angina estable, la de prinzmetal y la inestable.
Infarto del miocardio: en este caso la obstruccin del riego arterial es lo suficientemente
duradera o persistente como para causar necrosis tisular isqumica del miocardio.
Cardiopata isqumica crnica: son pacientes que generalmente han sufrido uno o ms
ataques cardacos y han sobrevivido a ellos, pero continan teniendo problemas
cardacos debido a que la parte del miocardio que no muere se hipertrofia para suplir las
necesidades del cuerpo y esto a su vez causa un aumento de la demanda cardaca
debido al aumento de los componentes estructurales de las clulas cardacas, trayendo
ms problemas porque no se podr suplir adecuadamente al corazn debido a la
obstruccin coronaria. Estos pacientes constituyen el 50% de los que reciben trasplantes
cardacos.
Muerte sbita cardaca: Es el paro cardaco en el cual se presentaron sntomas en una
hora antes de la muerte, o no se presentaron nunca. Causas: aterosclerosis coronaria,
estenosis artica, hipertensin sistmica, comnmente arritmias letales (asistlicas y
fibrilacin ventricular).
BIBLIOGRAFIA

1. Goldman L., Braunwald E. Molestias torcicas y Palpitaciones. En Isselbacher KJ.,


Braunwald E., Wilson JD., Fauci AS., Kasper DL., eds. Harrison, Principios de
Medicina Interna. McGraw-Hill. Interamericana de Espaa. 1994.
2. Braunwald E. Tratado de Cardiologa. Interamericana. Mcgraw-Hill. 1993.
3. Harkins SW. Geriatric pain. Pain perceptions in the old age. Clin Geriatric Med
1996.
4. Coto lpez, A., Morales JM., Gutierrez Rodero, F., Gonzalez E., .Dolor Torcico.

Manual de diagnstico y teraputica mdica. Gutierrez Rodero F y Garca Daz JD.


2 ed. Madrid, 1990; pag. 165-172.
5. Durn Serantes, M., Caldern de la Barca Gzquez, J.M., Romero Moreno M.,
Martinez Guilln, J., Montero Prez, FJ., Jimenez Murillo, L., Cardiopata
Isqumica ( I ): Angor. Protocolos de actuacin en Medicina de Urgencias.
Jimenez Murillo L y Montero Prez FJ. Mosby/Doyma Libros SA. Barcelona
1996; pg. 51-56.
6. James H. Chesebro. La clnica del dolor torcico en el Servicio de Urgencias: abordaje
de los pacientes y relacin coste-eficacia. Grandes temas de la cardiologa: avances
hacia el cambio de siglo. 1998, American College of Cardiology.
7. Tresch DD, Aronow Ws. Clinical manifestations and clinical diagnosis of coronary artery
disease. Clin Geriatr Med. 1996.
8. Owens, G.M.: Chest pain. Primary Care, 1986. 13; pg: 55-61.
9. Rutherford, J.D.; Braunwald, E.: Diagnstico diferencial del dolor precordial. En:
Braunwald E: Tratado de Cardiologa, 4 edicin. Interamericana McGraw-Hill,
Madrid, 1993; pg: 1448-1449.
9. Williams, E.S.: Approach to the patient with chest pain. En: Kelly WN, ed. Textbook
of Internal Medicine. Filadelfia, J.B. Lippincott Company, 1989; pg. 374-379.

31.- Masculino de 77 aos portador de virus C y diabtico desde hace 10 aos, que
presenta lesiones ampollosas de contenido serohemorrgico en extremidades superiores
e inferiores, y erosiones a nivel de la mucosa bucal. La biopsia cutnea muestra una
ampolla subepidrmica, se realiza inmunofluorescencia directa que muestra la presencia
de depsitos de IgG a nivel de la membrana basal. El diagnstico ms probable es:
a)
b)
c)
d)

Dermatitis herpetiforme.
Toxicodermia ampollosa.
Penfigoide ampolloso.
Porfiria cutnea tarda.

Penfigoide ampolloso
Casi siempre afecta a mayores de 60 aos. Se trata de una enfermedad ampollosa
crnica autoinmune causada por la unin de autoanticuerpos tipo IgG y complemento C3
al antgeno del penfigoide, una glucoproteina transmembrana de 180 (antgeno menor) o
230 kD (antgeno mayor, desmoplaquina I) situada en los hemidesmosomas del
queratinocito basal, a nivel de la lmina lcida. Aunque se haba dicho que el penfigoide
poda ser paraneoplsico, existen estudios epidemiolgicos que demuestran que el
incremento de carcinomas asociados se debe unicamente a que ambas patologas, el
penfigoide y el cncer, se dan con mayor frecuencia al mismo grupo de edad.
Patognesis
Se cree que la unin de los auto anticuerpos IgG al antgeno del penfigoide produce
activacin del complemento, lo que da lugar a la sntesis de C3a y C5a que causan
desgranulacin de los mastocitos. stos a su vez liberan mediadores entre los que

destaca el factor quimiotctico de los eosinfilos. Los eosinfilos reclutados liberan


enzimas lisosomales que destruyen los hemidesmosomas y los filamentos de anclaje con
lo que se separa la unin dermoepidrmica en la lmina lcida.
Clnica
Se caracteriza por lesiones ampollosas grandes y tensas de contenido seroso o
serohemtico, localizadas o generalizadas, con distribucin preferente en flexuras y que
curan sin dejar cicatriz residual. La aparicin de ampollas puede ir precedida de un cuadro
urticarial o eczematoso. Pueden ser o no pruriginosas. En cerca del 30% de los casos
existe afectacin mucosa (generalmente mucosa oral).

Penfigoide ampolloso
Diagnstico
El diagnstico se establece por criterios clnicos, histolgicos e inmunopatolgicos.
En el estudio histolgico de la biopsia cutnea de una ampolla se observa una hendidura
subepidrmica en el interior de la cual se aprecia un rico infiltrado inflamatorio mixto con
predominio de eosinfilos que tambin abundan en la dermis.
Mediante inmunofluorescencia directa se comprueba la existencia de depsitos lineales
de IgG y/o C3 en la regin de la membrana basal.
La inmunofluorescencia indirecta, utilizando fluido de la ampolla u orina, pone de relieve
en cerca de la mitad de los pacientes la existencia de IgG y/o C3 circulantes.
Bibliografa
1. Worjnarowska F, Kirtschig G, Highet AS, Venning VA, Khumalo NP. Guidelines for the
management of bullous pemphigoid. Br J Dermatol 2002; 147:214-221.
2. Downham TF, Chapel TA. Bullous pemphigoid. Arch Dermatol 1978; 114:1639-1642.
3. Chosidow O, Saas V, Diquet B. Correlation between the pretreatment number of blisters
and the time to control bullous pemphigoid with Prednisone 1mg/Kg/day. Br J Dermatol
1992; 127:185-195.

4. Fiveson D, Breneman D, Rosen G et al. Nicotinamide and tetracycline therapy of


bullous pemphigoid. Arch Dermatol 1994; 130:753-758.
5. Fleming TE, Korman NJ. Cicatricial pemphigoid. J Am acad Dermatol 2000; 43:571-591.
6. Eisen D, Ellis CN, Voorhees JJ. Topical Cyclosporine for oral bullous disorders. J Am
Acad Dermatol 1990; 23:936-937

32.- A 20-year-old man comes to the physician he has noticed blood in his urine on
several occasions in the past year. Each episode of hematuria occurred in association with
an upper respiratory tract infection or a flulike illness. Physical examination is
unremarkable. A urine dipstick test shows mild proteinuria and microhematuria. Serum
levels of electrolytes, creatinine, and blood urea nitrogen are within normal limits. Serum
levels of IgA are elevated. Which of the following is the most likely diagnosis?

a)
b)
c)
d)

Berger disease
Goodpasture syndrome
Henoch-Schnlein purpura
Postinfectious glomerulonephritis

HEMATURIA GLOMERULAR AISLADA PERSISTENTE:


Definicin
Presencia de hematuria de origen glomerular (con acantocitos, y ocasionalmente cilindros
hemticos), sin otro elemento de inflamacin glomerular, es decir, sin HTA, ni edema,
con proteinuria menor a 1 gm /24 hs, y funcin renal normal, estable.
En caso de hematuria persistente, aislada, las posibilidades principales es que se trate de
Glomerulonefritis por IgA (lo ms frecuente), enfermedad de membrana basal fina, Sd.
de Alport, (raro) o el inicio de una glomerulonefritis crnica (< probable) (15) .Tambin
puede tratarse de una GN Post-Streptoccica, con persistencia de hematuria.
A continucacin se describirn brevemente:
Nefropata por IgA: Enfermedad de Berger y sndrome de Shnlein-Henoch, se
considern por algunos como espectro de una misma enfermedad, siendo Berger la forma
limitada al rin.

Es la Glomerulonefritis aguda ms frecuente. Se da entre los 15 y 35 aos, y es 3


veces ms frecuente en hombres, tiene tres patrones clnicos de presentacin:
1. Como hematuria macroscpica, 24 48 hs. posterior a cuadro respiratorio alto,
asociado a dolor lumbar. Hematuria dura 2 6 das, rara vez presenta HTA o
deterioro de funcin renal, y tiene proteinuria mnima. Recurre hasta en un 50%.
De esta forma se presenta un 40-50%.

2. Como hematuria microscpica, con proteinuria leve, detectada solo en Sed. Orina,
por rutina. 30 40 % se presenta as.

3. Como Sd. Nefrtico (HTA, Edema, Hematuria), pudiendo evolucionar algunos de


estos
casos
como
GNRP.
Se
presentan
as
un
10
%.
Generalmente cursan con proteinuria menor a 1-2 gm/24 hs,. Pocos desarrollan Sd.
nefrtico (10 %)

Existe un grupo de pacientes (20 40 %), que evoluciona con falla renal progresiva,
llegando a IRT en 5 25 aos. Se ha detectado como factor de riesgo de esta evolucin
la presencia de Sd. nefrtico o proteinuria > 1gm/24h, , edad, HTA, deterioro de la funcin
renal, ausencia de hematuria macroscpica. Si no tiene ninguno de estos factores, tiene
poca probabilidad de desarrollar falla renal, y no se ha demostrado que el tratamiento, en
este grupo de bajo riesgo, altere la evolucin de la enfermedad.
Al laboratorio, el complemento es generalmente normal, rara vez elevado. Existe aumento
de IgA circulante entre un 30 50 %, sin ser especfico de esta enfermedad.
Para el diagnstico, se requiere de biopsia renal, donde se identifican por
inmunohistoqumica, la presencia de depsitos de IgA. Biopsia de piel carece de
especificidad y de sensibilidad para diagnstico de enfermedad de Berger.
BIBLIOGRAFA
1.Jennette C, Falk R: Small Vesel Vasculitides. N Engl J Med 337:1512, 1997
2.Kashtan, CE Alport Sndrome and thin glomerular basement Membrane disease. JAM
Soc Nephrol 1998;9:1736.
3.Falk R et al: Primary glomerular disease, en Breneer & Rectors, The Kidney, 6a ed,
BM Brenner (editor). Phyladelfia, Saunders, 2000. pp 1263-1349.
4.Hricik, DE et al: Glomerulonephritis. N Eng J Med 339:889,1998
5.Antony BF. Attack rates of acute nephritis after type 49 streptoccocal infection of the skin
and of the respiratory tract. J Clin Inv, 1969;48:1697.
6.Oliviera DBG. Poststreptococcal glomerulonephritis:getting to know an old enemy. Clin
Rxp Immunol 1997;107:8-10
7.LangeK, et all. Evidence for the in situ origin of poststreptococcal glomerulonephritis:
glomerular localization of endpstreptosin and the clinical significance of the subsequent

antibody response. Clin Nephrol 1983;19:3-10


8.Rodriguez-Iturbe, B. Epidemic poststreptococcal glomerulonephritis, Kidney int 1984,
25:129.
9.Tejani A. Poststreptococcal glomerulonephritis: current clinical and pathologic concepts.
Nephron 1990;55:15.
10. Potter EV. Twelve to seventeen year follow up of patients with poststreptococcal acute
glomerulonephritis in Trinidad. N Engl J Med 1982; 307:725-9.

33.- Paciente masculino de 27 aos que desde hace una semana inicia con erupcin
maculopapular que afecta fundamentalmente a tronco, muslos y palmas, de forma
simtrica. Las lesiones son rojas claras. Refiere Adinamia, astenia, con dolor de garganta
y cefalea leve, no refiere fiebre. Tras realizarle, entre otras pruebas, RPR y FTA-Abs,
ambas son positivas en suero, se le diagnostica de sfilis. De las siguientes afirmaciones
la correcta es:

a) Si el paciente es VIH positivo la actitud no cambia.


b) El tratamiento de eleccin es penicilina G benzatina.
c) El tratamiento siempre debe incluir probenecid.
d) Se trata de una sfilis primaria.

La penicilina es el antibitico de eleccin, con eficacia probada para cualquiera de los


estados de la enfermedad.
1) Para la sfilis primaria se recomienda penicilina G benzatnica 2,4 MUI i/m en 1 sola
dosis. Se prefiere repetir la dosis semanal de penicilina benzatnica (1 o 2 veces) cuando
se presume que la personas no va a concurrir a los controles posteriores y en la
embarazada. Con ello curan ms de 95% de los pacientes. Despus del tratamiento el
VDRL se hace negativo en 1 ao en casi todos los casos.
2) El mismo tratamiento tambin cura a la gran mayora de los enfermos con sfilis
secundaria, aunque algunos expertos recomiendan una segunda dosis con intervalo de 1
semana. El VDRL se negativiza en aproximadamente 2 aos, pero en algunos pacientes
las pruebas serolgicas reagnicas siguen siendo reactivas, con niveles bajos de
anticuerpos.
Para las personas alrgicas a la penicilina no embarazadas la alternativa es tetraciclina 2
g/d v/o fraccionados en 4 tomas diarias o doxiciclina 100 mg v/o c/12 horas,
cualquera de ellas durante 14 das. Para personas que no toleran la tetraciclina, puede
administrarse eritromicina 500 mg c/6 horas v/o por 14 das, aunque es menos eficaz y se
han observado fracasos. Por eso cuando se usan drogas alternativas, el control posterior
debe ser ms estrecho. Si se sospecha mala adherencia al tratamiento o que el paciente

no va a concurrir a los controles, se prefiere desensibilizarlo y tratado con penicilina G


benzatnica. Otra alternativa es la ceftriaxona 1 g/d i/m o i/v por 10 das.
3) La sfilis latente precoz se trata de igual manera.
4) Para los pacientes con sfilis latente tarda, latente de duracin no conocida y
terciaria no neurosfilis, se recomiendan 7,2 MUI de penicilina G benzatnica i/m en 3
dosis con intervalo de 1 semana. En caso de alergia a la penicilina la alternativa es
tetraciclina o doxiciclina en la forma dicha, por 4 semanas. Algunos expertos tratan a
todos los que tienen sfilis cardiovascular con planes de neurosfilis.
5) No se recomienda penicilina G benzatnica para el tratamiento de la neurosfilis pues no
alcanza niveles teraputicos en LCR. El plan para neurosfilis tanto sintomtica como
asintomtica es de penicilina G cristalina 12 a 20 MUI/d i/v en 6 dosis diarias, por 10 a
14 das. Se menciona la alternativa de ceftriaxona 1 g/d i/m o i/v por 14 das, pero se han
observado fallas teraputicas en pacientes infectados VIH con neurosfilis, sfilis latente o
presunta sfilis latente. Aunque no se recomienda la ceftriaxona para el tratamiento de la
neurosfilis, puede aceptarse como antibitico de alternativa para pacientes con
neurosfilis y sfilis secundaria concomitante.
Wardropper AG, Snow M. Neurosyphilis and HIV infection. Int J STD AIDS. 1994;
5(2):146-8.
O'Mahony C, Rodgers C.A., Mendelsohn S.S., Sissons G., et al. Rapidly progressive
syphilis in early HIV infection. Int J STD AIDS. 1997; 8(4): 275-78.
Hook EW, Marra CM. Acquired syphilis in adults. N Eng J Med. 1992; 326 (16):1060-68.
Guidelines for treatment of sexually transmitted disease. MMWR 1998; 47(RR-1):1-118.
Hutchinsom CM, Hook EW. Sfilis en adultos. Clin Med N. A.1990; 6: 1451-76.
Musher DM. Syphilis. Gorbach, Bartlett, Blacklow. Infectious diseases. Ed.Saunders.
1992: 822-28.
Charles D Ch. Sfilis. Sanford J P, Tyrrell D A J, Weller T H, Wolff S M. Infecciones
obsttricas y perinatales. Doyma. 1994:252-69.
Giovangrandi Y, Costa JM, Malka D, Belein V. Maladies infectieuses au cours de la
grossesse (II). Rev Prat (Paris).1995; 45:2215-37.

34.- Se trata de paciente femenino de 30 aos de edad la cual sufre cada sobre su
hombro izquierdo al conducir su bicicleta en una competencia. En la exploracin presenta
deformidad y signo de la tecla de piano en la extremidad distal de la clavcula. El
diagnstico ms probable de la lesin es:

a) b) Fractura de la cabeza humeral.

b) Luxacin escpulo-humeral.
c) Luxacin acromio-clavicular.
d) Fractura de escpula.

La luxacin acromio-clavicular es una de las lesiones traumticas del hombro que da a


da se hace ms comn, en virtud al aumento de la frecuencia de accidentes de trnsito y
deportivos, evidente en la ltima dcada.
El aporte funcional de la articulacin acromio clavicular a la biomecnica del hombro,
fuera de contribuir con el 20% del total de los arcos de movimiento, y quizs ms
importante, es el de actuar como fulcro, sobre el cual se completan la abduccin y flexin
del hombro, importantsimas para el desempeo armnico y funcional de la extremidad
superior.

La mayora de los pacientes sufre la lesin en actividades deportivas y accidentes


automovilsticos, y dependiendo de la duracin e intensidad de las fuerzas aplicadas, se
producirn las lesiones descritas como G I, G II y G III de Allman1:
G I: Lesin intraarticular sin ruptura del complejo capsuloligamentoso acromioclavicular,
generalmente producida por impacto directo de mediana intensidad sobre la cara externa
del acromin.
G II: Subluxacin de la clavcula por ruptura de la cpsula y de los ligamentos
acromioclaviculares, por una fuerza aplicada en direccin superior y lateral, produciendo
inicialmente una rotacin externa de la escpula que toma como punto de pivote los
ligamentos coracoclavculares.
G III: Luxacin completa de la clavcula en direccin postero superior por ruptura de los
ligamentos coracoclaviculares, debido a la progresin de la fuerza lesionante descrita en
el G II.
Generalmente se encuentran desgarros de los msculos deltoides y trapecio en sus
inserciones a nivel de clavcula distal y acromin.

DIAGNSTICO CLNICO
A. Mecanismo de trauma

B. Cuadro clnico
II. Signos de trauma agudo (escoriacin equinosis etc. aspecto postero lateral del
hombro).
II. Dolor
III. Limitacin funcional
IV. Deformidad (signo de la tecla y signo de la charretera)

DIAGNSTICO RADIOLGICO
Se estableci como tcnica estndar en este estudio, de acuerdo a lo indicado en la
literatura habitualmente (L. Bhler 1957 citado por otros, 4), la toma de placas AP de la
articulacin acromio-clavicular (es necesario especificarlo as, pues rutinariamente en las
placas simples de hombro, el tiempo de exposicin mayor hace que la articulacin
acromio- clavicular se vea radiolcida, (Zariczny

1. Weaver, J.K., Dunn, H.: Treatment of acromioclavicular injuries, especially


complete acromioclavicular separation. J. Bone and Joint Surg., 54-A; 1.187-1.194,

35.- Femenino de 47 aos que presenta palpitaciones, debilidad, palidez de tegumentos,


taquicardia, glositis, parestesias dstales y despus ascendentes; y que cursa con ataxia.
Bh con Hb de 8; VCM 110 fL; leucocitos y plaquetas discretamente disminuidos. El
diagnstico de mayor probabilidad es:
a)
b)
c)
d)

Infeccin crnica.
Leucemia.
Hemorragia.
Deficiencia de cobalamina.

Farreras. Medicina interna. Cap 206. Decimoquinta edicin 2004, Pp. 1652-1653. Dado
que la cobalamina en el trofismo adecuado de la piel y las mucosas y el mantenimiento de
la mielinizacin adecuada, pueden aparecer alteraciones digestivas y neurolgicas, como
glositis (de Hunter) y trastornos gastrointestinales inespecficos (flatulencia, digestiones
pesadas), aunque algunos pacientes presentan diarrea que slo cede con el tratamiento
con cobalamina. Las manifestaciones neurolgicas se deben a degeneracin axonal y
desmielinizacin de los cordones medulares posteriores (degeneracin combinada
subaguda).

36.- Se trata de paciente de 32 aos con antecedente de cuadros catarrales y


tabaquismo crnico, inicia con molestias farngeas que progresan rpidamente a dolor
intenso de garganta e impide la deglucin acompaado de fiebre. Al acudir al servicio y

ser asistido, se niega a acostarse en camilla, permaneciendo sentado e inclinado hacia


delante. El diagnstico ms probable es:
a)
b)
c)
d)

Edema de Reinke.
Laringitis catarral aguda.
Angina de Ludwick.
Amigdalitis bacteriana.

Edema de Reinke

Son lesiones en las que se produce un acumulo de un liquido fluido, gelatinoso, bajo la
cubierta mucosa de las cuerdas vocales. Son generados por fenmenos de inflamacin
crnica debidas al abuso vocal y al consumo importante de tabaco que encontramos en
ms del 90% de los pacientes. Esta descrito que puede aparecer en casos de
hipotiroidismo y asociado al reflujo gastroesofgico.

El edema suele afectar a ambas cuerdas vocales y es de aparicin ms frecuente en los


varones. Se presenta como una disfona progresiva con voz ronca y con un tono ms
bajo, voces ms graves y con menos potencia vocal y tendencia a la fatiga en los casos
ms intensos.
El edema de Reinke puede mejorar claramente evitando el consumo de tabaco, el
carraspeo y modificando los malos hbitos vocales. Una correcta rehabilitacin vocal ser
el
complemento
adecuado
en
la
gran
mayora
de
los
casos.
Se considerar el tratamiento con microciruga laringea en los casos crnicos, refractarios
al tratamiento conservador y en las que la calidad de voz es mala incapacitando al
paciente para su vida de relacin o de trabajo. En algunos profesionales de la voz
encontramos edemas de Reinke con voces ms graves, clidas que conforman una firma
vocal caracterstica que les identifica y que no les interesa cambiar por lo que no
precisaran de un tratamiento agresivo.

Clnica
Disfona de larga
evolucin con voz
grave sobre todo
matutina
Disnea ocasional
Relacin
con
abuso del tabaco
Predominio
en
varones de 50-60
aos,
pero

Exploracin
Laringoscopia
indirecta:
- Irritacin y edema
organizado
en
forma de bolsas
en
los
2/3
anteriores
de
ambos pliegues
- Defecto de cierre
posterior

Actitud y consejos
Medidas de higiene
vocal
Protectores
gstricos
Microciruga
larngea
con
incisin y aspirado
del edema en 1 2
intervenciones
Supresin absoluta

Evolucin
Mejora
Recidivas
si
persisten hbitos
tabquicos
Riesgo
de
cicatrices vocales
No
degeneran
aunque se asocian
a laringitis crnicas

aumenta
la
incidencia
en
mujeres
Relacin
con
reflujo
gastroesofgico

- Aspecto
gelatinoso
rojizo

del tabaco
Rehabilitacin
logopdica
ocasionalmente

Bruch JM. Hoarseness in adults [Internet]. Waltham, MA: UpToDate, Rose, BD (Ed),
version 17.1 ; 2009 [acceso 6/4/2009]. Disponible en:
Chadha NK, James AL. Tratamiento antiviral adyuvante para la papilomatosis respiratoria
recurrente (Revisin Cochrane traducida). En: La Biblioteca Cochrane Plus, nmero 3,
2008. Oxford, Update Software Ltd. Disponible en: Colton R, Casper J, Leonard R.
Understanding Voice Problems. A Physiological Perspective for Diagnosis and
Treatment. 3rd ed. Baltimore Philadelphia: Lippincott Williams & Wilkins; 2006
Gallagher TQ, Derkay CS. Recurrent respiratory papillomatosis: update 2008. Curr Opin
Otolaryngol Head Neck Surg. 2008;16(6):536-42
Garca-Tapia R, Cobeta I. Clasificacin de las disfonas. En: Diagnstico y tratamiento de
los trastornos de la voz. Eds: Garca-Tapia R y Cobeta I. Editorial Garsi, S.A. 1996.
Madrid.
Ishizuka T, Hisada T, Aoki H, Yanagitani N, Kaira K, Utsugi M et al. Gender and age risks
for hoarseness and dysphonia with use of a dry powder fluticasone propionate inhaler
in asthma. Allergy Asthma Proc. 2007 Sep-Oct;28(5):550-6

37.- Femenino de 42 aos de edad, que acude al servicio de oftalmologa para valoracin
por presentar dolor, enrojecimiento y sensacin de cuerpo extrao en ambos ojos, se
diagnstica queratoconjuntivitis sicca. La enfermedad sistmica ms comnmente
asociada a sta patologa es:

a)
b)
c)
d)

Lupus
Granulomatosis de Wegener
Artritis reumatoide
Espondilitis anquilosante

Queratoconjuntivitis Sicca

La queratoconjuntivitis sicca (QCS Figura 1) es la manifestacin ms frequente de la AR


ocular caracterizada por una falta de lubricacin lo que trae como consecuencia la
sensacin de cuerpo extrao o arenilla en los ojos empeorada por actividades que
disminuyan la frecuencia de parpadeo (la lectura y el uso del computador). El test de
Schirmer generalmente da valores disminuidos. El manejo en las formas leves es con
lubricantes tpicos as como el control de otras enfermedades asociadas como meibomitis
o rosacea. En formas moderadas el uso de los tapones lagrimales ha revolucionado el
tratamiento permitiendo conservar las pocas lgrimas disponibles que contienen factores
de crecimiento fisiolgicos y para los casos ms severos, ocasionalmente es necesaria
una tarsorrafia parcial.
1. Sall K, et al. Two Multicenter, Randomized Studies of Efficacy and safety of
Cyclosporine
Ophthalmic Emulsion in Moderate to Severe Dry Disease. Ophthalmology
2000:107(4):631-639.
2. Donato BF, et al. Senile Atrophy of the Human Lacrimal Gland: The contribution chronic
inflammatory disease. Br Ophtalmol 1984;68:674-80.
3. Schein O, et al. Prevalence of dry eye among the elderly. Am J Ophtalmol 1997;
124:732-8.
4. Lemp MA, et al. Recent development in dry eye management. Ophthalmology 1987;
94:1299-1304.
5. Lubniewsky AJ, et al. Diagnosis and management of dry eye and oculary sulfatance
disorders.
Ophthalmogy Clinic of North America 1990;3:575-94.
6. Makie I, et al. Diagnostic implication of tear protein profiles. Br J Ophtalmol 1984;68:332

38.- Masculino de 17 aos, que presenta hemartrosis. Tiene TPT alargado, TP y TT


normales, por lo que tiene una alteracin de la va:
a)
b)
c)
d)

Del Complemento
Intrnseca
Colinrgica
Extrnseca

Manual CTO 7 edicin, p. 723.


La va intrnseca est constituida por la activacin secuencial de los factores XII, XI, IX,.
VIII y V. el tiempo de tromboplastina parcial activada mide la actividad de la coagulacin
intrnseca y sirve para monitorizar el tratamiento con la heparina no fraccionada.

39.- Se trata de paciente femenino con sospecha de lquido libre en la cavidad peritoneal.
Despus de analizar los hallazgos de manera conjunta de las placas de abdomen y en el
ultrasonido abdomino-plvico, Dnde se debern de buscar los cambios de densidad de
manera inicial?
a)
b)
c)
d)

Infradiafragmtico
Fondos de saco de Douglas y de Morrison
Transcavidad de los epiplones
Correderas parieto-clicas

Pedrosa C, Casanova R. Diagnstico por imagen. Mc Graw Hill 2001 p. 209, 210. La
diseminacin del lquido intraperitoneal est producida por la gravedad y la presin
hidrosttica secundaria al movimiento diafragmtico.la pelvis es la parte ms dependiente
de la cavidad peritoneal, su capacidad es de 300 cc, a partir de lo cual el lquido asciende
por las correderas parietoclicas. El lquido plvico desplaza las asas intestinales que
rellenan los espacios plvicos, el leon del lado derecho y el colon sigmoides en el
izquierdo, lo que resulta de la aparicin de una densidad homognea en la pelvis menor,
en contra del patrn poco homogneo de las asas intestinales que tienen gas y lquido. En
la pelvis femenina el lquido se acumula en los recesos laterales de la vejiga y al tero, y
en el hombre puede verse en la fosa vesicorrectal y los recesos plvicos laterales. En el
ultrasonido el diagnstico de pequeas cantidades de lquido debe hacerse
fundamentalmente en el saco de Douglas, en la bolsa de Morrison y en el receso
yuxtaesplnico.

40.- A travs de un estudio de casos y controles se quiere conocer si existe asociacin


entre tabaquismo y cncer de lengua. La variable tabaquismo se medir como Leve (uno
a 10 cigarrillo en 24 hrs), moderado (11 a 20 cigarrillos en 24hrs) severo (ms de 21
cigarrillos en 24 hrs). Como podemos clasificar esta variable:

a)
b)
c)
d)

Cuantitativa continua
Cuantitativa discreta
Cualitativa ordinal
Cuantitativa discontinua

En las variables de tipo ordinal las observaciones se clasifican y ordenan por categoras
segn el grado en que los objetos o eventos poseen una determinada caracterstica. Por

ejemplo, se puede clasificar a las personas con respecto al grado de una enfermedad en
leve, moderado o severo.
Moreno A. Principales medidas en epidemiologa. Rev Salud Pblica Mex, 2000;42(4):
338

41.- Se trata de femenino de 64 aos de edad con la siguiente sintomatologa: sensacin


de plenitud y estreimiento, USG plvico que reporta una imagen qustica en ovario
derecho de 15 por 15 cms. El diagnstico ms probable es:
a)
b)
c)
d)

Cistoadenoma seroso.
Teratoma qustico.
Disgerminoma.
Endometrioma.

Los Tumores de Ovario son una patologa frecuente dentro del contexto de la patologa
femenina. Por esta causa consultan un grupo elevado de mujeres, tanto las consultas de
ginecologa como las de Ciruga propiamente dicha. Las edades oscilan desde las
tempranas hasta las ya avanzadas, siendo el riesgo de degeneracin maligna muy
variable y relacionado con le edad. La experiencia de la clnica revela la alta incidencia de
tumores de ovario en la etapa del climaterio, comprendida entre los 35 y 65 aos de edad
1
.

El cistoadenoma seroso de ovario (CSO) es un tipo de tumor derivado del epitelio


superficial (celmico), formado por reas qusticas. El cistoadenoma seroso de ovario es
el tumor ms frecuente de aquellos que provienen del epitelio celmico superficial. Hay
tumores pequeos macroscpicamente y tumores masivos que ocupan toda la pelvis e
incluso la cavidad abdominal. Estas frecuentes neoplasias qusticas uniloculares estn
tapizadas por clulas epiteliales altas, cilndricas y ciliadas, llenas de un lquido seroso
claro y de superficie lisa con abundantes vasos. Las variedades benigna, limtrofe y
maligna representan, en conjunto, 30% aproximadamente de todos los tumores del ovario.
El riesgo de presentar tumores epiteliales se incrementa con el paso de la edad, ya que
pese a que la declinacin de la funcin ovrica marca el envejecimiento gonadal
progresivo, el ovario humano nunca pierde su capacidad para generar tumores. Por lo
general, cuando es detectado, su tamao es grande, en donde la imagenologa puede
ayudarnos a considerar su diagnstico

1.
Captulo 22 Tumores Benignos de Ovario. En: Novak ER, Jones G., Jokes HW.
Tratado de Ginecologa. 9 ed. Ciudad de la Habana. Editorial Cientfico Tcnica;
1977.p.432 66.
2.
MedlinePlus Enciclopedia Mdica en Espaol: Quistes Ovricos. Disponible en:
http://vsearch.nlm.nih.gov/vivisimo/cgibin/querymeta?v%3Aproject=medlineplusspanish&s
pell=spell&query=Quistes+Ov%C3%A1ricos Acceso: Actualizado 20/6/06.
Captulo XL Tumores Ovricos En: Llusi Botella J, Nez Clavero JA. Tratado de
Ginecologa. Ciudad de la Habana. Editorial Cientfico Tcnica. 1983; T 3.1; p. 751 803.

42.- Femenino que cursa con 36. 5 semanas de gestacin acude al servicio por referir
malestar general, fosfenos, nausea y vmito, aprecia moderada ictericia, usted sospecha
de un sndrome de HELLP. Las alteraciones de laboratorio espera encontrar al confirmar
el diagnstico son :
a)
b)
c)
d)

Trombocitosis, Enzimas heptica elevadas, Anemia hemoltica.


Anemia hemolitica, Trombocitosis, Fosfatasa Alcalina elevada.
Anemia Hemoltica, trombocitopenia, enzimas hepticas elevadas.
Trombocitopenia, Leucopenia, Hipertensin Arterial.

DEFINICIN:
Es una complicacin de la preeclampsia en la cual adems de la Hipertensin
Arterial y proteinuria hay presencia de anemia hemoltica, enzimas hepticas
elevadas y recuento bajo de plaquetas

EPIDEMIOLOGIA:
Se presenta en un 4 a 10% de las preeclmpticas, diagnosticndose anteparto en un
70% de los casos preferentemente antes de las 37 semanas, mientras que el 30%
de los casos restantes enferma en los primeros 7 das del puerperio, sobre todo en
las 48 h iniciales.
La proteinuria e hipertensin pueden estar ausentes en un 15 al 20% de los casos.
Incidencia mayor en multigestantes y en edades avanzadas.
Ocurre ms frecuentemente cuando se demora la salida del feto y cuando se
presenta desprendimiento de la placenta
Mortalidad materna del 24% y mortalidad perinatal del 30-40%.

CLASIFICACION:
Sndrome de HELLP. Clasificacin de Mississipi.
CLASE
Plaquetopeni LDH
a
1
Severa
>600 IU/L
<50000
2
Moderada
>600 IU/L
>50000
<100000
3
Ligera
>600 IU/L
>100000
<150000
PE severa
>150000
<400 IU/L
Eclampsia (sin
HELLP)

AST-ALT
>70 IU/L
>70 IU/L

>40 IU/L
<70 IU/L
<40IU/L

MANIFESTACIONES CLINICAS:
Malestar general, fatiga y molestias inespecficas 90%
Cefalea 70%
Epigastralgia 64%
Vmito 22%
Fosfenos 15%
Visin Borrosa 11%
Acfenos 3%
Ictericia
Anemia no explicada
Oliguria

en el HOMBRO DERECHO y EL CUELLO, adems de las molestias abdominales.

Equimosis en los sitios de punciones venosas, petequias en los sitios de presin del
brazo, pero pueden tener pruebas de Rumpel Leed negativas.
En casos severos se pude presentar ascitis como causa de hipertensin portal.

DIAGNOSTICO:
El diagnstico clnico del sndrome de HELLP se plantea en gestantes o purperas con
preeclampsia severa-eclampsia, excepto en el 15-20%, en las cuales esta asociacin no
puede ser demostrada, en tanto se cumplan los criterios de Sibai:

MANIFESTACIONES CLINICAS:
Malestar general, fatiga y molestias inespecficas 90%
Cefalea 70%
Epigastralgia 64%
Vmito 22%
Fosfenos 15%
Visin Borrosa 11%
Acfenos 3%
Ictericia
Anemia no explicada
Oliguria

Si se aade una HEMORRAGIA HEPTICA, el paciente puede quejarse de dolor en


el HOMBRO DERECHO y EL CUELLO, adems de las molestias abdominales.
Equimosis en los sitios de punciones venosas, petequias en los sitios de presin del
brazo, pero pueden tener pruebas de Rumpel Leed negativas.
En casos severos se pude presentar ascitis como causa de hipertensin portal.

DIAGNOSTICO:
El diagnstico clnico del sndrome de HELLP se plantea en gestantes o purperas con
preeclampsia severa-eclampsia, excepto en el 15-20%, en las cuales esta asociacin no
puede ser demostrada, en tanto se cumplan los criterios de Sibai:

HEMOLISIS
Frotis perifrico anormal (eritrocitos fragmentados)
Hematocrito (>24%)
Bilirrubina indirecta (>1.2mg/dL)
Deshidrogenasa lctica (>218 UI/L)
ENZIMAS HEPTICAS ELEVADAS
LDH >218UI/L
AST >30UI/L
ALT >37UI/L
PLAQUETAS BAJAS
<100.000/mm3

BIBLIOGRAFIA
Sibai baha, El sndrome HELLP. Universidad de Valencia , revista quincenal de
Obstetricia clnica y ginecologa, Octubre 2003.
V. Cararach, Sndrome de HELLP y Repercusiones maternas. X curso intensivo de
formacin continuada materno fetal. Enero de 2003.

Toirac, Abelardo. Sndrome de Weistein HELLP Hospital Ginecoobstetrico Tamara


Bunke. Junio 2002
De la Fuente, David. Sndrome HELLP. Medicina Universitria 2003; 5 (19): 101 -9
Andrea G. Witlin, DO, Baha M. Sibai, MD. Diagnosis and Management of women with
Hemolysis Elevate Liver Enzymes, and Pletelet Count (HELLP) syndrome. Hospital
Physician. Febrero 1999.
CIFUENTES B, Rodrigo. Ginecologa y obstetricia basadas en las evidencias. Bogot:
Distribuna, 2006. Sexta edicin. 447 - 283 p.

43.- Lactante que es trado al servicio con datos de desnutricin severa, anemia clnica,
manifestaciones purpricas y como antecedentes llama la atencin procesos infecciosos
de difcil control. La posibilidad diagnstica es una deficiencia de:
a)
b)
c)
d)

Hierro
B 12
Folatos
Piridoxina

La anemia megaloblstica es un tipo de anemia en donde existe una disminucin de la


sntesis del ADN con detencin de la maduracin que compromete las tres lneas celulares
de la mdula sea (glbulos rojos, glbulos blancos y plaquetas). Las causas que la
producen son numerosas, pero aproximadamente el 95% de los casos es consecuencia de
una deficiencia de vitamina B 12 y/o de cido flico. Las manifestaciones clnicas y
hematolgicas son similares en ambos casos, pero las manifestaciones neurolgicas se
presentan slo en los casos de deficiencia de vitamina B12. El tratamiento est ligado a la
causa que la produce.
ETIOLOGA
La anemia megaloblstica en el nio casi siempre se debe a la carencia de folatos, con
menos frecuencia a la de vitamina B12 y ms raramente a defectos hereditarios de la
absorcin, transporte o metabolismo de estas vitaminas; en ms del 95% de los casos se
debe a carencia de estas vitaminas.
La deficiencia de folatos es relativamente frecuente en madres que presentan desnutricin y escaso
aporte de vegetales en la dieta; en estos casos con frecuencia se asocia a anemia por deficiencia
de hierro. En estas circunstancias no es posible para la madre proporcionar un aporte
suficiente de folatos al feto en rpido crecimiento o al nio durante la lactancia.
En nios mayores, la desnutricin contina siendo la principal causa de deficiencia de
folatos. Otras causas incluyen los defectos de absorcin en sndromes de absorcin
intestinal deficiente, aumento de los requerimientos (especialmente en pacientes con
anemia de clulas falciformes y talasemia) y administracin de medicamentos que
funcionan como antagonistas al inhibir la dihidrofolatoreductasa (triamtereno, sufisoxasol,
trimetoprim).
El cuadro clnico es similar tanto en la deficiencia de folatos como en la de vitamina B12. El
comienzo es insidioso y los primeros sntomas que se presentan son palidez, irritabilidad,
apata, anorexia y debilidad.4
La deficiencia de la vitamina B12 es rara antes de los dos aos de edad; algunos pacientes
presentan signos que permiten sospechar esta deficiencia: la lengua es dolorosa y
adquiere un color rojo intenso, presenta atrofia papilar y signos de glositis recidivante; con

frecuencia existe fiebre moderada y diarrea crnica o episdica. El bazo es palpable en la


mitad de los pacientes severamente anmicos.
La complicacin neurolgica es comn en adultos y muy rara en los nios y depende
principalmente de deficiencia de la vitamina B12. Se inicia con parestesias en pies y dedos
de las manos a consecuencia de neuropata perifrica, asociadas a alteraciones sensitivas;
sin tratamiento el cuadro puede evolucionar a ataxia espstica. Es importante destacar que
la anemia megaloblstica por deficiencia de folatos no se acompaa de las alteraciones
neurolgicas observadas en la deficiencia de vitamina B12.

ANEMIA MEGALOBLASTICA: Revisin bibliogrfica


Jorge Gustavo Romero Valdez, Carlos Eduardo Sandoval Benetti, Csar Luis
Snchez
Dra. Laura Adriana Acosta

44.- La indicacin quirrgica es obligada en un paciente afectado de enterocolitis


necrotizante cuando presente uno de los siguientes signos clnicos:
a)
b)
c)
d)

Heces mucosanguinolentas
Neumoperitoneo
Vmitos biliosos
Distensin abdominal

Enfermedad adquirida que afecta principalmente a RN prematuros o patolgicos y que se


caracteriza por necrosis de la mucosa o de capas incluso ms profundas del intestino,
sobre todo en el leon terminal y con menos frecuencia del colon y del intestino delgado
proximal
Sntomas, signos y diagnstico
La enfermedad puede iniciarse con un leo que se manifiesta con distensin abdominal,
residuos gstricos biliosos (tras las tomas) que pueden progresar a vmitos de bilis o
presencia de sangre macroscpica o microscpica en las heces. La sepsis puede ponerse
de manifiesto con letargia, inestabilidad trmica, aumento de las crisis de apnea y acidosis
metablica.
La deteccin sistemtica de sangre oculta o de sustancias reductoras en las heces de
los prematuros (que han recibido alimentacin oral o enteral) puede ayudar a diagnosticar
la ECN. Las radiografas iniciales pueden ser inespecficas o mostrar slo el leo. Sin
embargo, un asa intestinal fija y dilatada que no cambia en las radiografas posteriores
indica una ECN. Las radiografas diagnsticas son las que muestran neumatosis intestinal y

gas en el territorio de la vena porta. El neumoperitoneo es un signo de perforacin intestinal


e indica la necesidad urgente de una intervencin quirrgica.

BOL PEDIATR 2006; 46(SUPL. 1): 172-178


Protocolos de Neonatologa
Enterocolitis necrotizante neonatal
I. FERNNDEZ JIMNEZ1, I. DE LAS CUEVAS TERN2

45.- Masculino de 39 aos. Acude a la consulta solicitando un examen mdico. Tiene


antecedente familiar de diabetes en madre y dos tas maternas. El paciente es obeso
desde los 20 aos de edad. EF: peso 92 kg, estatura 1.75 m, TA 130/90, FC 78x,
obesidad generalizada. Laboratorio: glucosa 132 mg/dL, resto de qumica sangunea
normal. Se solicita otra glucemia que es de 129 mg/dL y una hemoglobina glucosilada con
resultado de 7 %.
El diagnstico del paciente es:
a)
b)
c)
d)

Diabetes mellitus
Intolerancia a la glucosa
Tolerancia a la glucosa normal
Intolerancia a la glucosa de ayuno

DIAGNSTICO DE DM
1. Sntomas de DM m
ms una glucosa
plasm
plasmtica casual 200 mg/dl.
2. Glucemia de ayuno 126 mg/dl.
3. Hemoglobina glucosilada 6.5%.
4. Glucemia 200mg/dl a las 2 horas
despu
despus de 75 g de glucosa oral.

46.- Masculino de 70 aos, diagnosticado de carcinoma de prstata, en este momento


acude al servicio de urgencias por presentar confusin mental, nuseas, vmitos y
estreimiento. Se realizan pruebas de laboratorio y destaca una calcemia de 16mg/Dl. La
primera medida teraputica que debe tomarse en ste paciente es:
a) Hormonoterapia (leuprolide y estrgenos).
b) Difosfonatos por va oral.
c) Glucocorticoides por va intravenosa.
d) Administracin de Solucin salina y furosemida por va intravenosa.

Los sntomas que provoca la hipercalcemia estn en relacin con sus niveles en sangre.
Valores comprendidos entre 10,5 y 12 gr/dl no suelen provocar sntomas; a partir de estos
valores la sintomatologa es progresiva, afectando con ms intensidad y gravedad a
todos los sistemas del organismo. No est establecido un orden de aparicin de la
sintomatologa, ni su correspondencia con los valores de calcemia.
Sntomas de la hipercalcemia
Sistema nervioso central
Desrdenes mentales
Dificultades cognitivas
Ansiedad
Depresin
Confusin, estupor y coma
Calcificacin corneal
Suicidios (descritos aisladamente)
Sistema neuro-muscular
Fatiga o cansancio muscular
Mialgias
Descenso de la funcin de msculos respiratorios
Laxitud articular
Sistema renal
Nefrolitiasis
Diabetes inspida nefrognica (poliuria y polidipsia)
Deshidratacin
Nefrocalcinosis
Sistema gastrointestinal
Nuseas y vmitos
Anorexia
Estreimiento
Dolor abdominal
Pancreatitis
lcera pptica
Sistema esqueltico
Dolor seo

Artritis
Osteoporosis
Ostetis fibrosa qustica
Resorcin subperistica
Quistes seos
Embarazo
Hipoparatiroidismo neonatal
Tetania neonatal
Bajo peso al nacer
Retraso crecimiento intrauterino
Hiperemesis gravdica
Alta morbilidad neonatal y materna
Partos pretrmino
Sistema cardiovascular
Hipertensin arterial
Calcificacin vascular
Calcificacin miocrdica
Hipertrofia miocrdica
Acortamiento intervalo QT
Arritmias cardacas
Otros
Queratitis
Conjuntivitis
Anemia normoctica normocrmica
Gota o pseudogota

El tratamiento de la hipercalcemia est orientado fundamentalmente a eliminar la causa


que la produce. En los casos sintomticos se requiere de un tratamiento inicial especfico.
Debe ser tratada la que presenta sntomas o supera los 14 mg/dl. Los mecanismos para
actuar sobre la hipercalcemia son:
Aumento de la eliminacin renal de calcio.
Disminucin de la absorcin intestinal de calcio.
Disminucin de la resorcin sea de calcio.
El tratamiento siempre debe iniciarse con la rehidratacin del paciente, que produce un
aumento del volumen extracelular, alcanzando una ganancia de volumen de 1,5- 2,5 litros
en las primeras 24 horas.
A continuacin, se utilizar un diurtico de asa, tipo furosemida, que aumenta la excrecin
renal de sodio y calcio. La dosis de diurtico se ajustar en funcin de las cifras de calcio
a lo largo del tratamiento.
Cuando la funcin renal est comprometida puede recurrirse a la hemodilisis con calcio
bajo en el lquido de dilisis.

En los casos en los que est implicado un aumento de la resorcin sea como causa de
hipercalcemia, deberemos controlarla con:
Bisfosfonatos: disponemos de clodronato y pamidronato. El ms empleado es el
primero, porque la dosificacin es ms fcil. Su administracin en el caso de la
hipercalcemia es intravenosa, inicia el efecto a las 72 horas y alcanza el mximo a
la semana.
Calcitonina: acta ms rpido que los bisfosfonatos, pero slo mantiene el efecto
durante 72 horas.
Otros frmacos antirresortivos: Mitramicina y el nitrato de galio, con menor
experiencia, pueden usarse en caso de fracaso de los anteriores.
En aquellos casos en los que est aumentada la absorcin intestinal de calcio, como en la
produccin endgena de vitamina D (enfermedades granulomatosas o linfomas) deben
tratarse con glucocorticoides.
LECTURAS RECOMENDADAS
1. Achogue HJ, Madias EN. Changes in plasma potassium concentration
during acute acid base disturbances. Am J Med 71:456, 1981
2. Humphreys MH. Urgencias por trastornos en los lquidos, electrolitos y
equilibrio cido bsico. En: Diagnstico y Tratamiento de Urgencias.
Editado por MT Ho y CE Saunders. Editorial El Manual Moderno. Mxico
DF, 1991
3. Mora JM, Delgado VA, D'Achiardi R. Trastornos del potasio. En: Manual de
Urgencias en Medicina Interna. Captulo Central. Ediciones Act Med
Colomb. Santaf de Bogot, 1994
Silva E, Lpez C, Ramrez J. Trastornos del potasio. En: Compendio de Teraputica.
Segunda edicin. Asociacin Colombiana de Medicina Interna. Captulo Central.
Ediciones Act Med

47.- Masculino de 35 aos con datos clnicos de importancia para diagnstico de leo
mecnico simple, los datos radiogrficos de esta patologa son:
a) Retencin de contenido en colon, sin niveles hidroaereos
b) Liquido y aire libres en cavidad peritoneal
c) Niveles hidroareos a diferentes alturas, con dilatacin de asas
e) Asas fijas y lisas, con niveles largos

ILEO MECANICO
A/ Extraluminal
-40%)

ceso)
B/ Parietal

C/ Intraluminal

es
leo mecnico
Los signos radiolgicos de una obstruccin intestinal simple son:

Asas distendidas proximales por retencin de lquido y gas.


Niveles hidroareos.
Reduccin o ausencia de gas y materia fecal en colon. Ante el hallazgo de gas en
colon se descarta la existencia de una obstruccin de intestino delgado.

BIBLIOGRAFA
1) Di Lorenzo C. Pseudo-obstrucction. Current approaches. Gastroenterology
1999;116:980-987
2) Stanghellini V, Cammilleri M, Malagelada JR.Chronic idiopatic intestinal
pseudobstruction: clinical and intestinal manometric findings. Gut 1987; 28: 5-12
3) Scolapio JS, Ukleja A, Bouras EP et al. Nutritional management of chronic intestinal
pseudo-obstruction. J Clin Gastroenterol 1991; 28:306-312.
4) Malagelada JR, Distrutti E. Managementof gastrointestinal motility disorders. A practical
guide to drug selection and appropiate ancillary measures. Drugs 1996;52:494-506

48.- Se trata de paciente masculino de 13 aos inicia con fiebre de 38.Oc y dolor en el
cuadrante inferior derecho del abdomen, nusea sin vmito. A la EF se encuentra
abdomen blando, no hiperestesias, a la palpacin del cuadrante inferior derecho dolor en
el punto de McBurney, Psoas y Obturador positivos, timpanismo normal y peristalsis con
algunos tonos metlicos en cuadrantes derechos:

La etiologa ms frecuente de la apendicitis aguda es:


a) Infecciosa
b) Vascular
c) Primaria
d) Obstructiva

ETIOPATOGENIA

Obstruccin del lumen apendicular.


Hiperplasia linfoide.
Obstruccin por fecalito, parsito o cuerpo
extrao.
Inicialmente: isquemia, edema y
acumulacin de moco Pus por las
bacterias. Destruccin de la pared apend.
(apend. Aguda supurada), dolor en FID.

ETIOPATOGENIA

Ms tarde: trombosis de vasos sanguneos,


necrosis de la pared y gangrena (migracin
de bacterias), irritacin peritoneal localizada
o generalizada.
Finalmente: Se perfora con escape de
contenido purulento; peritonitis.
Plastron o absceso apendicular.

49.- Femenino de 28 aos de edad con AV en los dos ojos de 20/30. Al explorarlo la
paciente entrecierra los ojos. De cerca ve bien, pero se cansa al estar leyendo despus

de un tiempo. El resto de la exploracin es normal. El diagnstico ms probable de sta


paciente es:
a)
b)
c)
d)

Miopa
Astigmatismo
Hipermetropa
Miopa y presbiopa

AMETROPAS
Cuando los rayos de luz paralelos que inciden en un globo ocular enfocan a nivel de la
retina, dicha condicin se conoce como emetropa. Si, por el contrario, estos rayos de luz
no enfocan a nivel de la retina, se habla entonces de ametropa. En esta ltima el foco
principal se encuentra por delante (MIOPA) o por detras de la retina (HIPERMETROPA),
tanto ms retirado de ella cuanto mayor sea la ametropa, o bien pierde su caracterstica
puntiforme para formar dos lneas focales principales perpendiculares entre s
(ASTIGMATISMO).
El astigmatismo es un defecto refractivo que hace que todo se vea deformado o
desenfocado, tanto en visin cercana como en visin lejana. En el ojo astgmata, la
crnea tiene forma de elipse, esto hace que las imgenes no enfoquen en un foco nico
como en el ojo normal.
El astigmatismo puede combinarse con miopa, dando lugar a un astigmatismo mipico
donde adems de ver desenfocado de lejos, las imgenes tanto cercanas como lejanas
se perciben deformes.
El astigmatismo puede combinarse con hipermetropa, dando lugar a un astigmatismo
hipermetrpico donde adems de ver desenfocado de cerca, las imgenes tanto cercanas
como lejanas se perciben deformes.
El sntoma ms importante del astigmatismo es la percepcin de imgenes
distorsionadas. As mismo son habituales los dolores de cabeza frontales y en algunas
ocasiones
inclinaciones
laterales
de
la
cabeza
compensatorias.
El astigmatismo distorsiona o borra la visin a una cierta distancia, tanto de lejos como de
cerca. La visin es similar al efecto de los espejos deformados, los cuales reproducen
objetos demasiado altos, demasiado anchos o demasiado delgados

PAC Oftalmologia Parte B Libro 4 Errores Refractivos y Ciruga Refractiva


GENERALIDADES PTICAS

50.- Paciente masculino con antecedentes de insuficiencia renal crnica debida a


hipotensin prolongada grave es atendido porque presenta dolor retroesternal. Se le
indic hemodilisis dos veces por semana en los ltimos dos aos y en fechas recientes
ha experimentado episodios de hipotensin al inicio del tratamiento. El dolor se localiza
sobre el msculo trapecio. Se reduce un poco al adoptar la posicin de pie y se exacerba
con la respiracin profunda. De los siguientes trastornos la causa ms probable del dolor
retroesternal que sufre este paciente es:
a)
b)
c)
d)

Pericarditis
Arteriopata coronaria
Espasmo esofgico difuso
Embolias pulmonares

El dolor torcico que sufri este paciente es caracterstico de pericarditis e inflamacin del
pericardio, complicaciones comunes en personas con insuficiencia renal crnica en
hemodilisis. Estos enfermos tambin pueden tener inflamacin de varios recubrimientos
serosos, como peritoneo y pleura; no se conoce el mecanismo de esta complicacin.
Aunque es comn la arteriopata coronaria en pacientes en dilisis, las caractersticas de
dolor en este individuo sugieren que no es el diagnstico. La enfermedad esofgica
tambin es comn en sujetos en dilisis y debe descartarse especficamente como
posible causa. La relacin aparente con la dilisis, as como la frecuencia de los sntomas,
van contra el diagnstico de embolia pulmonar como causa del dolor torcico. Adems, el
dolor musculoesqueltico debido a diversos trastornos se observa en pacientes en dilisis
y puede deberse a anomalas en el metabolismo del calcio y el fsforo, que produce
depsitos de calcio en diversos componentes del sistema musculoesqueltico.
1. Alexander JS. A pericardial effusion of gold paint appearance due to the presence of
cholesterin. Br Med J 1919; 2: 463
2. Brawley RK, Vasko JS, Morrrow AG. Cholesterol pericarditis: consideration of its
pathogenesis and treatment. Am J Med 1966; 41: 235-248.

51.- Acude a consulta externa femenino de 30 aos la cual labora en lavandera, hace
varios aos presenta enrojecimiento de los pliegues proximales de las uas de varios
dedos de las manos, que ocasionalmente le supuran. Eel diagnstico ms probable es:
a)
b)
c)
d)

Dermatitis de contacto.
Liquen plano.
Paroniquia candidisica crnica.
Dermatoficia.

Infeccin por Candida del lecho ungueal que se presenta con mayor frecuencia como una
onicolisis asociada con paroniquia, aunque tambin se observa la destruccin completa
del lecho ungueal y la erosin de la zona distal y lateral de las uas de los dedos, sin
distrofia ungueal total. La perionixis candidisica se trata de la inflamacin con eritema,
edema, dolor y salida ocasional de pus blanquecino escaso y espeso del reborde ungueal
de uno o varios dedos de la mano. Con frecuencia, se acompaa de onicopata
(discoloracin, onicolisis, distrofia total, etc.).
Predisponentes: Todas las labores manuales que generen humedad. (muy frecuente en
amas de casas, trabajadores de restaurantes, lavanderas, etc.)

BIBLIOGRAFA
1. Crespo Erchiga V. Protocolo diagnstico de contaminantes. En "Micologa para
dermatlogos" Ed.
Janssen, Madrid, 1994, pp:49-70.
2. Crespo V, De Lus B, Delgado V, Crespo A y Vera. Espectro etiolgico de las
onicomicosis en nuestro medio. CO7. II Congreso Nacional de Micologa. Santiago de
Compostela. Junio, 1994.
3. Crespo Erchiga V, Delgado Florencio V y Martnez Garca S. Micologa dermatolgica.
Ed. M.R.A. Barcelona, 2006.
4. Daniel III CR. The Diagnosis of Nail Fungal Infection. Editorial Arch Dermatol
1991;127:1566-1567.
5. Delgado Florencio V. Protocolo de identificacin de dermatofitos. En "Micologa para
dermatlogos".
Ed. Janssen, Madrid, 1994, pp:27-41.
6. Delgado Florencio V. Estrategia en el diagnstico y tratamiento de las micosis
superficiales. Ed.
Aula Mdica, Madrid, 1994.

7. Delgado V, Abad Romero-Balmas J, Armijo Moreno M y Dulanto F. Scopulariopsis


brevicaulis como agente de onicomicosis. Actas Deermo-Sif. 1976; 9-10:693-700.
8. English MP. Nails and fungi. Br J Dermatol 1976; 94:697-701.
9. Fevilhade de Chauvin M. Onicomicosis. Dermatologa prctica. 1994; 9:1-2.

52.- A 50-year-old obese female is taking oral hypoglycemic agents. While being treated
for an upper respiratory infection, she develops lethargy and is brought to the emergency
room. On physical exam, there is no focal neurologic finding or neck rigidity. Laboratory
results are as follows:
Na+
134 mEq/L
K+
4.0 mEq/L
HCO3
25 mEq/L
Glucosa
900 mg/dL
BUN
84 mg/dL
Cr
3.0 mg/dL

The most important treatment in this patient is:

a)
b)
c)
d)

Bicarbonate infusion 100 mEq/L


Rapid glucose lowering with intravenous insulin
Large volumes of fluid, insulin; seek concurrent illnesses
30 mEq/hr of KCl

El coma hiperosmolar, ms moderadamente, estado hiperosmolar hipeglucmico


(EHH), o estado hiperglucmico no cettico (EHNC), es una grave complicacin de la
diabetes mellitus, ms comn en los pacientes no insulinodependientes,1-13 cursa con
hiperglucemia mayor de 33 mmol/L, deshidratacin severa y plasma hiperosmolar (mayor
de 320 mosm/L sin cetosis importante, pH arterial a 7,30 y con toma progresiva del
estado de conciencia. Su elevada mortalidad an en nuestros das, entre el 20 y el 40 %
3,10 en relacin con las graves complicaciones a que pueda dar lugar, justifica su
inclusin en el captulo de las emergencias de los diabticos.
Una vez establecido el diagnstico se impone un tratamiento precoz y adecuado, de
preferencia en una unidad de Cuidados Intensivos y que se basa en 3 pilares
fundamentales la administracin de lquidos, la administracin de insulina y la reposicin
de potasio

REFERENCIAS BIBLIOGRFICAS
1. Gonzlez Campoy JM, Robertson RP. Diabetic ketoacidosis and hyperosmolar
nonketotic state: gaining
control over extreme hyperglycemic complications. Postgrad Med 1996;99(6):143-52.
2. Gottschalk ME, Ros SP, Zeller WP. The emergency management of hyperglycemichyperosmolar nonketotic
coma in the pediatric patient. Pediatr Emerg Care 1996;12(1):48-51.
3. Berger W. Diabetic emergencies. Schwiez Rundsch Med Prax 1997;86(8):308-13.
4. Umpierrez GE, Kelly JP, Navarrete JE, et al. Hyperglycemic crises in urban blacks. Arch
Intern Med
1997;157(6):669-75

53.- Se trata de masculino de 40 aos de edad con leucemia mieloblstica aguda recibi
su tercer ciclo de quimioterapia. Una semana despus inicia con fiebre y un sbito
empeoramiento del estado general. Al realizar una exploracin fsica se presenta una
lesin nodular equimtica y dolorosa, con centro ulcerado y rodeada de eritema y edema,
en el MPI. Present menos de 100 leucocitos/mm3, hemoglobina 7 gr/dL y 30.000
plaquetas/mm3. El germen que es aislado con mayor frecuencia en un caso como ste
es:
a)
b)
c)
d)

Pseudomonas aeruginosa.
Aeromonas hydrophila.
Klebsiella pneumoniae.
Staphyloccocus aureus.

Pseudomonas aeruginosa (o Pseudomonas pyocyanea) es una bacteria Gramnegativa, aerbica, con motilidad unipolar.1 Es un patgeno oportunista en humanos y
tambin en plantas.2
Como otras Pseudomonas, P. aeruginosa secreta una variedad de pigmentos como
piocianina (azul verdoso), fluorescena (amarillo verdoso fluorescente) y piorubina (rojo
pardo). King, Ward, & Raney desarrollaron "Pseudomonas Agar P" (tambin conocido
como "medio King A") para mejorar la produccin de piocianina y piorubina; y
"Pseudomonas Agar F" (tambin conocido como "medio King B") para la fluorescena.3

Este patgeno oportunista de individuos immunocomprometidos, P. aeruginosa


infecta el tracto pulmonar, el urinario, tejidos, heridas, y tambin causa otras
infecciones de sangre Pseudomonas puede causar neumonas a grupos,
necesitando a veces ayuda mecnica para superar dichas neumonas, siendo uno
de los ms comunes agentes aislados en muchos estudios. La piocianina es un factor
de virulencia de la bacteria y se ha conocido que puede hasta causar muerte en C.
elegans por estrs oxidativo. Sin embargo, la investigacin indica que el cido saliclico
puede inhibir la produccin de piocianina. Uno en diez hospitales se infectan con
pseudomonas. La fibrosis qustica est tambin predispuesta a la infeccin con P.
aeruginosa de los pulmones. P. aeruginosa es el causante de dermatitis, causada por
disminucin del control de la calidad del agua de bebida. El ms comn causante de altas
fiebres en infecciones es P. aeruginosa. Tambin ha estado involucrado en foliculitis de
tinas de agua caliente, en especial aquellas sin un control higinico contnuo.8
1. Ryan KJ; Ray CG (editors) (2004). Sherris Medical Microbiology, 4th ed. edicin,
McGraw Hill ISBN 0-8385-8529-9..
2. Iglewski BH (1996). Pseudomonas. In: Baron's Medical Microbiology (Barron S et al,
eds.), 4th ed. edicin, Univ of Texas Medical Branch (via NCBI Bookshelf) ISBN 09631172-1-1..
3. King EO, Ward MK, Raney DE (1954). Two simple media for the demonstration of
pyocyanin and fluorescin. J Lab Clin Med. Vol. 44. n. 2. pp. 301-7. PMID 13184240.
4. Todar's Online Textbook of Bacteriology
5. Fine et al, JAMA 1996: 275: 134
6. Diekema DJ et al. Clin Infect Dis 1999;29:595
7. Prithiviraj B, Bais H, Weir T, Suresh B, Najarro E, Dayakar B, Schweizer H, Vivanco
J (2005). Down regulation of virulence factors of Pseudomonas aeruginosa by
salicylic acid attenuates its virulence on Arabidopsis thaliana and Caenorhabditis
elegans. Infect Immun. Vol. 73. n. 9. pp. 5319-28. PMID 16113247.
8. MedlinePlus - Enciclopedia Mdica: Foliculitis de la tina.

54.- Masculino de 20 aos la cual refiere que desde hace varios meses presenta
astenia, cansancio, prdida de apetito y dificultades para concentrarse en los estudios. Al
interrogatorio refiere que a perdido inters en los estudios, frecuenta menos a sus amigos,
con pesimismo en la mayora de sus actividades. El diagnstico ms probable en ste
paciente es:

a) Anorexia nerviosa
b) Trastorno de ansiedad.
c) Distimia.
d) Depresin mayor

LA DISTIMIA
La distimia es un estado de nimo crnicamente deprimido, menos grave que la
depresin y que no cumple los criterios para una depresin mayor, o lo hace slo en
perodos muy cortos. Su evolucin suele ser de ms de dos aos. Se caracteriza por un
abatimiento prolongado del estado de nimo en que el sujeto distmico se describe a s
mismo como triste o desanimado, perdiendo el inters por las cosas y vindose a
menudo como intil y poco interesante. Posee sntomas persistentes o intermitentes, de
intensidad ms leve comparacin a la depresin mayor. Aiskal (1983) la define como mal
humor y se caracteriza porque el individuo est habitualmente triste, introvertido,
melanclico, excesivamente consciente, incapaz de alegra y preocupado por su
insuficiencia personal.
Los criterios de diagnstico de Distimia son los que a continuacin se detallan:
A. Estado de nimo crnicamente depresivo la mayor parte del da de la mayora de los
das, manifestado por el sujeto u observado por los dems, durante al menos 2 aos.
Nota: En los nios y adolescentes el estado de nimo puede ser irritable y la duracin
debe ser de al menos 1 ao.
B. Presencia, mientras est deprimido, de dos (o ms) de los siguientes sntomas:
1. Prdida o aumento de apetito
2. Insomnio o hipersomnia
3. Falta de energa o fatiga
4. Baja autoestima
5. Dificultades para concentrarse o para tomar decisiones
6. Sentimientos de desesperanza
C. Durante el perodo de 2 aos (1 ao en nios y adolescentes) de la alteracin, el sujeto
no ha estado sin sntomas de los Criterios A y B durante ms de 2 meses seguidos.
D. No ha habido ningn episodio depresivo mayor durante los primeros 2 aos de la
alteracin (1 ao para nios y adolescentes); por ejemplo, la alteracin no se explica
mejor por la presencia de un trastorno depresivo mayor crnico o un trastorno depresivo
mayor, en remisin parcial.
Nota: Antes de la aparicin del episodio distmico pudo haber un episodio depresivo
mayor previo que ha remitido totalmente (ningn signo o sntoma significativos durante 2
meses). Adems, tras los primeros 2 aos (1 ao en nios y adolescentes) de trastorno
distmico, puede haber episodios de trastorno depresivo mayor superpuestos, en cuyo
caso cabe realizar ambos diagnsticos si se cumplen los criterios para un episodio
depresivo mayor.

E. Nunca ha habido un episodio manaco, un episodio mixto o un episodio hipomanaco y


nunca se han cumplido los criterios para el trastorno ciclotmco.
F. La alteracin no aparece exclusivamente en el transcurso de un trastorno psictico
crnico, como son la esquizofrenia o el trastorno delirante.
G. Los sntomas no son debidos a los efectos fisiolgicos directos de una sustancia (p. ej.,
una droga, un medicamento) o a enfermedad mdica (p. ej., hipotiroidismo).
H. Los sntomas causan un malestar clnicamente significativo o deterioro social, laboral o
de otras reas importantes de la actividad del individuo.
Como vemos, la Distimia presenta sntomas ms o menos similares a la Depresin mayor,
pero se diferencian entre s respecto a:
Tipo de evolucin: el estado de nimo depresivo es crnico (no presenta intervalos libres
de sntomas o mejoras significativas) y dura por lo menos 2 aos.
Severidad de los sntomas: los mismos suelen ser leves o moderados, sin una
alteracin significativa de las relaciones familiares, sociales y laborales del individuo.

Referencias Bibliogrficas
- Diagnostic and Statistical Manual of Mental Disorders, 4 edition (DSM-IV TR). American
Psychiatric
Association.
American
Psychiatric
Press,
2000.
- Kaplan and Sadock's Synopsis of Psychiatry, 9 edition. Lippincott Williams & Wilkins
Press, 2003.

55.- Masculino de 43 aos refiere prdida de fuerza progresiva en miembros inferiores,


de unos 7 das de evolucin, dolores musculares y parestesias en pies y manos, como
nico antecedente refiere haber cursado con cuadro de faringoamiglatistis hace un par de
semanas con mejora al tratamiento. En su exploracin se aprecia nicamente debilidad
en los cuatro miembros, de predominio distal y en miembros inferiores, y arreflexia
generalizada. El diagnstico ms probable de ste paciente es :

a) Una miastenia gravis.


b) Un proceso expansivo medular cervical.
c) Una polimiositis.
d) Una polirradiculoneuritis aguda.

SINDROME DE GUILLAIN-BARR
El sndrome de Guillain-Barr fue descripto en el ao 1834 por Ollivier y Wardrop, Landry,
en 1859 hablo de una paralisis ascendente que segua con insuficiencia respiratoria y
muerte. Osler, en 1892 hablo de una polineuritis febril y Guillain-Barr y Strohl , en 1916
reconocieron una polineuritis benigna con disociacin albuminocitolgica.
Tambien llamada polirradiculoneuritis aguda, caracterizada por alteraciones
sensitivas, motoras, arreflexia y disociacin albmino citolgica en el LCR (lquido
cefaloraquideo).
Es una enfermedad que puede aparecer a cualquier edad afectando por igual a ambos
sexos.
La caracterstica neuropatolgica corresponde a focos de desmielinizacin focal y
segmentaria de los nervios perifricos en toda su longitud, comprometiendo incluso las
raices. Los nervios del sistema nervioso perifrico presenta infiltrados inflamatorios
perivasculares, y es ah donde se observan las lesiones mielnicas.
Contina en boga la teora inmunolgica como la responsable de estas
polirradiculoneuritis inflamatoria. Muchas veces la enfermedad surge luego de un proceso
de vacunacin, aplicacin de suero o de una infeccin vrica ( generalmente trancurre de
7 a 25 das entre el proceso que se cree causal y el comienzo de los sntomas), Los
organismos que se han visto con ms frecuencia relacionados con este transtorno fueron
el virus de Epstein-Barr, el citomegalovirus, el micoplasma y el virus de la hepatitis B.
La enfermedad se presenta, en su evolucin, en 3 (tres) fases
1- FASE DE EXTENSIN
2- FASE DE ESTADO
3- FASE DE RECUPERACIN
1- FASE DE EXTENSIN
El paciente comienza a sentir sensacin de "hormigueo, las piernas se me duermen",
"siento calambres en ambas piernas"... las parestesias en las extremidades y el deficit
motor son comunes, pero ms frecuente es que aparezca el compromiso motor. Se
afectan los miembros inferiores precozmente y la extensin suele ser ascendente, de
forma progresiva, en un tiempo estimado de 1 semana, 1 semana y media. Hay que
tener mucho cuidado con estos pacientes por la posible aparicin del compromiso de los
msculos respiratorios o algun trastorno en la deglucin (En estos casos son pacientes
pasibles de internacin y de cuidados intensivos).
2- FASE DE ESTADO
Es donde aparecen la mayora de los signos neurolgicos. El compromiso motor puede
llegar al extremo, puede aparecer una tetrapleja con compromiso respiratorio y
deglutorio. Otros casos pueden limitarse al compromiso de los miembros inferiores.
Aparece arreflexia generalizada. Suceden parestesias, se compromete la sensibilidad
tactil y profunda (por compromiso de los cordones posteriores, va de Goll y Burdach).
Toma nervios craneales, aparece una parlisis facial que puede ser uni o bilateral, puede
verse parestesia trigeminal. Cuando toma los pares craneales IX-X (glosofaringeo y
neumogstrico) aparecen los trastornos de la deglucin.
Cuando toma el sistema nervioso vegetativo se ve que el paciente cursa con hipertensin,
trastornos en la repolarizacin cardaca (visibles en el ECG), alteraciones en la glucemia,

hiponatremia, SIHAD (secrecin inapropiada de hormona antidiurtica) o bradicardias


(que puede llevar hasta un paro cardaco)
Un tema muy importante es el estudio del lquido cefalorraqudeo (LCR) que
muestra una elevacin de la proteinorraquia que no se acompaa de hipercitosis
(aumento de las clulas en el LCR), esto es lo que se denomina disociacin
albuminocitolgica. En el 80% de los casos la elevacin corresponde a las
gammaglobulinas. Esta disociacin no es inmediata, por lo que puede tardar un
tiempo en aparecer. (esto tiene que ver con que las punciones del LCR en un
perodo temprano pueden dar normales).
3- FASE DE RECUPERACIN
Los signos neurolgicos regresan en orden inverso al que aparecieron luego de 1 a 2
semanas de estabilidad en los sntomas. Estos cambios lo hacen en forma ms lenta en
relacin a como aparecieron. El tiempo de regresin puede durar desde semanas hasta
meses.
La recuperacin puede ser total o dejar secuelas motoras o sensitivas hasta en un
20% de los casos.
DIAGNOSTICO DIFERENCIAL
Esta enfermedad es una de las polineuropatias ms frecuentes, la que ms rpido
evoluciona y que tiene un alto potencial mortal.
Cualquier polineuropata, de evolucin ascendente, que ponga en peligro de vida al
paciente por compromiso respiratorio en el plazo de unos pocos das, nos deber
hacer sospechar de un sindrome de Guillain-Barr.
Pearce, j.m.s. Barr-Liou syndrome. J Neurol Neurosurg Psychiatry 2004: 75, 319.
-Pritchard, J.; Hughes, R.A.C. Guillain-Barr syndrome. The Lancet 2004; 363: 2186-88.
-Thibaut, F.J.A. Barr (1880-1967). J Neurol Sci 1968; 6: 381-2.

56.- Masculino de 56 aos con diagnstico probable de sinusitis aguda, la proyeccin


radiolgica que mejor valora los senos maxilares y las estructuras intranasales, en sta
patologa es:

a)
b)
c)
d)

Lateral.
Waters.
Submentoniana.
Anteroposterior.

La proyeccin Waters es la proyeccin que mejor permite valorar la neumatizacin,


opacificacin o engrosamiento de mucosa de los senos maxilares as como las
estructuras intranasales.

Proyeccin de Waters u occipito-mentoniana para senos maxilares


(Radiografa normal).
Gonzlez-Saldaa N, Infectologa Clnica Peditrica, 7 edicin, pginas 63-98.

57.- Masculino de 54 aos de edad, que se queja de debilidad muscular desde hace
aproximados 3 meses. A la E.F. Se aprecia eritema en heliotropo en su trax superior,
cuello y cara. Eritema maculopapular situado sobre nudillos de ambas manos.
De los siguientes paraclnicos el que presenta mayor sensibilidad para diagnstico en
sta enfermedad es:

a)
b)
c)
d)

Potasio srico.
Sodio srico
Factor reumatoide
CPK

Son un grupo heterogneo de enfermedades que se caracterizan por debilidad muscular


secundaria a la inflamacin del msculo estriado.
EPIDEMIOLOGA

INCIDENCIA: 5 casos nuevos por milln / ao


PREVALENCIA: 10 - 60 casos por milln de habitantes.

Frecuencia baja.

2 PICOS DE EDAD: 5-15 aos


40-60 aos
FRECUENCIA MUJER : HOMBRE
2:1
RAZA NEGRA > ORIENTALES (4:1)

La polimiositis respeta la piel, mientras que la dermatomiositis presentar alteraciones


cutneas caractersticas acompaando a la afectacin muscular
Cuadro Clnico:
- Alteraciones musculares. Vienen marcadas por la presencia de debilidad muscular
aguda o subaguda (generalmente de inicio insidioso), simtrica y difusa, con preferencia
por musculatura proximal de extremidades (cintura plvica y escapular), tronco y cuello.
En la mayora de los casos es indoloro. Con el tiempo, desarrollan atrofia, contracturas y
disminucin de los reflejos.
- Alteraciones cutneas. La ms frecuente en la DM es una erupcin cutnea
eritematoviolcea que afecta a cuello, cara y trax. Es caracterstico tambin, el eritema
heliotropo (en prpados), que puede extenderse a otras zonas fotoexpuestas), las
ppulas de Gottron (localizadas en los nudillos), telangiectasias periungueales, a veces
ulceracin drmica y calcinosis (fundamentalmente en la DM infantil).
- Articulares. Artralgias, artritis transitorias, no erosivas, con tendencia a la simetra.
- Otras. Afectacin cardiaca variable (alteracin ECG, arritmia, miocarditis), pulmonar
(fibrosis intersticial asociada con anti Jo-1), renal (muy rara), fenmeno de Raynaud
Diagnstico:
- Analtica: aumento de VSG y de enzimas musculares, (CPK, aldolasa, GOT, GPT,
LDH). La CPK es la ms sensible y la que guarda una mejor correlacin clnica con la
actividad de la enfermedad y la valoracin de recadas. El FR es + en 20% y
ANA es + en 10-30%. Si la destruccin muscular es intensa, puede producir mioglobinuria.
- Destacan anticuerpos: anti-Jo1: en casos de PM asociado a neumonitis intersticial
(sndrome antisintetasa-miosistis, fibrosis pulmonar, artritis no erosiva y fenmeno de
Raynaud).
anti-PM1 o PM-Scl: asociacin con esclerodermia.
anti-Mi, en DM.
antimioglobina.

BIBLIOGRAFA RECOMENDADA
Klippel JH, Stone JH, Crofford LJ, White PH, editors. Primer on the rheumatic
diseases. 13th ed. New York: Springer-The Arthritis Foundation; 2008.
Martnez-Elizondo P, editor. Introduccin a la Reumatologa. 4a ed. Mxico: Colegio
Mexicano de Reumatologa A.C./Intersistemas S.A. de C.V.; 2008.
Firestein GS, Budd RC, Harris ED Jr, McInnes IB, Ruddy S, Sergent JS, editors. Kelleys
Textbook of Rheumatology. 8th ed. Philadelphia: Saunders Elsevier; 2009.

58.- Estamos ante una epidemia cuando:

a) La tasa de ataque de la enfermedad sea mayor de 10 por 1000 habitantes


b) Se observa un aumento en la frecuencia esperada de cualquier dao a la
salud en el ser humano, durante un tiempo y un espacio determinados
c) Una enfermedad tiene una tasa baja de aparicin pero se encuentra de manera
constante en una comunidad o regin
d) Los casos de una enfermedad aumentan en una determinada estacin del ao.

Epidemia, al aumento en la frecuencia esperada de cualquier dao a la salud en el


ser humano, durante un tiempo y un espacio determinados. En algunos
padecimientos la ocurrencia de un solo caso se considera epidemia.
Norma Oficial Mexicana NOM-017-SSA2 -1994, Para la vigilancia
epidemiolgica. Apartado 3.1.24

59.- Se trata de paciente masculino de 57 aos de edad presenta una historia de 2 das
de hemoptisis. Reporta un inicio agudo de 8 episodios de tos con sangre fresca (aprox
una cucharadita de sangre por cada episodio). No reporta otros sntomas, excepto por
una tos productiva de 5-10ml de esputo cada maana. Tiene una historia de EPOC, para
lo cual toma broncodilatadores. Ha fumado 30 cigarrillos diariamente por los ltimos 30
aos. El examen fsico es normal, y una RX limpia. La causa ms probable de la
hemoptisis en ste paciente es:

a)
b)
c)
d)

Bronquiectasias
Carcinoma broncognico
Bronquitis crnica
Tb pulmonar

La EPOC puede presentar episodios de hemoptisis con placa radiogrfica normal, aunque
no es muy frecuente; las otras causas como TB pulmonar y Cncer pulmonar as como
las bronquiectasias presentan lesiones en la radiografa que hace sospechar en estos
diagnsticos y en el presente caso no se mencionan lesiones en la Rx, la deficiencia de

alfa 1 provoca la presencia de enfisema pulmonar la cual igualmente debera tener una
placa con alteraciones.
1.- Fishman AP, , Fishman JA, Grippi MA, Kaisser LR, Seor RM. Pulmonary Diseases
and disorder. 3a. Edicin McGraw-Hill, EUA, 2006.
2.- Fraser, R ; Neil, C; Par, P; Diseases of the Chest, Third Edition, Editorial Elsevier,
2005.
3.- Murray and Nadels; Textbook Respiratory Medicine, Vol 1-2, Elsevier editorial, 2005.

60.- Femenino de 52 aos con antecedente: Dx. De sndrome anmico sin causa
aparente. Exmenes de Laboratorio Reportan: Hb de 8,5 g/dl, VCM de 75 fl, y HCM de 25
pg. El examen de laboratorio que conforma el origen del diagnstico de esta paciente es
la determinacin de:
a) Sideremia.
b) Saturacin de la transferrina.
c) Hemoglobina A2.
d) Ferritina.
La anemia por dficit de hierro es un proceso que aparece de forma rpida o de
forma ms bien lenta?
La anemia por falta de hierro se va estableciendo de forma gradual, cuando el hierro que
se pierde supera al hierro aportado por la dieta; lo primero que ocurre es que el organismo
empieza a utilizar las reservas de hierro. En esta etapa la ferritina empieza a descender,
pero el hierro srico y la TIBC generalmente permanecen inalterados no establecindose
todava anemia. En el momento en que el dficit de hierro empeora, el hierro srico
disminuye mientras que la transferrina y la TIBC aumentan, empezando las clulas de la
serie roja (hemates) a palidecer (hipocromia) y a disminuir de volumen (microcitosis),
existiendo todava un nmero suficiente de clulas de la serie roja. Si la deficiencia de
hierro contina o empeora aparece entonces la anemia.
Mourey L. Manual de procedimientos del Laboratorio Clnico. Mxico:
IMSS, 1978: 158.

61.- Se trata de femenino de 19 aos con menarca a los 12 aos ritmo menstrual 45x4.
Refiere vida sexual activa desde los 17 aos con frecuencia de 4 veces por semana. A la
exploracin se encuentra acn intenso en la frente, mejillas y mentn. Acude a consulta
por que desea adoptar un tratamiento anticonceptivo por va oral, lo mas adecuado es:
a) Norgestimato
b) Ciproterona
c) Gestodeno
d) Levonorgestrel
El efecto antiandrognico especfico del acetato de ciproterona acta por inhibicin
competitiva de la unin de la 5 - alfa - dihidrotestosterona con el receptor citoslico de las
clulas blanco, que disminuye la produccin y la excrecin de sebo y el aumento y el
desarrollo del vello.
Es un derivado de la 17 - alfa - hidroxiprogesterona que posee accin progestgena. Su
accin antigonadotrfica se suma a la del etinilestradiol. El acetato de ciproterona no
posee accin estrognica sino un efecto antiestrognico, y tampoco posee accin nociva
sobre la funcin de la corteza suprarrenal;

Indicaciones en la mujer: Manifestaciones de androgenizacin de grado severo, por


ejemplo, hirsutismo grave, alopecia androgentica de tipo grave, a menudo acompaados
por manifestaciones graves de acn y/o seborrea.
Indicaciones en el hombre: Atenuacin del impulso en las desviaciones sexuales.
Tratamiento antiandrgeno del carcinoma de prstata inoperable.
BIBLIOGRAFA
1. Swift S. Current opinion on the classification and definition of genital tract prolapse. Curr
Opin Obstet
Gynecol 2002; 14: 503-7.
2. De Caro R, Aragona F, Herms A, Guidolin D, Bizzi E, Pagano F. Morphometric analysis
of the fibroadipose tissue of the female pelvis. J Urol 1998; 160: 707-13.
3. Gill E, Hurt W. Pathophysiology of pelvic organ prolapse. Clin Obstet Gynecol 1998;
25(4): 757-69.
4. DeLancey, J. Anatomic aspects of vaginal eversion after Hysterectomy. Am J Obstet
Gynecol. 1992;
166(6 pt 1): 1717-24.

62.- Se trata de femenino de 24 aos de edad gesta 1, tuvo un parto vaginal espontneo
con un producto con peso de 4,350 g. despus de 5 minutos de traccin suave del
cordn umbilical se expuls la placenta, que parece estar intacta. Se inici el masaje del
fondo uterino y se pidi a la enfermera que administrara 20 unidades de oxitocina en 100
ml de solucin Ringer lactato. Despus de una inspeccin cuidadosa del canal del parto
se observa una laceracin de segundo grado y una laceracin de 2 cm en la pared vaginal
izquierda que se intent reparar. En la exploracin fsica se encuentra un fonso uterino

blando y atnico. Los signos vitales son: temperatura 37.1C, TA 164/92, FC 130x, FR 18
X. El tratamiento de eleccin en sta paciente es:

(a) Oxitocina 10 unidades directas en goteo intravenoso


(b) Metilergonovina 0.2 mg IM
(c) Prostaglandina F 0.25 mg IM
(d) Legrado

Morgan M, Siddighi S. Ginecologa y obstetricia, National Medical Series. 5 edicin. Mc


Graw Hill. Pp. 28. La atona uterina es la causa ms comn de hemorragia puerperal. El
masaje energtico y la oxitocina diluida no han sido tiles para interrumpir la hemorragia y
por tanto el siguiente paso es agregar un frmaco uterotnico. La metilergonovina est
contraindicada porque la paciente se encuentra hipertensa a pesar de la hemorragia
intensa, el siguiente frmaco es la prostaglandina. La administracin de oxitocina no
diluida, 10 UI por va IV podra causar hipotensin grave. La exploracin manual podra
ser apropiada si se sospecha laceracin como causa de hemorragia. El legrado es
apropiado para la hemorragia puerperal tarda, cuando se sospecha retencin de los
productos de la concepcin.

63.- Se Tarata de femenino de 12 aos que es sometida a una prueba cutnea de


Mantoux como parte de una exploracin fsica de control. No ha sufrido exposiciones
conocidas a tuberculosis ni tiene factores de riesgo. A las 48 horas aparece una zona de
eritema de 20mm y un rea de induracin de 7mm Cmo pueden interpretarse estos
hallazgos?
a)
b)
c)
d)

Es un resultado positivo de exposicin a tuberculosis.


Es un resultado negativo de exposicin a tuberculosis.
Conviene hacer una radiografa de trax.
Deben obtenerse lavados gstricos de Mycobacterium tuberculosis.

Para determinar si el hallazgo de una prueba cutnea de Mantoux es positiva o no, se usa
la zona de induracin, y no la de eritema. En ausencia de factores de riesgo conocidos o
de exposicin una zona de induracin menor a 10 mm, es un resultado de Mantoux
negativo. No estn indicados ni radiografa de trax ni lavado gstrico.

Bibliografa:
1. Grupo de trabajo de Tuberculosis de la Sociedad Espaola de Infectologa Peditrica.
Interpretacin de la prueba de tuberculina.An Pediatr (Barc) 2003;59(6):582-5
2. Lpez-Hermosa, P. Papel actual del Mantoux en la poblacin Infantil. Rev.Ped.
Atencin Primaria.Vol I. N5, Jul/Sep 1999.
3. Alcaide Megas J et al. Epidemiologia de la tuberculosis. An Esp Pediatr 2000;53: 449457
4. Alonso Moreno FJ et al. Prevalencia de la infeccin tuberculosa en las personas
inmigrantes del rea de salud de Toledo. Rev Esp Salud pblica 2004; 78: 593-600.
Rodrguez Valvn E. Situacin actual de la tuberculosis en Espaa: incidencia y mortalidad
desde 1995.Caractersticas de los casos de tuberculosis y meningitis tuberculosa
declarados en 2000. SEMERGEN 2002; 28(7):395-400.

64.- Masculino de 3 aos es llevado a consulta por preentar hiporexia. Antecedentes:


Originario de zona rural del estado de Guerrero, geofagia positiva, dolor abdominal, clico
desde hace varios meses. Las evacuaciones en los ltimos 5 das son semilquidas
acompaadas de moco y pujo no sangre. E.F.: mala higiene personal, desnutrido, abdomen
globoso blando y dolor a la presin en colon descendente, peristalsis aumentada. La
complicacin mas frecuente que se puede presentar en este paciente es:

a) Apendicitis.
b) Obstruccion intestinal.
c) Prolapso rectal, trichurosis
d) Perforacin intestinal.
Introduccin.
La trichuriosis es una geohelmintiasis frecuente en zonas tropicales, rurales. Se estima que
se encuentran infectadas unos 100 millones de personas en Latinoamrica y Caribe (Hotez
PJ, et al., 2008). Predomina en nios en edad escolar, en quienes se asocia a colitis
crnica y sndrome disentrico, retardo en el crecimiento y disminucin de peso; la
deficiencia en las funciones cognitivas y alteraciones conductuales se han relacionado con
anemia ferropriva, altas cargas parasitarias y desnutricin. Los casos de la parasitosis en
adultos que viven en zonas endmicas han aumentado, pero no se reportan usualmente.
(Khuroo M, et al. 2010).
La ascariosis y la trichuriosis son las infecciones por geohelmintos ms frecuentes en
Mxico.
NTDs en LAC: Prevalencia y distribucin. Hotez PJ, et al, 2008.
LAC: Latin American and Caribbean Region

NTDs. Neglected Tropical Diseases.


Las lesiones intestinales y el cuadro clinico varan en relacin directa al nmero de
parsitos y factores dependientes del hospedero (edad, estado nutricional, infecciones
concomitantes).
En infecciones leves y moderadas el dao, apenas apreciable, consiste en compresin
mecnica de las clulas de la mucosa colnica y se asocia a dolor abdominal de tipo clico
y episodios diarreicos.

En infecciones masivas la mucosa intestinal se encuentra edematosa y friable, con


sangrado fcil; es caracterstica la degeneracin y necrosis de las clulas cercanas a la
cabeza del parsito, con pequeas hemorragias subepiteliales e inflamacin con infiltracin
difusa de linfocitos y eosinfilos.

Induce, al igual que los otros geohelmintos, una respuesta de tipo Th2 y respuesta
reguladora Th2/Treg (Jackson JA, et al. 2009).

Las manifestaciones clnicas varan de acuerdo a la masividad de la infeccin y la


presencia de otros parsitos (poliparasitismo) e incluyen dolor abdominal, cefalea,
hiporexia, prdida de peso, diarrea crnica, disentera, pujo, tenesmo, prolapso rectal y
signos y sntomas relacionados con anemia hipocrmica microctica; cada tricocfalo
expolia alrededor de 0.005 ml de sangre/da y restos tisulares. Adems, la irritacin
constante de las terminaciones nerviosas intramurales redunda en hiperperistaltismo.
Complicaciones.

- Poliparasitismo
- Prolapso rectal
- Anemia
- Apendicitis
- Infeccin bacteriana 2aria
- Retraso pondoestatural y dficit cognitivo en escolares.
TRICHURIOSIS

Dra. Teresa Uribarren Berrueta


Departamento de Microbiologa y Parasitologa, Facultad de Medicina, UNAM October,
2010
Vnculos.
- Mohammad S. Khuroo, Mehnaaz S. Khuroo, and Naira S. Khuroo. Trichuris dysentery

syndrome: a common cause of chronic iron deficiency anemia in adults in an endemic area
(with videos). Gastrointestinal Endoscopy, Jan 2010; 71(1):200-204.
doi:10.1016/j.gie.2009.08.002
- Geary TG, Woo K, McCarthy JS, Mackenzie CD, Horton J, Prichard RK, de Silva NR, (...),
Bundy DA. Unresolved issues in anthelmintic pharmacology for helminthiases of humans.
Int J Parasitol 2010;40(1):1-13. doi:10.1016/j.ijpara.2009.11.001 Geohelmintos y otros
nematodos.
- Kyung-Sun Ok, et al. Trichuris trichiura Infection Diagnosed by Colonoscopy: Case
Reports and Review of Literature. Korean J Parasitol. Sept 2009;47(3):275-280
DOI: 10.3347/kjp.2009.47.3.275
- Hu Y, Xiao S-H, Aroian RV. The new anthelmintic tribendimidine is an L-type (Levamisole
and Pyrantel) nicotinic acetylcholine receptor agonist. PLoS Neglected Tropical Diseases
2009;3(8), art. no. e499.
- Jackson JA, Friberg IM, Little S, Bradley JE. Review series on helminths, immune
modulation and the hygiene hypothesis: Immunity against helminths and immunological
phenomena in modern human populations: Coevolutionary legacies? Immunology 2009;126
(1):18-27. doi:10.1111/j.1365-2567.2008.03010.x

65.- Femenino de 40 aos de edad acude al servicio de urgencias quejndose de


calambres en las piernas y parestesias en los dedos de las manos. Un ao antes se le
someti a una operacin del cuello, pero no est segura qu fue lo que se le hizo. El signo
de Chevostek es positivo: la percusin sobre el nervio facial por delante de la oreja
desencadena una contraccin espasmdica del labio superior. El trastorno ms probable
que sugiere estas alteraciones es:
a) Hipercalciemia
b) Acidosis
c) Hiperpotasemia
d) Hipocalciemia

SINTOMATOLOGA
Neuromuscular: la hipocalcemia aguda se manifiesta por parestesia (hormigueo y
adormecimiento de los dedos y regin peribucal) y calambres o contracturas musculares.
La sintomatologa subclnica de tetania, evidenciada por el signo de Chvostek, (ocurrencia
de espasmo facial, especialmente del orbicular de los labios, al percutir el nervio facial a
mitad de distancia entre la comisura labial y el odo). El signo de Trousseau es un
espasmo doloroso del carpo, que se presenta luego de mantener por tres minutos una
presin >20 mmHg por encima de la sistlica, siendo un signo ms de tetania.
La hipocalcemia crnica se presenta con irritabilidad, confusin, demencia e incluso, en
infantes, como retardo mental. Tambin se reportan movimientos coreicos, distonas y
convulsiones. Se ha reportado calcificacin de ganglios basales en la radiografa de
crneo, que no es reversible al tratamiento.

Cardiovascular: prolongacin de la fase de potencial de accin y por lo tanto


prolongacin del segmento ST en el ECG. En casos de severa deficiencia se presentan
arritmias, hipotensin o falla cardiaca; la hipocalcemia aumenta la cardiotoxicidad de los
digitlicos.
Pulmonar: broncoespasmos y laringoespasmos vistos, sin embargo, con poca frecuencia.

Dermatolgica: piel seca, uas quebradizas y cada del cabello como signos no
especficos en la hipocalcemia crnica

LECTURAS RECOMENDADAS
1. Carlstedt F, Lind L. Hypocalcemic syndromes. Crit Care Clin 2001; 17:139-153.
2. Gibbs M, Wolfson A, Tayal V. Electrolyte disturbances. En: Rosens Emergency
Medicine. Concepts and Critical Practice. J Marx, Hockberg
R, Walls R, et al (eds). Fith edition. Mosby. St Louis,1998.
3. Kapoor M, Chan GZ. Fluid and electrolyte abnormalities. Crit Care Clin 2001; 17:503529.
4. Lind L, Carlstedt F, Rastad J, et al. Hypocalcemia and parathyroid hormone secretion in
critically ill patients. Crit Care Med 2000; 28:93-99.
5. Lo CY. Postthyroidectomy hypocalcemia. J Am Coll Surg 2003; 196:497-498.
6. Marx SJ. Hyperparathyroid and hypoparathyroid disorders.N Engl J Med 2000;
343:1863-1875.

66.- Un varn de 73 aos acude a una cena familiar de celebracin, durante su estancia
ingiere bebidas alcohlicas. Al dejar el restaurante sufre un colapso. Cul de los
siguientes trastornos es ms probable?
a) Convulsiones
b) Arritmia cardaca
C) Quiste coloide del tercer ventrculo
d) Sncope posprandial
Allen R. M. MMS Medicina Interna. 5. Edicin. National Medical Series. Mc. Graw Hill.
2006. (captulo 11IIA 3 a (2), b (2), d (1) (8)). El sncope posprandial es causa comn de
desvanecimiento en ancianos. El consumo de alcohol tambin puede producir sncope y a
menudo es un factor contribuyente. Puede presentarse sncope posprandial cuando la
sangre es desviada al lecho mesentrico, lo que produce disminucin relativa del riego
cerebral. El paciente no muestra datos de actividad convulsiva; por tanto, es poco
probable que se trate de una convulsin. Las arritmias cardacas y la hipersensibilidad del
seno carotdeo pueden causar sncope y deben considerarse siempre en ancianos. Sin
embargo, el medio en que este paciente se desvaneci indica que es ms probable un
sncope posprandial. Un quiste coloide del tercer ventrculo es causa rara de prdida
sbita de la conciencia. Los quistes pequeos actan como vlvulas de bola y obstruyen
el flujo por el agujero de Monrow, lo que causa hidrocefalia aguda.

67.- Masculino de 66 aos con antecedentes de constipacin y dolor abdominal en


hipogastrio y fosa ileacaa izquierda de manera recurrente. El enema baritado muestra
mltiples divertculos de pequeo calibre en el sigmoides. El tratamiento ms apropiado
para ste paciente es:

a)
b)
c)
d)

Colonoscopa con biopsia


Ranitidina
Enema con esteroides
Dieta alta en residuos

Tratamiento
El tratamiento de la diverticulosis intenta reducir el espasmo segmentario. Una dieta rica
en residuos es de utilidad y puede suplementarse con preparados de semillas. Las dietas
bajas en residuos estn contraindicadas. Tericamente deberan ser de ayuda los
antiespasmdicos (p. ej., la belladona); su valor en la prctica es difcil de enjuiciar. Su
utilizacin crnica, especialmente en los ancianos, suele causar efectos secundarios
adversos. No est justificada la ciruga en caso de enfermedad diverticular sin
complicaciones. La reseccin con anastomosis del rea del intestino afectada en la colitis
espstica (una combinacin de divertculos, espasmo y diarrea) puede llevar a resultados
equvocos.
Deben cateterizarse las arterias mesentricas superior e inferior. La inyeccin selectiva de
vasopresina controla la hemorragia en el 70% de los Pacientes. En algunos casos la
hemorragia recurre a los pocos das y es necesaria la ciruga. Es posible practicar la
reseccin segmentaria si se conoce el punto sangrante; en alrededor del 75% de los
Pacientes este punto se encuentra en una zona proximal respecto a la flexura esplnica,
aun cuando los divertculos predominen en el lado izquierdo. Si no se puede identificar el
punto sangrante est indicada una colectoma subtotal.
Los divertculos gigantes deben ser quirrgicos. Estas lesiones pueden observarse en las
radiografas abdominales simples o demostrarse mediante enema de bario. Dado que la
posibilidad de infeccin o perforacin es alta, se prefiere la reseccin del rea afectada
del colon.
Blibliografa
Shakelfords. Surgery of the alimentary tract. 5a. Ed. 2002. Tomo 3.
Feldmans. Gastroeneterology. 2002.
Perez. Anatoma y fisiologa del hgado. Univ. Catlica de Chile. 2005.
Bratiz. Serum laboratory test in cirrhosis. Journal of Hepatology. Slovakia. 2005.
Paradis. Glycomics. Journal of hepatology. Ireland. Agosto 2005.

68.- Femenino de 40 aos de edad refiriere disfagia restroesternal baja y de intensidad


variable, desde hace unos aproximados 4 aos. Ocasionalmente presenta episodios de
dolor retroesternal de carcter opresivo que en los ltimos aos ha disminuido en
intensidad y frecuencia, al tiempo que se intensificaba la disfagia. Refiere que desde hace
varios meses, viene presentando con el decbito regurgitaciones no cidas ni amargas.
Hace 2 meses present una neumona por aspiracin. Ha presentado prdida de
aproximados 5 kilogramos desde el comienzo del cuadro. De entre los siguientes, De los
siguientes el diagnstico ms probable es:

a) Cncer de esfago.
b) Acalasia esofgica.
c) Estenosis esofgica pptica.
d) Hernia hiatal con reflujo gastroesofgico.

La acalasia consiste en la incapacidad para relajar las fibras de msculo liso del aparato
gastrointestinal en cualquier sitio de unin de una parte con otra. Dcese, en especial, de
la acalasia esofgica, o la incapacidad del esfnter gastroesofgico para relajarse al
deglutir, por degeneracin de las clulas ganglionares en la pared del rgano.1 El esfago
torcico tambin pierde la actividad peristltica normal y se vuelve dilatada produciendo
un megaesfago.
La acalasia esofgica o simplemente acalasia es una rara enfermedad en la cual el
esfago se encuentra inhabilitado para llevar el alimento hacia el estmago. La
enfermedad afecta ambos sexos y puede aparecer a cualquier edad, sin embargo se
diagnostica generalmente entre la tercera y la cuarta dcada de la vida.
Los sntomas ms notables son: 1. Dolor retroesternal, que en fases iniciales es
intermitente y que se va haciendo progresivo. 2. Disfagia esofgica (el alimento, una vez
tragado, se "atasca" por el aumento de presin de la parte distal del esfago y el cardias).
3. En fases avanzadas, se puede dar regurgitacin, dolor torcico y prdida de peso que
puede confundir con un cncer de esfago. Disfagia que se inicia de forma brusca,
generalmente a los lquidos, casi siempre relacionada con grandes emociones, de
evolucin caprichosa, con perodos de deglucin normal y que se acompaa en ocasiones
de dolor retroesternal por espasmos, y regurgitaciones de lquido claro e inspido, la cual
puede ser en la noche y provocar sntomas respiratorios. La presencia de sialorrea llama
la atencin, as como la halitosis, casi siempre presente.
En la acalasia tipo I el dolor es raro y el paciente regurgita por rebosamiento, casi siempre
cuando est dormido. En la acalasia tipo II predomina el dolor al deglutir o de forma
espontnea y la regurgitacin es inmediata.

Gisbert
J.P.,
Losa
C.,
Barreiro
A.,
Pajares
J.M.
Servicio de Aparato Digestivo, Hospital de la Princesa, Universidad Autonoma de Madrid.
Rev Clin Esp 2000 Aug; 200(8):424-31

69.- Femenino de 37 aos que est siendo evaluada por probable esclerosis mltiple.
Empez con parestesias intermitentes en ambas piernas hace varios meses y ahora tiene
incontinencia urinaria. De los siguientes, el hallazgo fsico ms comn en esclerosis
mltiple es:

a)
b)
c)
d)

Prdidad de memoria
Constipacin
Oftalmoplegia internuclear
Temblor

Manifestaciones clnicas
Puede ser asintomtica y descubrirse como hallazgo en RMN
Primer ataque no tiene premonicin y puede ser monosintomtico o
polisintomtico
20% inicio agudo con deficit mximo en minutos u horas
Remisin frecuente del cuadro despues del primer ataque
Las recurrencias reprsentan recrudenscencias de lesiones ms tempranas o los
efectos de otras nuevas
Signos y sintomas:
Debilidad o insensibilidad de una extremidad o facial
Perdida de la visin monocular
Oftalmoparesia pleja internuclear
Diplopia atrofia ptica
Vrtigo , ataxia y nistagmo
Disartraria
Incontinencia urinaria
Convulsiones en 3 a 4 %
Cambios psiquicos

Referencias bibliogrficas
1. Noseworthy JH, Lucchinetti C, Rodrguez M, Weinshenker BJ. Multiple sclerosis.
N. England J Med. 2000;343:938-52. Disponible en:.
2. Omari KM, John GR, Sealfon SC, Raine CS. CXC chemokine receptors on human
oligodendrocytes: implications for multiple sclerosis. Brain 2005;128:1003-15.
3. Mi S, Miller RH, Lee X, et al. LINGO-1 negatively regulates myelination by
oligodendrocytes. Nat Neurosci. 2005;8:745-51.

4. John GR, Shankar SL, Shafit-Zagardo B, et al . Multiple sclerosis: re-expression of


a developmental pathway that restricts remyelination. Nat Med. 2002;8:1115-21.

70.- Masculino de 66 aos, con Insuficiencia Cardiaca por cardiopata hipertensiva, en


situacin estable (en clase funcional I segn grado de disnea), presenta en el Ecocardiograma, Disfuncin Sistlica (Fraccin de Eyeccin < 35%). EL grupo de frmacos que
est ms indicado como tratamiento inicial en ste paciente es:

a)
b)
c)
d)

Digitalicos.
Inhibidores de la ECA.
Antagonistas de Calcio.
Betabloqueantes.

IC Izquierda:
Disnea al ejercicio, tos, fatiga, ortopnea, DPN, cardiomegalia, estertores,
ritmo de galope, congestin venosa pulmonar.
IC Derecha:
Presin venosa elevada, hepatomegalia, edema, usualmente asociado a IC
Izq.
TIPOS:
Insuficiencia Cardaca Aguda

Edema pulmonar agudo


Shock Cardiognico

Insuficiencia Cardaca Crnica

Estado Fisiopatolgico Caracterizado por la incapacidad del corazn para bombear la


cantidad de sangre necesaria para abastecer el metabolismo celular.
La ICC representa un sndrome clnico complejo caracterizado por alteraciones de funcin
ventricular y de regulacin neurohormonal que se acompaa de:
-intolerancia al esfuerzo.
-Reduccin en la calidad de vida.
-Reduccin de la esperanza de vida.

SINTOMAS:
Disnea
Disminucin capacidad funcional
Sntomas urinarios
Sntomas cerebrales
Sntomas insuficiencia cardaca derecha

PROPUESTA DE UN ESQUEMA DE TRATAMIENTO


Fibrilacin A.

Diurtico
I ECA

contraindicacin o efecto
adverso

ARA
Siguen sntomas

BB

Espironolactona 12,5 a 25 mg/d


o

Digoxina 0,125 mg a 0,25 mg/d

Preferible ingreso hospitalario


inicial con seguimiento
mensual

Se ha demostrado claramente la importancia del control neurohumoral en el


paciente con disfuncin ventricular.

Manejo inicial del paciente con ICA


= Diurtico + Vasodilatador.

Solo el paciente con ICA + BGC = Inotrpico.

Nuevos estudios clnicos e investigacin bsica se requiere para buscar nuevas


estrategias de manejo

REFERENCIAS
Los inhibidores de la enzima conversora de angiotensina Rev Cubana Cardiol Cir
Cardiovasc 1997:11;29-47.

71.- Se trata de femenino de 4 aos de edad tiene lesiones eczematosas crnicas en


flexuras de brazos y piernas que producen intenso prrito, asociadas a una queilitis
descamativa de labios. Entre los siguientes, es el diagnstico ms probable es:
a)
b)
c)
d)

Eczema seborrico.
Prrigo nodular.
Dermatitis atpica.
Eczema microbiano

La dermatitis atpica llamada comnmente eccema (atpico), es una enfermedad que


consiste en un estado reaccional de la piel caracterizado por erupciones pruriginosas y
con aspecto de escamas, ms frecuente en nios, multifactorial, en la cual intervienen
factores tanto ambientales como constitucionales. Las personas con eccema a menudo
tienen antecedentes de condiciones alrgicas como asma, fiebre del heno o eccema. La
dermatitis atpica fue originalmente conocida como prrigo de Besnier y eccema
constitucional, actualmente tambin es llamada neurodermatitis diseminada, por las
escuelas europeas.
Clnica
Las manifestaciones clnicas tpicas de la dermatitis atpica se dividen en tres etapas, que
suelen denominarse del lactante, infantil y del adulto. Junto a ellas se encuentran otras,
con frecuencia llamadas atpicas, a pesar de que muchas, como la xerosis, son muy
constantes.

Dermatitis atpica del lactante


Suele empezar hacia los cinco meses de vida, pero puede hacerlo antes. Algunos nios
desarrollan lesiones de eccema seborreico, que de forma gradual va adquiriendo el
aspecto de la dermatitis o eccema atpico.
.
La localizacin ms habitual es en la cara, respetando las zonas alrededor de los ojos, la
nariz y la boca (. Son tambin frecuentes en el cuero cabelludo, las orejas, el dorso de las
manos y las zonas de extensin de las extremidades.
Las lesiones suelen ser ppulas o placas eritematosas y edematosas, muchas veces con
erosiones, exudacin y costras.
Es muy raro que se aprecien las vesculas caractersticas del eccema. El prurito es un
sntoma constante.
Dermatitis atpica infantil
Este periodo suele considerarse con un inicio hacia los dos aos y un final entre los siete
aos y la pubertad. Las lesiones caractersticas se observan sobre todo en las flexuras,
en especial en los codos y las rodillas (Fig. ), pero pueden aparecer en otras zonas.
En esta fase es ms fcil ver lesiones eccematosas con vesculas, pero el intenso prurito
hace que enseguida se transformen en erosiones, con exudacin y formacin de costras.

Bibliografa
1. Bielsa Marsol I. Eccemas (II). En: Ferrndiz C, ed. Dermatologa Clnica. Madrid,
Mosby/
Doyma Libros 1996, 113-124.
2. Fernndez Vozmediano JM y cols. Dermatitis atpica. Madrid, Jarpyo 1994.
3. Fernndez Vozmediano JM, Armario Hita JC. Tacrolimus. Piel 2001;16:48-54.
4. Fonseca Capdevila E. Dermatitis atpica. Protocolo teraputico. (En lnea)
(14.02.2001).
Disponible en www.especialistasdermatologia.com.
5. Fonseca E. Dermatitis atpica en la infancia. Salud Rural 1997;14:92-105.
6. Guerra Tapia A. Dermatitis atpica. En: Fonseca Capdevila E, ed. Dermatologa
Peditrica.

Madrid, Aula Mdica 1999, 83-180.

72.- Which one of the following conditions results in prologation of the partial
thromboplastin time (PTT), but not the prothrombin time (PT)?
a)
b)
c)
d)

Varicela hemorrhage as a result of cirrosis


Therapy with broad-spectrum antibiotics
Therapy with coumarin for phlebitis
Menorrhagia resulting from von Willebrands disease

Enf. Von Willebrand

Manifestaciones

Epistaxis

60 %

Hemorragia transvaginal

50 %

Equimosis

40 %

Gingivorragias

35 %

Hematomas

5%

Hemartrosis

3%

EvW
Pruebas de escrutinio
T. Hemorragia

Prolongado

C. Plaquetaria

Normales, excepto 2B

TTPa

Normal o prolongado

TP

Normal

Gpo AB0

25 % bajo en 0

1.-Lee GR, Foerster J, Lukens J, Paraskevas F, Greer JP and Rodgers GM.


Wintrobes clinical haematology; 10th Edition, Lippincott Williams & Wilkins, United
States of America, 1999.
2.-Williams WJ. Manual Williams de hematologa
5a Edicin McGraw-Hill Interamericana, Mxico, 1997.
3.-Beutler E, Lichtman MA, Coller BS, Kipps T. Hematology. 5th International
Edition. United Stated of America, 1995.

4.-Ruiz Argelles GJ. Fundamentos de hematologa ; 2. Edicin, Editorial Mdica


Panamericana, Mxico, 1998.

73.- Masculino 60 aos de edad de ocupacin ganadero, originario de Tabasco Mx.


Presenta cambios de personalidad. Su familia indica que varias semanas antes se quej
de dolor en la mueca y hace una semana tuvo asimetra facial transitoria. De los
siguientes cual es el dato de mayor probabilidad para presentar estos sntomas:

a) Exposicin a toxinas
b) Hbitos sexuales
c) Picadura de garrapata
d) Picadura de mosquito
Allen R. M. MMS Medicina Interna. 5. Edicin. National Medical Series. Mc. Graw Hill.
2006. (captulo 8 VII F 3 b, c).

La enfermedad de Lyme se transmite por picadura de garrapata. Las complicaciones


neurolgicas son neuropata craneal, radiculopata y encefalopata. Las actividades en el
exterior de este paciente lo ponen en riesgo de exposicin a enfermedad de Lyme si vive
en un rea endmica y la sugerencia de artritis de la mueca concuerda con las
manifestaciones sistmicas de aqulla. La exposicin a toxinas puede causar confusin,
pero no produce artritis o debilidad facial. Una conducta sexual de alto riesgo predispone
a infeccin por virus de la inmunodeficiencia humana (VIH) que puede causar parlisis de
nervio facial y encefalopata. Sin embargo, dados los antecedentes y la presentacin de
este enfermo, es ms probable la enfermedad de Lyme. Una picadura de mosquito puede
transmitir encefalitis vrica. Sin embargo, no hay antecedente de fiebre y la artritis de la
mueca y la debilidad facial no apoyan un diagnstico de encefalitis vrica temprana. El
consumo excesivo de vitamina no produce artritis, neuropata de pares craneales ni
cambios de la personalidad.

74.- Se trata de masculino de 6 aos de edad quien es llevado a consulta por sus padres
quienes refieren que el menor lleva varias noches despertndose agitado como si
hubiera soado algo que le angustia. Cuando acuden a su lado por la noche, el nio les
mira y dice palabras que no tienen ningn significado. Al cabo de un rato vuelve a
dormirse y por la maana no recuerda nada de lo ocurrido. El diagnstico ms probable
es:

a)
b)
c)
d)

Pesadillas
Disomnia.
Terrores nocturnos.
Sonambulismo.

Criterios para el diagnstico de F51.5 Pesadillas (307.47)


A. Despertares repetidos durante el perodo de sueo mayor o en las siestas diurnas,
provocados por sueos extremadamente terrorficos y prolongados que dejan recuerdos
vividos, y cuyo contenido suele centrarse en amenazas para la propia supervivencia,
seguridad o autoestima. Los despertares suelen ocurrir durante la segunda mitad del
perodo de sueo.
B. Al despertarse del sueo terrorfico, la persona recupera rpidamente el estado
orientado y despierto (a diferencia de la confusin y desorientacin que caracterizan los
terrores nocturnos y algunas formas de epilepsia).
C. Las pesadillas, o la alteracin del sueo determinada por los continuos despertares,
provocan malestar clnicamente significativo o deterioro social, laboral o de otras reas
importantes de la actividad del individuo.
D. Las pesadillas no aparecen exclusivamente en el transcurso de otro trastorno mental
(p. ej., delirium, trastorno por estrs postraumtico) y no se deben a los efectos
fisiolgicos directos de una sustancia (p. ej., drogas, frmacos) o de una enfermedad
mdica.
Criterios para el diagnstico de F51.4 Terrores nocturnos (307.46)
A. Episodios recurrentes de despertares bruscos, que se producen generalmente durante
el primer tercio del episodio de sueo mayor y que se inician con un grito de angustia.
B. Aparicin durante el episodio de miedo y signos de activacin vegetativa de carcter
intenso, por ejemplo, taquicardia, taquipnea y sudoracin.
C. El individuo muestra una falta relativa de respuesta a los esfuerzos de los dems por
tranquilizarle.

D. Existe amnesia del episodio: el individuo no puede describir recuerdo alguno detallado
de lo acontecido durante la noche.
E. Estos episodios provocan malestar clnicamente significativo o deterioro social, laboral,
o de otras reas importantes de la actividad del individuo.
F. La alteracin no se debe a los efectos fisiolgicos directos de una sustancia (p. ej.,
drogas, frmacos) o de una enfermedad mdica.

75.- Masculino de 66 aos diabtico, hipertenso que inicia con cefalea intensa, sbita
posterior a esfuerzo fsico acompaada de vmito, malestar general, posteriormente se
presenta prdida del estado de alerta, usted piensa en:
a) Infarto Cerebral Agudo
b) Oclusin de cartida izquierda total
c) Hemorragia subaracnoidea
d) Migraa complicada

De las enfermedades cerebro-vasculares, la hemorragia subaracnoidea es la causa ms


comn de muerte sbita y usualmente los pacientes fallecen antes de llegar al hospital
(hipertensin intracraneana sbita, hemorragia intraventricular, edema pulmonar).
De fcil reconocimiento es el cuadro clnico; una cefalea sbita, intensa que el paciente
describe "como la ms fuerte toda la vida", relacionado con el ejercicio o maniobras de
Valsalva (esfuerzo, coito, pujo, bao con agua fra), se acompaa de vmito y prdida del
estado de alerta. En el examen fsico los clsicos signos menngeos con alteracin en
estado de conciencia.
A veces, durante el examen podemos ubicar signos focalizadores que nos indican dnde
est la lesin responsable del sangrado (III par: comunicante posterior; paresia
faciobraquial: cerebral media, paraparesia: comunicante anterior). La presin arterial est
elevada y la bradicardia expresa la hipertensin endocraneana.
Cuando se interroga a estos pacientes se encuentra en un porcentaje que oscila entre 15
y 39% un dolor de cabeza no muy fuerte, "distinto", asociado con esfuerzo y que no se
acompaa de fotofobia o meningismo. Es la clsica cefalea centinela y cuya clave
diagnstica no est en la intensidad del dolor sino en lo diferente de sus caractersticas
(especialmente en pacientes jvenes con antecedentes de migraas). Se debe a un
pequeo sangrado subaracnoideo que incluso puede transcurrir sin mayor impacto clnico.
Berbeo M., Alvernia J. y col. Protocolo para el diagnstico y el tratamiento de la
hemorragia subaracnoidea espontnea. Universitas Mdicas. Edicin Especial, vol. 41
Nmero 120, 34-38.
Fisher C.M., Kistler J.P., y cols. Relation of cerebral vasospasm to subarachonoid
hemorrhage visualized by computed tomography. Neurosurg 6: 1-9,

76.- Masculino de 19 aos que es enviado al servicio de otorrino por presentar una
tumoracin en la lnea media del cuello, la cual es renitente, no dolorosa y asciende al
protruir la lengua.
El diagnstico clnico ms probable es:
a)
b)
c)
d)

Qiste tirogloso
Quiste Branquial
Quiste demoide
Linfangioma cervical.

Los quistes del conducto tirogloso constituyen las lesiones cervicales congnitas ms
frecuentes en los nios. Habitualmente son detectados entre los 2 y 10 aos de edad
aunque un porcentaje importante no son identificados sino hasta despus de los 20 aos.
No existe una predileccin por sexo.
La forma de presentacin clnica clsica es la de una lesin redondeada, lisa, blanda,
indolora, ubicada en la lnea media y en relacin al hueso hioides. Existe un pequeo
porcentaje de presentaciones sublinguales o supraesternales. Como el quiste tiene
fijaciones al hueso hioides y al foramen ciego de la lengua, puede ser traccionado hacia
arriba, cuando el nio traga o saca la lengua.
El diagnstico es confirmado por ultrasonido, que corrobora la lesin qustica. Este
estudio habitualmente permite tambin evaluar la presencia de la glndula tiroides. Al no
encontrarse sta en el examen, es preciso realizar una cintigrafa, ya que en esos casos
el nico tejido tiroideo existente pudiera estar en relacin al conducto o quiste tirogloso.
Embriolgicamente estas lesiones son consecuencia de la falla de obliteracin del
conducto tirogloso posterior al descenso de la glndula tiroides alrededor de la sexta
semana de vida fetal. Anatomo-patolgicamente los quistes se encuentran revestidos de
epitelio pseudeoestratificado columnar o estratificado escamoso y glndulas secretoras de
mucus.
La complicacin habitual de estas malformaciones es la infeccin con la flora bacteriana
de la boca, consecuencia de su comunicacin persistente con la base de la lengua a
travs del foramen ciego. Los quistes infectados presentan signos inflamatorios y pueden
drenar al exterior.
El tratamiento recomendable es la extirpacin quirrgica y as prevenir su infeccin. La
tcnica fue descrita en 1920 por Sistrunk e incluye la extraccin del centro del hueso
hioides, por cuyo interior va el conducto tirogloso, y as, evitar la recidiva. La extirpacin
debe llegar hasta el foramen ciego lingual.
En presencia de infeccin qustica, es necesario tratar sta con antibiticos y
posteriormente operar.

77.- Femenino de 50 aos de edad, que acude al servicio de consulta externa , refiere
rubor, tumefaccin y rigidez de las articulaciones interfalngicas dstales desde hace tres
meses, rigidez que no rebasa ms de 30 mins. Niega otras molestias articulares. De los
siguientes diagnsticos el ms probable es:
a) Osteoartritis erosiva
b) Artritis reumatoide
c) Espondilitis anquilosante
d) Esclerodermia
Allen R. M. MMS Medicina Interna. 5. Edicin. National Medical Series. Mc. Graw Hill.
2006. (captulo 10 V E 1). Tpicamente, la osteoartritis erosiva afecta las articulaciones
interfalngicas dstales en mujeres de edad madura. Es improbable que esos sntomas
articulares dstales prominentes sucedan en pacientes con artritis reumatoide o con lupus
eritematoso diseminado sin molestias articulares ms generalizadas. No hay pruebas que
indiquen espondilitis anquilosante o esclerodermia.

78.- La diferencia principal entre los estudios observacionales y los experimentales es que
en los estudios experimentales:
a) Los grupos de estudio y de control son del mismo tamao
b) El investigador determina el grupo de sujetos que recibir la exposicin
c) Los grupos de estudio y de control son compatibles y tienen las mismas caractersticas
d) Los sujetos en estudio no reciben ninguna intervencin
Aunque existen mltiples clasificaciones de los diseos de investigacin, la mayora de los
autores estn de acuerdo en la existencia de dos grandes tipos: experimentales y
observacionales (no experimentales). La diferencia bsica estriba en el mtodo empleado
para asignar a los sujetos del estudio a la exposicin o intervencin.
Ruiz M. A. Epidemiologa Clnica, 1. Ed. 2004; pg: 234

79.- Femenino de 2 aos, es llevada a urgencias refiriendo la madre llanto enrgico al


orinar. Antecedentes: Manejo con antibitico por cuadro de IVU previo, 2 semanas
despus los estudios complementarios de imagen muestran reflujo vesicouretral grado II.
El tratamiento a corto plazo de esta paciente debe incluir:

a) Cultivo de orina inmediato.


b) Profilaxis antibitica
c) Ciruga.
d) Cistoureterografa miccional cada 6-8 semanas.

El tratamiento depender del tipo de reflujo y graduacin del mismo. Habitualmente el


RVU grado 1 y grado 2 remiten espontneamente con el crecimiento del nio.Debern
recibir profilaxis antibitica y realizar controles mensuales de urocultivo durante los
primeros tres meses, luego en forma bimestral hasta el sexto mes y luego cada tres
meses hasta los 2 aos de producida la infeccin urinaria.El RVU grado 3 requiere
iguales controles como los mencionado previamente pero con la salvedad de que no
remiten espontneamente. Aproximadamente en ms del 50% de los nios ser
necesario la correccin quirrgica. En el RVU 4 y 5 la conducta teraputica habitual es la
correccin quirrgica.

Tanagho E, Mc Aninch J, Urologa general de Smith, Manual Moderno, 14 Edicin,


p.p. 210-211

80.- Mujer de 23 aos G/1 la cual cursa con 5 semanas de gestacin tom cumarina
hasta el da en que se enter que estaba embarazada. Est preocupada de que la
cumarina ocasione defectos congnitos en el producto. En el caso anterior el defecto
congnito ms frecuente tras el uso de cumarina es

a)
b)
c)
d)

Hipoplasia nasal
Espina bfida
Anencefalia
Acondroplasia

Tabla
Frmacos

I
cuya

FRMACO

teratognia

se

ha

comprobado

MALFORMACIONES DETECTADAS

en

humanos

RIESGO

ANTIBIOTICOS
Tetraciclinas

Tincin dental, hipoplasia enamel. Posible Alrededor de 50%


retraso crecimiento seo.
de los expuestos a
tetraciclina; 12,5%
de
expuestos
a
oxitetraciclina.

ANTICOAGULANTES
Cumarinas

Sndrome fetal de la warfarina:


hipoplasia nasal, condrodisplasia
punctata, braquidactilia, defectos
craneales, orejas anomalas, ojos
malformados, malf. en sistema nervioso
central, microcefalia, hidrocefalia,
deformidades esquelticas, retraso
mental, atrofia ptica, espasticidad;
malformacin de Dandy Walker.

El 16% tiene
malformaciones,
un 3%
hemorragias. Se
producen hasta un
8% de abortos.

ANTIEPILEPTICOS
Carbamacepina

Aumento del riesgo de sufrir defectos del Se estima un riesgo


tubo neural.
del 1%.

Fenitoina

Sndrome
fetal
de
la
fenitoina:
aplanammiento puente nasal, pliegues
epicantales internos, ptosis, estrabismo,
hipertelorismo, orejas anormales o de baja
implantacin, hipoplasia de falanges

El 5-10% puede
sufrir el sndrome
tpico.
Un
30%
parcialmente. Hasta
un
7%
de

distales y uas, anomalas esquelticas, descendientes con


microcefalia y retraso mental, deficiencias coeficiente
del crecimiento, neuroblastoma, defectos intelectual bajo.
cardacos, paladar hendido y labio
leporino.
Trimetadiona
(Troxidona)

Sndrome fetal de la trimetadiona: retraso


crecimiento
intrauterino,
anomalas
cardacas, microcefalia, labio y paladar
hendidos, anomalas en orejas, facies
dismorfica, retraso mental, fstula traqueoesofgica, muerte postnatal.

Basndose en casos
notificado: 83% y
32%
de
muerte
infantil o neonatal.

Acido valproico

Espina bfida con mielomeningocele, Un 1% de riesgo de


defectos en sistema nervioso central, defectos del tubo
microcefalia, defectos cardiacos.
neural.

ANTINEOPLASICOS
Alquilantes
Busulfan
Clorambucil
Ciclofosfamida
Clormetina

Retraso del crecimiento, paladar hendido,


microftalmia, hipoplasia ovrica, opacidad
corneal, agenesia renal, malformaciones
de los dedos, defectos cardacos, otras
anomalas mltiples.

Entre el 10-50% de
los casos pueden
presentar
malformaciones,
aunque este riesgo,
extrado de series
de casos puede
estar exagerado.

Hidrocefalia, meningoencefalocele,
anencefalia, malf. craneales, hipoplasia
cerebral, retraso del crecimiento,
malformaciones oculares y del odo,
malformaciones nasales, paladar hendido,
malf. en miembros y dedos.

Entre el 7-75% de
los casos expuestos,
aunque este riesgo,
extrado de series
de casos puede
estar exagerado.

Antimetabolitos
Aminopterina
Azauridina
Citarabina
Fluorouracilo
Mercaptopurina
Metotrexato

Sndrome fetal de la aminopterina:


disostosis craneal, hidrocefalia,
hipertelorismo, anomalas de odo externo,
micrognatia, paladar hendido.
HORMONAS
Dietilestilbestrol

Descendencia femenina:
Adenocarcinoma de clulas claras vaginal
o cervical en mujeres jvenes expuestas
"in utero" (antes de la semana 18);

Cambios
morfolgicos
congnitos
en
epitelio vaginal en el

oligomenorrea, reduccin de las tasas de


embarazo, incremento de las tasas de
embarazos pretermino, incremento de la
mortalidad perinatal y de aborto
espontneo.
Descendencia masculina:
Quistes epididimarios, criptorquidia,
hipogonadismo, disminucin de la
espermatognesis, estenosis de meato,
hipospadias.

39%
de
las
expuestas.
En
exposicin antes de
la
18
semana:
riesgo de carcinoma
1.4 por 1000 de
expuestas.

NEUROLEPTICOS
Litio

Mayor riesgo de anomala de Ebstein; no


se ha detectado un mayor riesgo de otro
tipo de malformaciones.

PENICILAMINA
Hiperclastosis cutnea (cutis laxa)

Pocos
casos.
Riesgo desconocido.

Aborto
espontneo,
deformidades
craneales, de orejas, cara, corazn,
extremidades e hgado; hidrocefalia,
microcefalia, defectos cognitivos (incluso
sin malformaciones aparentes).

Isotretinoina: 38%.
El
80%
son
malformaciones del
sistema
nervioso
central.

Focomelia,
amelia,
hipoplasia
de
miembros, defectos cardacos congnitos,
malformaciones renales, criptorquidia,
parlisis VI par, sordera, microtia, anotia.

Alrededor del 20%


cuando
la
exposicin
ocurre
entre 4-8 semana.

RETINOIDES
SISTEMICOS
Isotretinoina
Etetrinato

TALIDOMIDA

Bibliografa.

McBride WG. Thalidomide and congenital abnormalities. Lancet 1961; 2:1358.


Lenz W. Thalidomie and congenital abnormalities. Lancet 1962;1:45.
Shepard TH. Catalog of Teratogenic Agents. Johns Hopkins University Press
London, 1987.
Piper JM. Banur C, Kennedy DI. Prescription drugs use in pregnancy in a Medicaid
population. Am J Obstet Gynecol 1987; 157:148-156.

Salvador J, Martnez-Fras ML, Rodrguez Pinilla E. Consumo de medicamentos


por la mujer embarazada en Espaa. Ministerio de Sanidad y Consumo. Madrid,
1989.
Grupo de trabajo DUP. Estudio multicntrico sobre el uso de medicamentos
durante el embarazo en Espaa (I). Mtodos y caractersticas demogrficas de la
poblacin estudiada. Med Clin 1990;95:764-767.
Grupo de trabajo DUP. Estudio multicntrico sobre el uso de medicamentos
durante el embarazo en Espaa (II). Los frmacos utilizados durante la gestacin.
Med Clin 1991;96:11-15.
Grupo de trabajo DUP. Estudio multicntrico sobre el uso de medicamentos
durante el embarazo en Espaa (III). Los frmacos utilizados durante el primer
trimestre de la gestacin. Med. Clin 1991;96:52-57.

Martnez-Fras ML. Medicamentos y teratogenia. Revisin bibliogrfica y situacin en


Espaa. Ministerio de Sanidad y Consumo. Madrid, 1989.

81.- Femenino de 34 aos, es atendida en sala de partos secundario a eutocia, durante


la reparacin de la episiotoma media hay un marcado incremento en el sangrado
transvaginal. La medida teraputica inmediata en esta paciente es:

a) Masaje y compresin del fondo uterino


b) 20 unidades IV de oxcitocina
c) 0.2mg Im de metilergonovina
d) Empaquetar con gasas

El tratamiento clsico de la atona uterina est constituido por la reposicin volumtrica


(sangre, coloides y cristaloides), la utilizacin de masajes y por la administracin de
drogas que promueven la contraccin del msculo uterino. Cuando estas
Medidas no son eficientes, se procede casi invariablemente a la histerectoma de
hemostasia. Es de notar, que la atona uterina puede aparecer en el primer embarazo y
sin ningn antecedente previo. La atona uterina constituye, en casi todas las series
mundiales, ms del 50% de las hemorragias graves del posparto.
Patologa de Urgencia, Ao 9, Nro. 3, Septiembre de 2001

82.- Femenino de 23 aos de edad, refiere que desde hace un par de meses ha
presentado hemorragia irregular o postcoital, actualmente presenta disuria y dolor
abdominal usted debe sospechar en cervicitis por:

a)
b)
c)
d)

Micoplasma
Gardnerella
Chlamydia
Candida

Segn los datos de la Organizacin Mundial de la Salud, anualmente se detectan 89 000


000 de nuevas infecciones por Chlamydia trachomatis en el mundo. Esta infeccin
provoca uretritis y cervicitis, y las secuelas incluyen enfermedad inflamatoria plvica,
embarazo ectpico, infertilidad por dao tubrico, epididimitis, proctitis y artritis reactiva.
Se considera principalmente un problema de salud en la mujer, en ella las
manifestaciones y consecuencias son ms dainas para la salud reproductiva. Los
individuos infectados con Chlamydia trachomatis pueden portar el microorganismo por
meses o aos y transmitir la enfermedad a sus parejas sexuales. Su diagnstico sigue
siendo un reto, ya que quienes la padecen presentan sntomas muy leves o son
portadores asintomticos.

Manifestaciones clnicas

La cervicitis es la manifestacin clnica ms frecuente de la infeccin por C. trachomatis


en la mujer. Sin embargo, el 70% de las mujeres infectadas no tienen sntomas, mientras
que en el tercio restante las evidencias clnicas son poco especficas de infeccin, como
flujo
genital,
dolor
abdominal
o
pelviano,
sangrado
y/o
disuria.
La presencia de disuria puede indicar una uretritis acompaante, lo que sucede en el 35%
de los casos. En otras oportunidades, solo la uretra est comprometida, y la infeccin
uretral se manifiesta como piuria o disuria con cultivo negativo (23% de los casos).
El diagnstico se realiza al examinar el hisopado endocervical, que muestra flujo
amarillento o verdoso con ms de 10 PMN por campo de inmersin en el examen de
Gram. Este resultado define la cervicitis mucopurulenta (CMP) la cual tambin puede ser
producida en casos de infeccin por gonococo o mixta (C. trachomatis y gonococo). Por lo
tanto, el diagnstico debe confirmarse mediante estudios de mayor especificidad, como
las tcnicas moleculares (test de ligasa, PCR), que tienen una sensibilidad del 96%
aproximadamente, o la deteccin del antgeno por tcnica de ELISA, con una sensibilidad
del 75%. Tambin se ha demostrado que las tcnicas moleculares en el primer chorro de
orina son especficas y altamente sensibles.

Cuatro de cada diez mujeres con cervicitis no tratada adquieren enfermedad inflamatoria
pelviana (EPI), con mayor riesgo de sufrir embarazo ectpico, infertilidad y dolor crnico
pelviano. El riesgo de infertilidad se eleva segn el nmero e intensidad de los episodios:
alrededor del 10% despus de un episodio, del 30% despus de dos, y mayor del 50% si
ha habido tres o ms episodios. Por otro lado, el embarazo ectpico es cinco a siete
veces ms frecuente cuando se trata de pacientes con antecedentes de EPI.

Tratamiento
CLAMIDIA
TRACHOMATIS
Azitromicina 1 g
VO dosis nica
Doxiciclina 100 mg VO cada 12 horas por 7 das
Eritromicina 500 mg VO cada 6 horas por 7 das
Ofloxacina 300 mg
VO cada 12 horas por 7das
Levofloxacina 500 mg VO cada 24 horas por 7 das

Basado en Guas Clnicas para el manejo de las ITS, OMS, 2003 / Norma Oficial
Mexicana NOM -039-SSA2-2002
Berek J. (2002) Ginecologa de NOVAK. Mxico. Ed. Mc Graw Hill Interamericana. Pag
293.

83.- Las caractersticas ante la sospecha en un recin nacido a trmino de ictericia


fisiolgica son:

a) Inicia el primer da de vida, tiene una duracin menor de 10 das y la concentracin


mxima es de 20 mg/dl
b) Inicia entre el segundo y tercer da de vida, tiene una duracin mayor de 10 das y la
concentracin mxima es de 12 mg/dl.
c) Inicia a la semana de nacimiento y dura una semana ms.
d) Inicia entre el segundo y tercer da de vida, tiene una duracin menor de 10
das y la concentracin mxima es de 12 mg/dl

Manual CTO pediatra, 7 edicin p. 1306.


La ictericia fisiolgica del recin nacido inicia entre el segundo y tercer da de vida, tienen
una duracin entre 5 y 7 das y alcanza una concentracin mxima de 12 mgdl a los 2 4
das. En cambio la ictericia no fisiolgica inicia dentro de las primeras 24 horas de vida,

tiene una duracin superior de 10 a 15 das, la bilirrubina total en RNT es mayor de 12


mg/dl o mayor de 14 mg/dl en RNPT, el incremento de bilirrubina es mayor de 5 mg/dl en
24 horas y la bilirrubina directa es mayor de 1 mg/dl.

84.- Se trata de masculino recin nacido quien referido a consulta con sospecha de
luxacin de la articulacin de la cadera, la siguiente maniobra que nos corrobora dicha
patologa es:

a) Signo de Galeazzi
b) Signo de Barlow
c) Signo de Ortolani
d) Signo de Pistn

Prueba de Ortolani: Con esta prueba se detecta una cadera ya luxada, se coge con la
mano el miembro flexionado, la cadera se coloca en abduccin mientra se levanta el fmur
con cuidado y se sita los dedos a nivel del trocnter mayor. Si la prueba es positiva se
siente la reduccin de la cadera dentro del acetbulo.
Prueba de Barlow: es una prueba inductora para identificar una cadera inestable pero an
localizada en su sitio; no es una prueba adecuada para diagnosticar luxacin de cadera. La
cadera en aduccin ligera y con la palma de la mano se empuja suave y cuidadosamente
hacia atrs, la presencia de un movimiento de pistn o al percepcin de una cabeza
femoral subluxada sobre el borde posterior del acetbulo.
Prueba de Galeazzi: con el nio acostado se le flexionan las caderas y rodillas de modo
que los talones se apoyen sobre la mesa y reconocer el acortamiento relativo del muslo.

Skinner, H. Diagnstico y tratamiento en Ortopedia. Ed. Manual Moderno. Mxico, 2004.


pp. 625

85.- Femenino de 33 aos que inici con disfona y disfagia, es referido al servicio de
endocrinologa donde es confirmado el diagnostico de ndulo tiroideo nico. El estudio
recomendado de eleccin en sta paciente es:
a) Biopsia por aspiracin con aguja fina
b) Gammagrama tiroideo
c) Tomografa computada de cuello
d) Repetir Pruebas de funcin tiroidea

El ndulo tiroideo solitario se define como el crecimiento localizado de la glndula tiroides,


usualmente es benigno, la prevalencia es del 4 al 7% en la poblacin general. Aunque el
cncer tiroideo es el tumor endocrino ms comn slo representa el 1% de todos los
cnceres y 5% de todos los ndulos tiroideos. El estudio clnico diagnstico y teraputico
debe iniciarse con historia clnica completa, exploracin fsica y exmenes de laboratorio
que incluyan un perfil tiroideo para evaluar funcin de la glndula. La citologa tiroidea
por aspiracin (CTA) es el principal procedimiento diagnstico en los pacientes con
ndulo tiroideo solitario, por ser capaz de diferenciar lesiones benignas de las
malignas, sus principales ventajas son: segura, reduce costos de atencin mdica,
selecciona mejor los pacientes que sern sometidos a tratamiento quirrgico y se
realiza en pacientes ambulatorios

1. Torres AP, Hernndez SE, Caracas PN, Serrano GI et al Diagnstico y tratamiento


del ndulo tiroideo. Rev Edocrinol Nutr 2000; 8 (3): 87-93

86.- Mujer de 37 aos. Acude a consulta por presentar cefalea, cansancio e


irregularidades menstruales con ritmo de 36 a 50 x 2-3 das. No se ha podido embarazar
despus de 18 meses de actividad sexual regular. No tiene antecedentes importantes. EF:
Campos visuales normales, tiroides aumentada de tamao una vez y aumentada de
consistencia, no tiene galactorrea. Resto normal. Laboratorio: qumica sangunea, Bh y
electrolitos normales. Prolactina 47 ng/dL (< 25), perfil tiroideo: TSH 18 mUI/ml, T4t: 50
nmol/L (57.9 a 154.4), T4L: 7.7 pmol/L (9 a 24), T3T: 1.06 nmol/L (1.2 a 2.9), T3L: 1.96
pmol/L (3 a 6.31)
La etiologa ms frecuente de este problema es:
a)
b)
c)
d)

Tiroiditis autoinmune crnica


Microadenoma hipofisario
Resistencia a la insulina
Enfermedad de Graves

Inicialmente el hipotiroidismo se diagnosticaba mediante la cuantificacin por tcnicas de


Radio Inmuno Anlisis (RIA) de las hormonas circulantes triyodotironina y tiroxina; el
proceso era lento y sometido a muchos factores de error que hacan su sensibilidad y
especificidad poco confiables. Posteriormente, se desarrollaron tcnicas para la medicin
de la TSH hipofisiaria igualmente mediante el RIA lo que mejor en forma importante la
sensibilidad para el diagnstico de esta enfermedad; sin embargo, los niveles de
deteccin de la prueba se encontraban en el orden de 1 IU/ml lo que haca que la prueba
no fuera sensible para valores menores de 1 IU/ml. Debido a esto se crearon tcnicas de
segunda generacin mediante la cuantificacin de TSH por anticuerpos monoclonales y
RIA, el IRMA (Immuno Radiometric with Monoclonal Antibodies) que permiti detectar
valores de TSH en rangos de 0.1 IU/ml; posibilitando desde entonces diagnosticar
pacientes con hipertiroidismo primario; pero con la limitante de que para esta tcnica era
imposible detectar valores de TSH menores de 0.1 IU/ml por lo que se creo la medicin
de TSH mediante quimioluminiscencia o mtodos enzimticos, es decir las tcnicas de
tercera generacin, las cuales pueden detectar valores de TSH de 0.01 IU/ml; con lo
que se logra el espectro ideal para una prueba de laboratorio que tiene la capacidad de
diagnosticar tanto la hipofuncin como la hiperfuncin(20).

Adems el avance no slo fue en la medicin de TSH sino tambin en las hormonas
tiroideas que han evolucionado simultneamente con la TSH y ya se miden incluso las
fracciones libres de hormonas y las fracciones totales, lo que ha facilitado el manejo de
estos pacientes. Gracias a esta evolucin en tcnicas de laboratorio, el diagnstico de
hipotiroidismo primario es bastante sencillo. Niveles de TSH superiores al valor mximo

de la tcnica seran diagnsticos de la disfuncin; pero no es tan fcil. Cuando tenemos


un paciente con toda la sintomatologa del hipotiroidismo y la TSH se encuentra elevada
el diagnstico es obvio; pero podemos tener pacientes con sntomas muy inespecficos
como depresin y con examen fsico normal a quienes se les encuentran valores de TSH
por encima del lmite superior y con hormonas tiroideas normales. Se trata de un
hipotiroidismo o es un valor ligeramente elevado ocasional de una persona sana(21).
Igualmente tenemos otra circunstancia que ha sido descrita con mayor frecuencia:
pacientes con valores de TSH en el lmite superior normal y con dislipidemia a quienes se
les da tratamiento con hormonas tiroideas y su dislipidemia se corrige manteniendo
valores de TSH en rangos normales. Todas las circunstancias anteriores han hecho que
aparezca en el hipotiroidismo primario la expresin de hipotiroidismo subclinico, que ha
sido objeto de reuniones y congresos dedicados exclusivamente a este tema. La sociedad
Europea de Tiroides hace algunas recomendaciones para el manejo de esta situacin que
se consideran tiles como gua (Tabla ).

Tabla. Enfoque del paciente con disfuncin tiroidea de acuerdo a los niveles de
hormona estimulante de la tiroides (TSH).

Si TSH < 0.4m U/L

Si TSH 0.4
a
2.0
mU/L
Si TSH 2.01 a 5.0mU/L

Si TSH
mU/L

>

5.0

Normal,
Medir T3 y T4 totales o Repetir
Dar tratamiento
libres para diagnostico cada cinco Medir T4 libre y anticuerpos para
de hipertiroidismo.
aos
antitiroideos
hipotiroidismo
1. Si AAT (-) y T4 libre es
normal repetir screening cada
ao. Si TSH es > 4.0mU/l en
dos ocasiones dar tratamiento
2. Si AAT (+) y/o T4 libre esta
baja o normal baja tratar si TSH
es mayor de 3.0 mU/l y
observar a los otros
Tomado de Koutras DA. Subclinical hypothyroidism. En G. Hennemann, E.P.
Krenning, Thyroid International Merck KGaA, Darmstadt 1999 (3), 6-9

HIPOTIROIDISMO
PRIMARIO:
congnito: agenesia
autoinmune
postablativo
tiroiditis subaguda
deficiencia de yodo

SECUNDARIO
tumor hipofisiario
iatrognico

TERCIARIO

87.- Masculino de 42 aos alcohlico con cirrosis y ascitis. Es hospitalizado por agitacin
y comportamiento extrao. De los siguientes hallazgos el ms til para hacer el
diagnstico de encefalopata heptica es:

a)
b)
c)
d)

Ictericia
Asterixis de las manos
Hemangiomas superficiales
Signo de la ola positivo

ENCEFALOPATA
Alteracin en el estado mental, en el comportamiento y en el sueo que progresa a
la desorientacin y al coma.
Indica una insuficiencia heptica severa.
Fisiopatologia
Las toxinas son inactivadas por el hgado pero entran a la circulacin
portal por los cortos circuitos.
Toxinas no son inactivadas y excretadas.
Las toxinas se encuentran elevadas en el lquido cerebroespinal.

TOXINAS.
Amonio.

Neuroexcitatorio y depresor del sistema nervioso central.


Aminocidos aromticos.
Son precursores de neurotransmisores.
Si incrementan en sangre la encefalopata no progresa.
cido gammaaminobutrico (GABA).
Incrementado significativamente en la encefalopata.
Benzodiacepinas endgenas.
Compiten por los receptores de GABA y barbitricos.

CUADRO CLINICO
El diagnstico de la encefalopata heptica depende de la existencia de una enfermedad
heptica, desde la insuficiencia heptica aguda y la toxicidad por frmacos, hasta las
enfermedades crnicas, tales como la hepatropata alcohlica o la cirrosis.
En la historia clnica se hace nfasis especial en los antecedentes de enfermedades del
hgado diagnosticadas con anterioridad, una historia de alcoholismo o de hepatitis.
Transfusiones de sangre previas o abuso de drogas por va intravenosa pueden sugerir
una hepatitis crnica B o C. La utilizacin de frmacos hepatotxicos, como la metildopa,
la nitrofurantoina o la isonicida, puede ser causa de una hepatopata crnica, mientras
que una dosis alta de paracetamol puede ser causante de una necrosis hepatocelular
fulminante.
El cuadro clnico incluye tres elementos: cambios en el estado mental, hedor heptico y
asterixis. El hedor heptico se refiere al aliento ftido del paciente. La asterixis es un
temblor por sacudidas, irregular y bilateral de las manos, debido a una interrupcin
momentnea y brusca del tono muscular de los antebrazos.
El examen fsico se centra en la bsqueda de los estigmas caractersticos de las
enfermedades hepticas: ictericia, nevus en araa, ginecomastia, atrofia testicular, venas
distendidas en la pared abdominal (cabeza de medusa) y ascitis.
La exploracin neurolgica durante los estados precoces suele mostrar apraxia de
construccin y dificultad para escribir. Otros hallazgos fsicos son la rigidez de las
extremidades e hiperreflexia. En el estado de coma profundo suele haber prdida del
tono muscular y disminucin de los reflejos tendinosos profundos.

RESUMEN:
Petequias.
Rinofima.
Contractura de Dupuytren (aponeurosis palmar).
Telangiectasias en el tronco.
Ascitis.
Asterixis.
Eritema palmar.
Atrofia testicular.

Blibliografa
Shakelfords. Surgery of the alimentary tract. 5a. Ed. 2002. Tomo 3.
Feldmans. Gastroeneterology. 2002.
Perez. Anatoma y fisiologa del hgado. Univ. Catlica de Chile. 2005.
Bratiz. Serum laboratory test in cirrhosis. Journal of Hepatology. Slovakia. 2005.
Paradis. Glycomics. Journal of hepatology. Ireland. Agosto 2005.

88.- Se trata de masculino de 43 aos de edad acude a consulta por presentar nausea y
vmito, aumento en el nmero de evacuaciones de consistencia aguada, (diarrea sin
productos patolgicos), afebril, T.a 110.70 mmhg, refiere que su hijo present los misma
sintomatologa casi al mismo tiempo que el, como antecedente refiere haber comido
juntos en un restaurante hace aproximadamente 4 hrs. De los siguientes microorganismos
el de mayor probabilidad de ocasionar ste cuadro es:

a) Salmonella enteriditis.
b) Shigella sonnei.
c) Staphylococcus aureus.
d) E. Coli

La contaminacin de alimentos por S. aureus, est asociada con una forma de


gastroenteritis que se manifiesta clnicamente por un cuadro caracterizado por vmitos
(76% de casos) y diarrea (77% de casos). El corto perodo de incubacin de 1-6 horas
orienta a la sospecha de enfermedad producida por ingestin de una o ms enterotoxinas
preformadas en el alimento que ha sido contaminado con cepas de S. aureus productor
de la misma.
Son raramente observados signos de toxicidad sistmica, tales como fiebre e hipotensin
En general, es un cuadro autolimitado que tpicamente se resuelve en 2448 horas desde el inicio.
No est claro si se desarrolla en humanos inmunidad a largo plazo, pero anticuerpos
frente a una SE no necesariamente confieren inmunidad frente a la intoxicacin por S.
aureus, ya que existe mltiples SE capaces de producir enfermedad. En algunos casos,
anticuerpos producidos frente a una SE confieren proteccin cruzada contra otra SE, ya
que algunas comparten eptopes.
Todas las SE son capaces experimentalmente en primates de producir emesis, y no se
registra enterotoxemia, excepto en dosis muy altas, probablemente debido a su dificultad
para atravesar mucosas.
El 99% de casos de intoxicacin alimentaria por enterotoxinas
Estafilocccicas est asociado a S. aureus y ocasionalmente se reportan casos por
Staphylococcus epidermidis.
Las cepas estafilocccicas enterotoxignicas aisladas de alimentos implicados en brotes
de infeccin son mas a menudo lisadas por fagos del grupo

lll, y menos frecuentemente simultneamente por los grupos l y lll.


Tratamiento.
Como para la mayora de enfermedades trasmitidas por alimentos autolimitadas, las
medidas de sostn son la base del tratamiento. No est indicado tratamiento con
antimicrobianos.
Referencias bibliogrficas.
- Manual of Clinical Microbiology. Murray, P. 1995 6th edition.
- Principles and Practice of Infectious Diseases. Mandell, Douglas, Bennett.
1995. 4th edition.
- Dinges M, Orwin P, Schlievert P. 2000. Exotoxins of Staphylococcus aureus.
Clinical Microbiology Reviews, vol 13; 16-34

89.- Se trata de paciente femenino de 6 aos de edad la cual presenta ceguera nocturna
(hemeralopia) en la exploracin oftalmolgica se observa constriccin del campo visual
con escotoma anular, prdida de la agudeza y electrorretinograma anmalo asi mismo
arterias estrechadas Cul de las siguientes patologas es la ms probable?

a)
b)
c)
d)

Retinosis pigmentaria (retinitis pigmentaria).


Retinoblastoma.
Catarata congnita o infantil.
Persistencia de vtreo primario

Retinosis pigmentaria. Bilateral, de curso lento y progresivo, comienza en la edad


escolar, pudiendo causar ceguera hacia los 40 aos. Es una alteracin de los bastones.
Puede presentarse aislada o asociada a otras malformaciones, como la polidactilia, el
sndrome de Laurence-Moon-Bield.
Se hereda bajo diferentes patrones. Clnicamente presentan mala visin nocturna
(hemeralopa) y escotoma anular en el campo visual .Oftalmoscpicamente se aprecian
arterias estrechadas, atrofia de papila y acumulaciones de pigmento que asemejan
osteocitos en la retina perifrica. Evolucionan hacia una reduccin del campo visual y
finalmente ceguera. Adems presentan complicaciones adicionales como aparicin
temprana de cataratas y glaucoma.

Retinosis pigmentaria

Retinosis pigmentaria, preguntas y respuestas, Universidad Miguel Hernndez de Elche,


2007
Science Daily (ed.): Retina Transplants Show Promise In Patients With Retinal
Degeneration (11-7-2008).

90.- Paciente masculino de 42 aos, con antecedente de alcoholismo, diabtico,


hospitalizado por TCE. Durante su estancia intrahospitalaria inicia con fiebre en picos
(38.5-39 C), escalofros, aparicin de soplo y la biometra hemtica muestra leucocitosis
(16,300) se descarta IVU y neumona. Se sospecha endocarditis infecciosa. La conducta
inicial es:
a)
b)
c)
d)

Ecocardiograma transtorcico
Hemocultivo a travs de catter
Tomar hemocultivos seriados
Retiro de catter intravascular

La endocarditis infecciosa es una infeccin microbiana que se localiza sobre las vlvulas
cardacas o sobre el endocardio mural. A pesar que la mayora de estas infecciones son
causadas por bacterias, se considera ms apropiado denominarla en forma global como
endocarditis infecciosa debido a que tambin puede ser producida por hongos, rickettsias
o clamidias.
La endocarditis bacteriana es la forma ms reconocida y se clasifica en aguda o
subaguda segn su presentacin clnica. La endocarditis mictica se detecta casi
exclusivamente en drogadictos o en pacientes con prtesis valvulares.
Hemocultivos con antibiograma
Se recomiendan tomar tres muestras a la llegada del paciente (preferiblemente sin tomar
antibiticos), las muestras de sangre separado por un intervalo mnimo de 1 h, en un
perodo de 24 horas; al da siguiente debe repetirse el mismo proceder. Debe realizarse
una correcta antisepsia previa de la piel y cambiar de aguja cuando se vaya a verter la
sangre en los frascos. El cultivo de sangre se realiza en un tubo especializado para
hemocultivos automatizados.
Se prefiere dejar al paciente, si el estado clnico as lo tolerara, hasta un mnimo de 72
horas sin antibiticos, antes de tomar la muestra, pero si el paciente por su situacin de
sepsis avanzada y repercusin hemodinmica no se le puede retirar el antibitico
entonces entonces se utiliza el frasco de hemocultivo automatizado con concentracin
mnima inhibitoria del antibitico, precisando en la orden el los tipos de antibiticos que
se estn empleando en la teraputica antimicrobiana.

Si al realizar los hemocultivos automatizados estos todos vienen negativos a pesar de


que la sintomatologa del paciente no mejora se pueden indicar los hemocultivos
automatizados con medios de cultivos especiales para grmenes de crecimiento lento, por
ejemplo en medio de Sabureao.
A estos frascos se le aade sangre venosa, entre 5-10 mL en adultos. Se llevar la
muestra lo ms rpido posible al laboratorio de Microbiologa.
La toma de muestra se realizar independientemente de la temperatura del paciente o
cuando se presentan los escalofros y no en pico febril.
Para confirmar criterio de curacin de la infeccin previo al egreso hospitalario se
realizan 6 hemocultivos automatizados una semana despus de suspendidos los
antibiticos.

La endocarditis infecciosa (EI) es una enfermedad en la que uno varios microorganismos infectan el endocardio, las vlvulas las estructuras relacionadas,
generalmente sobre una lesin ante-rior: cardiopata reumtica en otros tiempos,
cardiopatas congnitas en la actualidad. Puede ser sbita y aguda, pero ms
frecuentemente es subaguda y larvada lo que retrasa el diagnstico. Tiene morbilidad y
mortalidad impor-tante a pesar de los tratamientos antimi-crobianos y de la difusin de su
profi-laxis entre los nios susceptibles. Nue-vos grupos de riesgo son los pacientes
sometidos a ciruga cardiaca, los nios que precisan catteres intravasculares,
especialmente neonatos, los inmunode-primidos y los adictos a drogas por va
intravenosa. La ecocardiografa- Dop-pler aporta actualmente criterios bsicos para el
diagnstico y seguimiento. Pue-den ser tiles a nios nuevos recursos en el tratamiento y
una profilaxis precisa.

91.- Masculino de 33 aos con presencia de desarrollo de manchas lenticulares de color


caf, rosado y blanco, levemente descamativas, se diagnostica micosis crnica
asintomtica por Malassezia furfur. La localizacin anatmica ms frecuente de esta
patologa es:
a)
b)
c)
d)

Dorso de manos y pies.


Codos y rodillas
Pecho y espalda.
Cara y cuello.

La pitiriasis versicolor (PV) es una micosis superficial crnica, caracterizada por mculas
hipo o hiperpigmentadas levemente descamativas. El trmino versicolor se refiere al
color variable de las lesiones que pueden ser caf,rosadas o blancas. Esta enfermedad
tiene distribucin mundial y es uno de los desrdenes ms comunes en la pigmentacin
de la piel. La mayor prevalencia y el mayor porcentaje de recurrencias se observan en
reas clidas y hmedas.
Eichstedt, en 1846, fue el primero en reconocer la naturaleza fngica de la PV, al describir
un hongo asociado a esta afeccin. Sin embargo, el gnero Malassezia, con Malassezia

fufur (M. furfur) como especie tipo, fue creado por Baillon medio siglo despus, en 1889,
en honor a Luis Malassez.
Pitiriasis versicolor, a veces denominada Tinea versicolor, es uno de los trastornos de
pigmentacin ms comunes en el mundo. Es una micosis crnica, leve y usualmente
asintomtica.
Se produce por el crecimiento de la levadura lipoflica Malassezia furfur en la capa ms
superficial de la piel o estrato crneo.

La infeccin se caracteriza por el desarrollo de manchas o mculas lenticulares de color


caf, rosado o blanco, levemente descamativas, en tronco y brazos.

BIBLIOGRAFA
1. Ashbee HR, Evans E.G. Inmunology of Diseases Associated with Malassezia species.
Clin Microbiol Rew. 2002;21-57.
2. Crespo Erchiga V, Delgado Florencio V. Malassezia species in skin diseases. Curr
Opinin Infect Dis, 2002, 15:133-142.
3. Guho E, Boekhout T, Ashbee HR, Guillot J, Van Belckum A, Faergeman J. The role of
Malassezia species in the ecology of
human skin and as pathogen. Med Mycol 1998;36 (supp1):220-229.
4. Crespo Erchiga V, Ojeda Martos A, Vera Casao A, Crespo Erchiga A, Snchez
Fajardo F, Guho E. Mycology of pityriasis
versicolor. J. Mycol. Med. 1999; 9:143-148.
5. Katoh T, Irimajiri J. Pityriasis versicolor and Malassezia folliculitis. Nippon Ishinkin
Gakkai Zasshi. 1999;40:69-71.
6. Gupta AK, Batra R, Bluhm R, Boekhout T, Dawson T. Skin diseases associated with
Malassezia species. J Am Acad Dermatol
2004; 51:785-798.

92.- A 38-year-old man comes to the physician because of slowly progressive visual
problems that make him bump into objects on both sides. He also reports that, while
driving, he has trouble switching lanes because he needs to turn his head all the way
backward to look for other cars. Ocular examination shows bitemporal field loss with
preserved visual acuity.Examination of the fundus is unremarkable. Which of the following
is the most likely diagnosis?

a)
b)
c)
d)

Pituitary adenoma
Occipital lobe meningioma
Optic neuritis
Retinal detachment

. PRESENTACIN CLNICA
Los tumores hipofisarios se reconocen clnicamente por uno o ms de tres patrones de
presentacin muy constantes:
- Sntomas de hipersecrecin hipofisiaria
- Sntomas de hiposecrecin hipofisaria
- Sntomas neurolgicos
El tercer patrn de presentacin es el dominado por los sntomas neurolgicos, aislados o
coexistentes con una o varias de las alteraciones endocrinolgicas antes descritas. Como
se ha dicho, una masa hipofisaria progresivamente creciente generar una constelacin
de signos y sntomas neurolgicos que dependern de la trayectoria del crecimiento y de
las estructuras nerviosas vecinas que resulten alteradas. Los sntomas y signos a
encontrar son:
a. Cefaleas: Pueden ser un signo precoz y se atribuyen al estiramiento de la duramadre
que recubre la hipfisis o del diafragma de la silla turca. Est presente en el 75% de los
casos y se localiza ms frecuentemente en la regin frontal y orbital.
b. Prdida de visin: Debido a la compresin de los nervios pticos o del quiasma
ptico. El patrn clsico de prdida visual es una hemianopsia bitemporal a menudo
asociada con disminucin de la agudeza visual. Posteriormente puede haber una ceguera
completa de uno o ambos ojos.
c. Manifestaciones hipotalmicas: Debido a la compresin del hipotlamo por grandes
adenomas hipofisarios que provocan:
- Alteraciones del sueo

- Alteraciones de la atencin
- Alteraciones de la conducta
- Alteracin de la alimentacin
- Alteracin de las emociones
d. Hidrocefalia obstructiva: Debido a la infiltracin de las lminas terminales que hace
que el tumor penetre en la regin del III ventrculo, ocasionando obstruccin a la salida de
LCR.
e. Compromiso de nervios craneales. Debido a la extensin lateral del tumor hacia la
regin del seno cavernoso, por donde transcurren los nervios craneanos. As las
manifestaciones ms caractersticas son:
- Ptosis: Por compromiso del III nervio craneal (motor ocular comn).
- Dolor o alteraciones sensitivas faciales. Por compromisos de las ramas oftlmica y
maxilar superior del Nervio Trigmino.
- Diplopia: Por compromiso del III, IV y VI nervios craneales.

Bibliografa
1. Kovacs K, Horvath E, Vidal S: Classification of pituitary adenomas. J Neurooncol
54 (2): 121-7, 2001.
2. Ironside JW: Best Practice No 172: pituitary gland pathology. J Clin Pathol 56 (8):
561-8, 2003.
3. Scheithauer BW, Kovacs KT, Laws ER Jr, et al.: Pathology of invasive pituitary
tumors with special reference to functional classification. J Neurosurg 65 (6): 73344, 1986.
4. Ezzat S, Asa SL, Couldwell WT, et al.: The prevalence of pituitary adenomas: a
systematic review. Cancer 101 (3): 613-9, 2004.

93.- Se trata de masculino de 49 aos quien posterior a realizar estudios de laboratorio y


gabinete se determina como portador de Isosporosise, el tratamiento de eleccin para
ste agente es:
a) Trimetoprim Sulfametoxazol
b) Albendazol
c) Metronidazol
d) Pentamidina

El tratamiento ms efectivo contra la isosporiasis consiste en una combinacin de dos


medicamentos: trimethoprim y sulfamethoxazole (TMP-SMX, Bactrim, Septra). Para tratar
la isosporiasis, se toman dos pastillas de TMP-SMX de doble potencia dos veces al da.
Otra posibilidad es tomar una pastilla de doble potencia tres veces al da. El tratamiento
con TMP-SMX suele continuarse durante dos a cuatro semanas.

Referencia Bibliogrfica:
Ttulo Del Libro: Gua De Antimicrobianos, Antivirales, Antiparasitarios, Antimicticos E
Inmunomod.
Autor Principal: Dr. Napolen Gonzlez Saldaa
Editorial: Mc Graw Hill Interamericana
Edicin: 6.
Ao: 2004
Pginas 257-278

Se han usado muchos agentes para tratar las infecciones por I. belli. Las combinaciones
de inhibidores de la dihidrofolato reductasa timidilato sintetasa, como el trimetropim (TMP)
o la pirimetamina, con sulfonamidas como el sulfametoxazol (SMX), sulfadiazina o
sulfadioxina son de probada eficacia, siendo el cotrimoxazol (TMP-SMX) el tratamiento de
eleccin. El uso de cotrimoxazol para el tratamiento o prevencin de la neumona por
Pneumocystis carinii previene la adquisicin de la primoinfeccin por I. belli o las
recrudescencias de la infeccin. La pirimetamina sola tambin es eficaz en pacientes con
alergia a las sulfonamidas
CAMARENA JJ, BORRS R, GARCA DE LOMAS J. Coccidios intestinales.
Cryptosporidium e Isospora. En: Perea EJ. Enfermedades Infecciosas y
Microbiologa Clnica. Vol. II. Ediciones Doyma, Barcelona 1992, pp 1027-1033.

94.- Un sujeto de 30 aos de edad adicto a las drogas intravenosas presenta debilidad del
hemicuerpo derecho y cefalea en un periodo de dos das. La exploracin revela un
individuo mal nutrido, afebril con hemiparesia derecha leve. De los siguientes el
diagnstico ms probable es:

a)
b)
c)
d)

Endocarditis bacteriana
Meningitis por virus de la inmunodeficiencia humana (VIH)
Absceso cerebral
Meningitis criptoccica.

Los adictos a drogas intravenosas estn propensos a sufrir bacteriemia, que a su vez
puede producir absceso cerebral y disfuncin neurolgica progresiva. Los pacientes con
aqul, por lo comn se encuentran afebriles a no ser que haya endocarditis acompaante
u otro origen endovascular de infeccin. Los adictos a drogas intravenosas tienden a
presentar endocarditis bacteriana y pueden cursar con dficit neurolgicos en forma
apopltica debido a embolia sptica del cerebro. Sin embargo, por lo regular tienen fiebre.
La meningitis por VIH produce cefalea y datos de irritacin menngea, pero no se presenta
dficit neurolgico focal. La meningitis criptoccica se manifiesta con conducta alterada y
cefalea y los pacientes estn afebriles. Sin embargo, es raro que haya datos de
seudoapopleja. Por ltimo, el uso de drogas intravenosas puede provocar embolia de
cuerpo extrao pero con problemas neurolgicos apoplticos. Un mbolo puede llegar al
cerebro por un cortocircuito cardaco de derecha a izquierda o bien por una malformacin
arteriovenosa pulmonar si la inyeccin es venosa. El mbolo puede entrar a la circulacin
cerebral de manera directa en caso de inyeccin intracarotdea.
Referencias
Nath A. Brain abscess and parameningeal infections. In: Goldman L, Ausiello D, eds. Cecil
Medicine. 23rd ed. Philadelphia, Pa: Saunders Elsevier; 2007: chap 438
Allen R. M. MMS Medicina Interna. 5. Edicin. National Medical Series. Mc. Graw Hill.
2006. (Captulo 11 XVI B 1-2).

95.- Se trata de masculino de 41 aos de edad, diagnosticado por trastorno obsesivocompulsivo. Los actos obsesivos ms frecuentes en este trastorno son:
a)
b)
c)
d)

Recuentos mentales.
Evitar pisar las cruces de las baldosas.
Acumulacin y coleccin de objetos.
Comprobaciones y rituales de limpieza.

Sntomas del TOC


Obsesiones

Las obsesiones son ideas o impulsos no deseados que aparecen repetidamente en la


mente de la persona que padece TOC. Los pacientes suelen tener miedo a sufrir dao
ellos mismos, o alguien al que quieren, se preocupan irracionalmente por no
contaminarse, o tienen una necesidad excesiva de hacer las cosas correctamente o con
perfeccin. Una y otra vez, la persona piensa algo que le inquieta, como por ejemplo, "Mis
manos pueden estar contaminadas--debo lavarlas" o "Puedo haber dejado el gas abierto"
o "Estoy hacindole dao a mi hijo." Estos pensamientos angustiosos se inmiscuyen con
los otros pensamientos del paciente, y causan ansiedad. A veces, las obsesiones son de
carcter violento o sexual, o tienen que ver con enfermedades.
Compulsiones
En respuesta a sus obsesiones, la mayora de las personas con TOC recurren a
comportamientos repetitivos llamados compulsiones. Los ms frecuentes son los rituales
de limpieza y las comprobaciones. Otros comportamientos compulsivos incluyen
recuentos (a menudo al mismo tiempo que se realizan otras acciones compulsivas, tales
como lavarse las manos), hacer repeticiones, acaparamiento, y recolocaciones sin fin de
objetos en un esfuerzo para mantenerlos perfectamente alineados. Tambin son
corrientes los problemas mentales, tales como repetir frases mentalmente y hacer listas.
Estos comportamientos, en general, tienen por objeto proteger de peligros a la persona
que padece TOC, o a los otros.
Algunas personas con TOC tienen rituales establecidos; otros tienen rituales que son
complejos y cambiantes.
El ejecutar estos rituales slo proporciona alivio temporal de la ansiedad, pero no hacerlos
incrementa la ansiedad de la persona.

96.- La mejor medida para calcular la probabilidad de muerte en pacientes diagnosticados


con sndrome pulmonar por hantavirus es:
a)
b)
c)
d)

Prevalencia
Tasa de letalidad
Mortalidad
Tasa de incidencia

La tasa de letalidad es la medida que representa la probabilidad de muerte entre los


casos diagnosticados o el poder que tiene una enfermedad para producir la muerte.
Morton R. F. Bioestadstica y Epidemiologa, Interamericana, 3. Ed. 1993; pg: 23

97.- Masculino de 62 aos con antecedentes de insuficiencia cardiaca de 2 aos de


evolucin, actualmente se presenta a consulta por presentar edema en MPS. Una de las
siguientes puede ser la causa de un edema no inflamatorio:

a)
b)
c)
d)

Un aumento de la permeabilidad vascular.


Un aumento de la presin hidrosttica intravascular.
Una eliminacin excesiva de sal y agua por el rin.
La disminucin de la presin hidrosttica intravascular.

Edema
Acumulo de un exceso de lquido en el espacio tisular intercelular (intersticial) o en las
cavidades del organismo.
El edema localizado: se puede producir en el caso de una obstruccin del flujo
seroso (producindose la inflamacin o hinchazn de una pierna por ejemplo). Tambin
puede ser causado por un proceso infeccioso (como los abscesos causados por
Staphylococcus aureus).
Edema generalizado: es de carcter sistmico como en el caso de insuficiencia
cardiaca o de sndrome nefrtico (que se caracteriza por una intensa proteinuria
secundaria a la permeabilidad glomerular anmala). Cuando el edema es intenso y
generalizado de tal forma que provoca hinchazn difusa de todos los tejidos y rganos de
la economa, especialmente a nivel del tejido subcutneo, se denomina Anasarca.
Exudado: lquido inflamatorio extravascular rico en protenas, detritus celulares y
leucocitos. Se trata de un acumulo de lquido que se encuentran rico en restos celulares,
leucocitos, restos protenicos y por lo general presenta una r>1.020. Se debe a un
incremento en la permeabilidad endotelial con la salida de protenas plasmticas
(principalmente albmina). Se presenta por infecciones pigenes, TB, etc. (de origen
inflamatorio).
Trasudado: en el lquido de edema de origen no inflamatorio y est relacionado
con los casos de insuficiencia cardiaca y nefropatas por lo general no hay (es pobre)
protenas, ni leucocitos, con una r1.012. nicamente es el acumulo de lquidos.

El trastorno hemodinmico (trasudado) se presenta cuando se altera la Ley de


Starling que es el equilibrio normal de los lquidos (intravascular y extravascular). Se
mantiene por la accin de dos grupos de fuerzas opuestas, los que hacen que el lquido
tienda a salir de la circulacin son la presin osmtica del lquido intersticial y la presin
hidrosttica intravascular; los que hacen que el lquido pase a la circulacin son la
presin osmtica de las protenas plasmticas (principalmente albmina) (presin
coloidosmtica) y la presin hidrosttica tisular. Por lo tanto el equilibrio entre estas
fuerzas es tal que en los capilares musculares perifricos existe un movimiento neto de
lquidos hacia fuera, pero ste lquido es drenado a los linfticos, por lo que no se produce
edema.
Los factores que aumentan la presin hidrosttica intravascular o disminuye la
presin coloidosmtica intravascular dan lugar al aumento de la salida del lquido desde
los capilares, con el consiguiente trastorno hemodinmico el edema.
Nota: en la lesin (por falta de la Angiotensina, que regula las sales) por prdida de
protenas (principalmente albumina) (o sea la proteinuria) se produce el Edema
generalizado (Anasarca).

Fisiopatologa del edema

Autores: Jos Mara Sillero Fernndez de Caete


Localizacin: Seminario mdico, ISSN 0488-2571, Vol. 49, N. 1, 1997 , pags. 7486

Organizacin Panamericana de la Salud


Programa de Publicaciones (DBI/E)
525 Twenty-third Street, NW

98.- Masculino de 37 aos acude a consulta porque tiene dolor sordo perineal y
persistente, disuria de 6 meses de evolucin. Niega IVUS y descarga uretral. Su
temperatura es de 37C . Al examen rectal la prstata es ligeramente dolorosa, pero no
esta aumentada de tamao ni indurada. Uroanlisis normal. Secrecin prosttica, muestra
30 leucos por campo sin bacterias. Cultivos de secrecin prosttica y orina negativos. El
diagnstico ms probable es:

a)
b)
c)
d)

Cistitis aguda
Prostatitis aguda
Prostatitis bacteriana crnica
Prostatitis no bacteriana crnica

La Prostatitis es uno de los ms comunes problemas urolgicos. Cerca del 50% de los
hombres experimentan un episodio de prostatitis una vez en la vida.
Debido a que la Hiperplasia Prosttica Benigna (HPB), el Cncer de Prstata (CP) y la
Prostatitis Crnica Abacteriana (PCA) pueden coexistir, es difcil distinguir entre estas
entidades.
Los sntomas de PCA y de HPB se superponen de tal forma que muchos ancianos
reciben equivocadamente ste ltimo diagnstico. Aunque a diferencia de la HPB y el CP
que son preferentemente del hombre anciano, la PCA puede presentarse a cualquier
edad. La Prostatitis puede elevar los niveles de Antgeno Prosttico Especfico (PSA) lo
que conduce a la realizacin de un gran nmero de biopsias prostticas innecesarias.
En la Prostatitis Crnica se desconoce no slo qu la causa sino cul es la mejor forma
de manejarla.
Drach y col clasificaron y definieron a la Prostatitis de la siguiente forma:
* Prostatitis Bacteriana Aguda: se define por la recuperacin de bacterias del fluido
prosttico, fluido purulento y sntomas sistmicos de infeccin (fiebre, mialgia , etc.)
* Prostatitis Bacteriana Crnica: recuperacin de bacterias en nmero significativo del
fluido prosttico en ausencia de infeccin urinaria o signos significativos de infeccin
sistmica.
* Prostatitis Abacteriana: sin recuperacin de nmero significativo de bacterias del fluido
prosttico
pero
con
pus
microscpica
en
el
mismo
* Prostatodinia: sin recuperacin de bacterias o pus en el lquido prosttico, pero el
paciente tiene urgencia urinaria persistente, disuria, discomfort prosttico.
El National Institute of Health (NIH) de USA estableci en 1995 un nuevo sistema de
clasificacin. Es el siguiente:
* Prostatitis Tipo I: infeccin aguda de la prstata. Corresponde a la Prostatitis bacteriana
aguda de la antigua clasificacin.

* Tipo II: infeccin recurrente de la prstata. Corresponde a la Prostatitis Bacteriana


Crnica
* Tipo III: Sin infeccin demostrable. Corresponde a Prostatitis Crnica
Abacteriana/Sindrome de Dolor plvico crnico. Tipo III A: leucocitos en semen. Tipo III B:
Sin leucocitos en semen.
* Tipo IV: sin sntomas subjetivos, detectado por biopsia prosttica o por la presencia de
leucocitos en secreciones prostticas. Corresponde a la Prostatitis inflamatoria
asintomtica.
La etiologa de la Prostatitis es desconocida en el 90% de los casos, siendo bacteriana en
el 10% de los casos. La literatura antigua describe como determinantes potenciales de
esta condicin los niveles de hormonas sexuales, dieta, enfermedades del tracto urinario,
stress, factores psicolgicos, alergia. Estudios ms recientes examinaron la edad,
etnicidad, agentes infecciosos, niveles de cido rico, actividad sexual, clculos y quistes
prostticos, citoquinas proinflamatorias y biopsia prosttica.
Bibliografa
1- McNaughton C. Diagnosis and Treatment of Chronic Abacterial Prostatitis: a Systematic
review. Ann Intern Med 2000 ; 133 : 367-381
2- Leskinen M , Lukkarinen O, Marttila T. Effects of finasteride in patients with
inflammatory chronic pelvic pain syndrome: a double blind, placebo-controlled, pilot study.
Urology 1999 ; 53 : 502-505

99.- Masculino de 3 aos, ingresa al servicio urgencias 30 mins. Despus de haber


ingerido comprimidos de sulfato ferroso de 325 mg. La madre indica que faltan 20
comprimidos del frasco. Cada comprimido tiene 65 mg de hierro elemental. El nio ha
vomitado una vez y su comportamiento parece totalmente normal. Los hallazgos de la
exploracin fsica no arrojan nada importante. Peso 13kg. Cul de los siguientes
enunciados sobre el estado del nio es el correcto?
a)
b)
c)
d)

La dosis de hierro no debera causar secuelas clnicas.


Ese episodio nico de vmito probablemente no esta relacionado con la ingestin.
Aunque ha vomitado una vez debera administrarse ipecacuana.
Los catrticos son ineficaces en intoxicaciones.

En situaciones en las que no hay testigos de la ingestin debe presuponerse inicialmente


el peor escenario posible para estimar la toxicidad potencial. La dosis mxima de hierro
ingerida por este nio es de 65 mg de hierro elemental x 20 pldoras / 13 kg = 100 mg /kg
que es una dosis potencialmente grave. Entre los sntomas causados por la ingestin
grave de hierro estn los vmitos por tanto en este caso el vmito debe considerarse
relacionado con la ingestin. Ello implica un riesgo potencial de secuelas graves. En
conjunto la ingestin debe valorarse como clnicamente importante.
Como un solo episodio de vmitos no vaca el estmago de manera suficiente debe
administrarse ipecacuana o alternativamente proceder a un lavado gstrico. El carbn
activado no adsorbe el hierro y no es necesario. Los catrticos sirven de ayuda una vez
que e ha vaciado el estmago. Los niveles de hierro srico y de capacidad de enlace del
hierro deben estimarse segn las circunstancias. Tambin es til obtener una radiografa
de abdomen por cuanto las pldoras son radioopacas y puede deducirse as algn indicio
de su permanencia en el tracto GI despus del tratamiento inicial.

1.- Montoya-Cabrera MA. Intoxicaciones y envenenamientos en nios. Mxico,


Intersistemas , 2000.
2.-Montoya CMA. Toxicologa clnica. 2. Ed, Mxico, Mndez Editores, 1997

100.- Masculino que se presenta a consulta externa, refiere que su pareja ha sido
diagnosticada por infeccin de virus del papiloma. La forma ms frecuente de la
infeccin por papiloma virus en varones es?
a)
b)
c)
d)

Verrugas inguinales
Verrugas larngeas
Subclnica en genitales
Condilomas prepuciales

En diferentes estudios de varones compaeros de mujeres con lesiones cervicales por


virus del papiloma humano se demostr que el 88% tenan signos histolgicos de
condilomas, de los cuales el 72% eran en forma subclnica es decir no haba ningn tipo
de lesin visible o sntoma. Otros estudios han demostrado que hasta el 66% o mas de
los compaeros de mujeres con neoplasias intraepiteliales cervicales tienen infecciones
subclnicas por virus del papiloma humano en el pene.

Obstet Gynecol 2001;185:308-17.


Kurtycz DFI, Hoerl HD. Thin-Layer Technology: Tempered Enthusiasm. Diag Cytopath
2000;23(1):1-5.

S-ar putea să vă placă și